Download as pdf or txt
Download as pdf or txt
You are on page 1of 201

The past

pappers
From passtest website

Prepared by
Dr. Hani Abdelfattah Hashem
february 2022
8/25/22, 9:29 PM MyPastest

0:00:00/03:00:00

A 32-year-old female nurse is reviewed in the Occupational Health Department 24 hours after a needlestick
injury from a man who is known to be hepatitis B surface (HBs) antigen positive. Her anti-HBs antibody titre
is < 10 three months after her last booster jab.

What is the most important intervention?

A Hepatitis B immunoglobulin

B Hepatitis B polymerase chain reaction (PCR) in three months

C Hepatitis B vaccine booster

D Oral entecavir for four weeks

E Oral telaprevir for four weeks

Explanation 

A Hepatitis B immunoglobulin

With significant exposure to hepatitis B from a hepatitis B surface (HBs) antigen-positive source, the risk of
infection may be as high as 30%. Hepatitis B immunoglobulin is therefore warranted. The patient should also
be offered a further hepatitis B vaccine booster.

B Hepatitis B polymerase chain reaction (PCR) in three months

With a needlestick injury from a HBs antigen-positive patient, the risk of infection may be as high as 30%,
given she has a very low antibody titre.

C Hepatitis B vaccine booster

Although a hepatitis B vaccine booster is recommended for this patient, it is not the most important
intervention because it will take some time to become effective. Immunoglobulin is a much more important
intervention.

D Oral entecavir for four weeks


https://mypastest.pastest.com/Secure/TestMe/TimedBrowser/1098274 1/3
8/25/22, 9:29 PM MyPastest

Entecavir is a nucleoside analogue that can be used in the treatment of hepatitis B. It does not, however, have
a role in post-exposure prophylaxis.

E Oral telaprevir for four weeks

Telaprevir is an NS3/4a protease inhibitor which specifically targets viral replication of hepatitis C. It does not
have a role in post-exposure prophylaxis of hepatitis B.
73164

Rate this question:

Next Question

Previous Question Tag Question

Feedback End Review

Difficulty: Average

Peer Responses %

Q. Answered Flagged

Q1

Q2

Q3

Q4

Q5

Q6

https://mypastest.pastest.com/Secure/TestMe/TimedBrowser/1098274 2/3
8/25/22, 9:29 PM MyPastest

0:00:00/03:00:00

A 23-year-old man complains of intermittent diarrhoea, bloating and abdominal pain for the past three
weeks, having returned from a hiking holiday in Vietnam. He tells you he has lost three kilograms in weight.
There is mild abdominal tenderness on abdominal palpation.

Investigation Result Normal value

Haemoglobin (Hb) 140 g/l 115–155 g/l

White cell count (WCC) 7.0 × 10 9/l 4–11 × 10 9/l

Platelets (PLT) 181 × 10 9/l 150–400 × 10 9/l

Sodium (Na +) 143 mmol/l 135–145 mmol/l

Potassium (K +) 3.5 mmol/l 3.5–5.0 mmol/l

Creatinine (Cr) 95 µmol/l 50–120 µmol/l

C-reactive protein (CRP) 45 mg/l < 10 mg/l

What is the most likely diagnosis?

A Chronic pancreatitis

B Giardiasis

C Irritable bowel syndrome

D Microscopic colitis

E Salmonellosis

Explanation 

B Giardiasis

Giardia lamblia is a protozoan which is transmitted via the faecal-oral route and is endemic in many regions of
the world, including East Asia. It leads to symptoms similar to those of irritable bowel syndrome and
malabsorption over time. It can be treated most effectively with a single dose of two grams tinidazole and

https://mypastest.pastest.com/Secure/TestMe/TimedBrowser/1098274#Top 1/3
8/25/22, 9:29 PM MyPastest

most patients are treated empirically.

A Chronic pancreatitis

Chronic pancreatitis is associated with abdominal pain and more severe symptoms of malabsorption than
those seen in this patient.

C Irritable bowel syndrome

Irritable bowel syndrome does present with symptoms of intermittent diarrhoea and abdominal bloating.
However, the elevated CRP level here raises the possibility of an infectious cause for this patient’s
presentation.

D Microscopic colitis

Microscopic colitis is associated with chronic watery diarrhoea and occurs in association with a number of
medications, including proton pump inhibitors.

E Salmonellosis

Salmonellosis is associated with acute symptoms, including fever, diarrhoea and abdominal pain. It does not
fit with the chronic picture seen here.
73166

Rate this question:

Next Question

Previous Question Tag Question

Feedback End Review

Difficulty: Average

Peer Responses %

https://mypastest.pastest.com/Secure/TestMe/TimedBrowser/1098274#Top 2/3
8/25/22, 9:29 PM MyPastest

0:00:00/03:00:00

An 18-year-old woman is admitted to the Emergency Department with nausea and vomiting. She works on a
show farm and has been unwell with diarrhoea a few days earlier. Her blood pressure is 135/90 mmHg, and
her heart rate is 90 bpm.

Investigation Result Normal value

Haemoglobin (Hb) 82 g/l (fragmented red cells seen on film) 115–155 g/l

White cell count (WCC) 7.3 × 10 9/l 4–11 × 10 9/l

Platelets (PLT) 61 × 10 9/l 150–400 × 10 9/l

Sodium (Na +) 140 mmol/l 135–145 mmol/l

Potassium (K +) 5.9 mmol/l 3.5–5.0 mmol/l

Creatinine (Cr) 298 µmol/l 50–120 µmol/l

What is the most likely diagnosis?

A Campylobacter gastroenteritis

B Haemolytic uraemic syndrome

C Idiopathic thrombocytopenic purpura

D Primary C3 deficiency

E Typhoid fever

Explanation 

B Haemolytic uraemic syndrome

This patient works on a show farm. Unfortunately, these establishments have been implicated in the spread
of Escherichia coli on a number of occasions over the years, often due to poor hand hygiene. E. coli infection
leading to haemolytic uraemic syndrome fits as the likely diagnosis, given the short episode of diarrhoea

https://mypastest.pastest.com/Secure/TestMe/TimedBrowser/1098274#Top 1/3
8/25/22, 9:29 PM MyPastest

initially, with fragmented red cells, low platelets (PLT) count and significant renal impairment seen on blood
testing. Management is supportive. Plasma exchange or complement inhibitor antibodies may be considered
in some patients.

A Campylobacter gastroenteritis

Campylobacter gastroenteritis is self-limiting in the majority of cases, with only a few, mainly elderly, patients
suffering from prerenal impairment.

C Idiopathic thrombocytopenic purpura

Idiopathic thrombocytopenic purpura is associated with a reduction in PLT count, but not with severe renal
impairment seen here.

D Primary C3 deficiency

Primary C3 deficiency predisposes patients to serious infections with capsulated bacteria. It does not increase
the risk of haemolytic uraemic syndrome, the diagnosis here.

E Typhoid fever

Typhoid fever is associated with headache, fever and a salmon pink rash. There may be a short episode of
diarrhoea at the beginning of the illness. It is also associated with relative bradycardia.
73192

Rate this question:

Next Question

Previous Question Tag Question

Feedback End Review

Difficulty: Average

Peer Responses %

https://mypastest.pastest.com/Secure/TestMe/TimedBrowser/1098274#Top 2/3
8/25/22, 9:30 PM MyPastest

0:00:00/03:00:00

A 74-year-old man presents to the Haematology Clinic with lumbar spine and right humerus pain. His
General Practitioner has also noted deteriorating renal function over the past five months. His blood pressure
is 132/82 mmHg and his heart rate is 90 bpm and regular. He looks pale.

Investigation Result Normal value

Haemoglobin (Hb) 92 g/l 115–155 g/l

White cell count (WCC) 6.1 × 10 9/l 4–11 × 10 9/l

Platelets (PLT) 155 × 10 9/l 150–400 × 10 9/l

Sodium (Na +) 142 mmol/l 135–145 mmol/l

Potassium (K +) 5.2 mmol/l 3.5–5.0 mmol/l

Creatinine (Cr) 205 µmol/l 50–120 µmol/l

Total protein 61 g/l 60–83 g/l

Albumin 28 g/l 35–45 g/l

What is the most likely diagnosis?

A Chronic myeloid leukaemia

B Multiple myeloma

C Myelofibrosis

D Non-Hodgkin’s lymphoma

E Prostate cancer

Explanation 

B Multiple myeloma

https://mypastest.pastest.com/Secure/TestMe/TimedBrowser/1098274#Top 1/3
8/25/22, 9:30 PM MyPastest

This patient is anaemic, with bone pain affecting both the lumbar spine and the right humerus. There is also a
significant elevation in immunoglobulins (calculated by taking albumin away from total protein): 61 - 28 = 33
g/l (vs a normal range of 7–21.5 g/l). Coupled with significant renal impairment, this raises the possibility of
multiple myeloma with light chain disease. Immunoglobulin electrophoresis to confirm the presence of a
paraprotein band is the next investigation of choice.

A Chronic myeloid leukaemia

Chronic myeloid leukaemia results in splenomegaly and usually has an elevated WCC, often above 25 × 10 9/l
making it an unlikely diagnosis in this patient.

C Myelofibrosis

Myelofibrosis is associated with anaemia and thrombocytopenia, as well as with splenomegaly. It does not fit
with the clinical picture seen in this patient.

D Non-Hodgkin’s lymphoma

Elevated immunoglobulins are seen in patients with non-Hodgkin’s lymphoma, although not associated with
significant bone disease, and renal failure is more rarely seen vs myeloma.

E Prostate cancer

Prostate cancer with metastases is a potential cause of bone pain and anaemia in elderly men. However, the
raised immunoglobulin fraction seen here is more consistent with myeloma.
73156

Rate this question:

Next Question

Previous Question Tag Question

Feedback End Review

Difficulty: Average

Peer Responses %

https://mypastest.pastest.com/Secure/TestMe/TimedBrowser/1098274#Top 2/3
8/25/22, 9:30 PM MyPastest

0:00:00/03:00:00

A 42-year-old bar owner is brought to the Emergency Department suffering from confusion, short-term
memory loss and diplopia. He has nystagmus and bilateral lateral rectus palsy. He also appears to be
unsteady on his feet with an ataxic gait.

Investigations:

Investigation Result Normal value

Haemoglobin (Hb) 92 g/l 115–155 g/l

White cell count (WCC) 7.1 × 10 9/l 4–11 × 10 9/l

Platelets (PLT) 75 × 10 9/l 150–400 × 10 9/l

Sodium (Na +) 134 mmol/l 135–145 mmol/l

Potassium (K +) 3.9 mmol/l 3.5–5.0 mmol/l

Creatinine (Cr) 85 µmol/l 50–120 µmol/l

Computed tomography (CT) head Unremarkable

What is the most likely diagnosis?

A Creutzfeldt–Jakob disease

B Hepatic encephalopathy

C Idiopathic intracranial hypertension

D Normal pressure hydrocephalus

E Wernicke’s encephalopathy

Explanation 

E Wernicke’s encephalopathy

https://mypastest.pastest.com/Secure/TestMe/TimedBrowser/1098274#Top 1/3
8/25/22, 9:30 PM MyPastest

The gaze palsy, coupled with short-term memory loss and an ataxic gait, is very suggestive of Wernicke’s
encephalopathy due to thiamine deficiency. In this case, the patient is a bar owner, which raises the possibility
of alcoholism as the likely cause. Long-term alcohol abuse is also consistent with the anaemia and low
platelets count seen here. Intravenous thiamine replacement is the key intervention of choice and should be
instigated early in anyone with suspected Wernicke’s encephalopathy.

A Creutzfeldt–Jakob disease

Sporadic Creutzfeldt–Jakob disease is predominantly a disease of the elderly and is associated with rapidly
progressive dementia. Rapidly progressive cerebral atrophy is seen on scanning.

B Hepatic encephalopathy

Hepatic encephalopathy is an alternative diagnosis in anyone presenting with features of liver disease,
drowsiness and confusion. However, the gaze palsy here is a strong pointer towards Wernicke’s
encephalopathy.

C Idiopathic intracranial hypertension

Idiopathic intracranial hypertension is predominantly seen in younger, obese women and is characterised by
headache and visual disturbance.

D Normal pressure hydrocephalus

Normal pressure hydrocephalus is seen in the elderly and is associated with memory loss, gait disturbance
and urinary incontinence.
73173

Rate this question:

Next Question

Previous Question Tag Question

Feedback End Review

Difficulty: Average

Peer Responses %

https://mypastest.pastest.com/Secure/TestMe/TimedBrowser/1098274#Top 2/3
8/25/22, 9:30 PM MyPastest

0:00:00/03:00:00

A 45-year-old man presents to the Gastroenterology Clinic for a routine review. He has a diagnosis of cyclical
vomiting syndrome and a number of attacks have been severe, requiring hospital admission for intravenous
fluids. Since his last hospital admission, he has been prescribed an antiemetic to control his symptoms.
However, he claims that since he has started taking this medication, he is suffering from diarrhoea. There is
no other past medical history of note and he has no known allergies.

What is the most likely medication to be causing this patient’s symptoms?

A Cyclizine

B Haloperidol

C Metoclopramide

D Ondansetron

E Prochlorperazine

Explanation 

C Metoclopramide

Metoclopramide is an antiemetic with prokinetic properties. By encouraging peristalsis of stomach contents, a


very common side-effect is diarrhoea. It is likely that this patient has been prescribed metoclopramide.

A Cyclizine

Cyclizine is a H1-receptor antagonist. Side-effects of cyclizine include urinary retention, dry mouth, visual
blurring and constipation or diarrhoea. However, diarrhoea is not as commonly seen with cyclizine use
compared to a prokinetic antiemetic such as metoclopramide.

B Haloperidol

Haloperidol is a D2-receptor blocker. Common side-effects include constipation, dry mouth and dizziness.
However, diarrhoea is not a symptom of haloperidol use.

https://mypastest.pastest.com/Secure/TestMe/TimedBrowser/1098274#Top 1/3
8/25/22, 9:30 PM MyPastest

D Ondansetron

Ondansetron is a 5-HT3 receptor antagonist. A common side-effect of ondansetron is constipation. QTc


interval prolongation and arrhythmias may also be seen. However, ondansetron does not cause diarrhoea.

E Prochlorperazine

Prochlorperazine is a D2-receptor blocker. Common side-effects include constipation, dry mouth and
dizziness. However, diarrhoea is not a symptom of prochlorperazine use.
73210

Rate this question:

Next Question

Previous Question Tag Question

Feedback End Review

Difficulty: Average

Peer Responses %

Q. Answered Flagged

Q1

Q2

Q3

Q4

Q5

https://mypastest.pastest.com/Secure/TestMe/TimedBrowser/1098274#Top 2/3
8/25/22, 9:30 PM MyPastest

0:00:00/03:00:00

A 45-year-old woman presents to the Emergency Department with sudden painless visual loss affecting her
left eye. She has hypertension and diet-controlled type II diabetes.

Ophthalmoscopy:


What is the most likely cause of this patient’s symptoms?

A Acute closed-angle glaucoma

B Central retinal artery occlusion

C Central retinal vein occlusion

D Open-angle glaucoma

E Optic neuritis

Explanation 

C Central retinal vein occlusion

This presentation is consistent with central retinal vein occlusion where there is rapid painless visual loss and
the development of multiple flame-shaped haemorrhages. Diabetes is a potential risk factor for central retinal
vein occlusion. Over the longer term, anticoagulation may be required to prevent further events. Macular

https://mypastest.pastest.com/Secure/TestMe/TimedBrowser/1098274#Top 1/3
8/25/22, 9:30 PM MyPastest

oedema post-retinal vein occlusion is managed with anti-vascular endothelial growth factor injections and
laser photocoagulation.

Image source: By Sue Ford- Science Photo Library

A Acute closed-angle glaucoma

Acute closed-angle glaucoma leads to visual loss and severe eye pain. It does not fit with the painless loss of
vision and flame-shaped haemorrhages seen in this patient.

B Central retinal artery occlusion

Central retinal artery occlusion leads to retinal pallor and a cherry red spot. It does not fit with the multiple
haemorrhages displayed by this patient.

D Open-angle glaucoma

Open-angle glaucoma leads to slow-progressing visual loss, particularly affecting night vision and the
development of a central scotoma.

E Optic neuritis

Optic neuritis leads to swelling of the optic nerve and often pain in the affected eye. It does not lead to the
multiple flame-shaped haemorrhages seen here.
73171

Rate this question:

Next Question

Previous Question Tag Question

Feedback End Review

Difficulty: Average

Peer Responses %

https://mypastest.pastest.com/Secure/TestMe/TimedBrowser/1098274#Top 2/3
8/25/22, 9:30 PM MyPastest

0:00:00/03:00:00

A 26-year-old heroin addict is brought unconscious to the Emergency Department. He has pinpoint pupils
and his blood pressure is 90/60 mmHg. His respiratory rate is six breaths per minute and his oxygen
saturation is 90% on 10 litres of oxygen.

What is the most appropriate next step?

A Adrenaline

B Doxapram

C Flumazenil

D Naloxone

E Salbutamol

Explanation 

D Naloxone

Naloxone is a competitive opioid receptor inhibitor. It can be used in the treatment of opiate overdose,
although it is important to note that it has a relatively short half-life of 60–90 minutes. This means that
recurrent doses are likely to be required, necessitating either frequent observation for redosing or the use of a
naloxone infusion. An initial dose of 400–2000 μg can be given intravenously; it can also be given
intramuscularly and subcutaneously where access is a problem.

A Adrenaline

There is no indication in this situation for use of adrenaline. Blood pressure is relatively well maintained and
with the use of an opiate reversal agent, such as naloxone, there should be an improvement in consciousness
and respiratory rate.

B Doxapram

https://mypastest.pastest.com/Secure/TestMe/TimedBrowser/1098274#Top 1/3
8/25/22, 9:30 PM MyPastest

Doxapram is a central respiratory stimulant that was primarily used in the treatment of chronic obstructive
pulmonary disease exacerbation before the advent of therapies such as non-invasive positive pressure
ventilation.

C Flumazenil

Flumazenil is the reversal agent for benzodiazepine overdose. Its use is not recommended for an overdose of
benzodiazepines in chronic users, because rapid reversal can lead to the development of seizures.

E Salbutamol

Salbutamol has no value in the treatment of opiate overdose. It is not a respiratory stimulant and merely acts
as a bronchodilator.
73123

Rate this question:

Next Question

Previous Question Tag Question

Feedback End Review

Difficulty: Average

Peer Responses %

Q. Answered Flagged

Q1

Q2

Q3

https://mypastest.pastest.com/Secure/TestMe/TimedBrowser/1098274#Top 2/3
8/25/22, 9:30 PM MyPastest

0:00:00/03:00:00

A 21-year-old man from a travelling family is referred to the Cardiology Clinic, complaining of headaches,
nosebleeds and hypertension. He says he always feels cold, particularly in his feet. His blood pressure is
155/95 mmHg in the right arm, and 122/75 mmHg in the left. Pulses in the feet appear diminished.

The right-hand side of his chest X-ray is shown below:


What is the most likely cause of this patient’s symptoms?

A Aortic stenosis

B Atrial septal defect (ASD)

C Coarctation of the aorta

https://mypastest.pastest.com/Secure/TestMe/TimedBrowser/1098274#Top 1/4
8/25/22, 9:30 PM MyPastest

D Hypertrophic obstructive cardiomyopathy

E Patent ductus arteriosus

Explanation 

C Coarctation of the aorta

Travelling families may fail to engage with medical services, accounting for this patient’s relatively late
presentation. The headache and nosebleeds, coupled with differential blood pressures between the arms,
raise the possibility of coarctation of the aorta. The chest X-ray shows inferior rib notching, which occurs due
to enlargement of intercostal vessels which act as collaterals to bypass the coarctation. The pre-stenotic
aorta feeds the anterior intercostal branches, which anastomose with posterior intercostal branches
originating from the post-stenotic aorta.

Blood flow in the posterior intercostal arteries is reversed and hence feeds the post-stenotic aorta. Magnetic
resonance imaging is indicated to delineate the anatomy of the coarctation and determine the most
appropriate surgical approach to intervention. Angioplasty and stenting may be possible in some cases.

Image credit: Frank Gaillard, CC BY-SA 3.0

A Aortic stenosis

Aortic stenosis has an ejection systolic murmur and would not be expected to cause the difference in blood
pressures between the arms here, which suggests any lesion is between the right brachiocephalic trunk and
the origin of the left subclavian artery.

B Atrial septal defect (ASD)

Atrial septal defects (ASDs) may be asymptomatic or present with palpitations due to periods of atrial
fibrillation. An ASD would not fit with the differential blood pressures seen in this patient.

D Hypertrophic obstructive cardiomyopathy

Hypertrophic obstructive cardiomyopathy may present with hypertension, palpitations and signs and
symptoms of cardiac insufficiency. However, it would not account for the differential blood pressures seen in
this patient.

E Patent ductus arteriosus

Patent ductus arteriosus more usually presents in childhood with failure to thrive, and a machinery-like
murmur due to continuous flow through the ductus arteriosus is noted on physical examination.
73113

https://mypastest.pastest.com/Secure/TestMe/TimedBrowser/1098274#Top 2/4
8/25/22, 9:31 PM MyPastest

0:00:00/03:00:00

A 32-year-old woman who has moved to the United Kingdom from Pakistan presents to the Emergency
Department with irregular palpitations. She is 24 weeks pregnant. On further questioning, it transpires she
has been short of breath on exercise for some months before getting pregnant and has had occasional
palpitations. She also reports being ill as a child, taking antibiotics and being confined to bed for some time.
Investigations:

Investigation Result Normal value

Haemoglobin (Hb) 112 g/l 115–155 g/l

White cell count (WCC) 6.9 × 10 9/l 4–11 × 10 9/l

Platelets (PLT) 189 × 10 9/l 150–400 × 10 9/l

Sodium (Na +) 143 mmol/l 135–145 mmol/l

Potassium (K +) 3.9 mmol/l 3.5–5.0 mmol/l

Creatinine (Cr) 75 µmol/l 50–120 µmol/l

Electrocardiogram (ECG) Atrial fibrillation ventricular rate 110 bpm

What is the most likely cause of this patient’s atrial fibrillation?

A Atrial septal defect

B Hypertrophic obstructive cardiomyopathy

C Mitral stenosis

D Peripartum thyroiditis

E Wolff–Parkinson–White syndrome

Explanation 

C Mitral stenosis

https://mypastest.pastest.com/Secure/TestMe/TimedBrowser/1098274#Top 1/3
8/25/22, 9:31 PM MyPastest

The childhood illness is suspicious for rheumatic fever. Although rheumatic fever is now very rarely seen in
Europe and the United States, it is responsible for a high percentage of pregnancy-related cardiovascular
diseases in developing countries. Mitral stenosis is therefore the most likely cause of this patient’s
presentation with atrial fibrillation and an uncontrolled ventricular rate. Echocardiography is the most
appropriate next step, with valvotomy an option for severe mitral stenosis, accompanied by medication to
slow the ventricular rate. Although beta-blockers are associated with some growth retardation in the fetus,
the benefit-risk is generally considered positive.

A Atrial septal defect

Atrial septal defects are a common cause of atrial fibrillation in pregnancy, although possible rheumatic fever
in childhood is a stronger pointer towards rheumatic heart disease as the underlying cause of the atrial
fibrillation.

B Hypertrophic obstructive cardiomyopathy

Hypertrophic obstructive cardiomyopathy is associated with left ventricular hypertrophy on the ECG and
symptoms of left ventricular outflow tract obstruction, including syncope and angina.

D Peripartum thyroiditis

Postpartum thyroiditis is associated with initial thyrotoxicosis, followed by a period of hypothyroidism. There
are no other features of thyroid disease described here such as changes in body weight.

E Wolff–Parkinson–White syndrome

Wolff–Parkinson–White syndrome is associated with ECG abnormalities, including a short PR interval and
delta wave, not described here. The illness in childhood also raises the possibility of mitral stenosis related to
rheumatic fever.
73117

Rate this question:

Next Question

Previous Question Tag Question

Feedback End Review

Difficulty: Average

Peer Responses %

https://mypastest.pastest.com/Secure/TestMe/TimedBrowser/1098274#Top 2/3
8/25/22, 9:31 PM MyPastest

0:00:00/03:00:00

A 31-year-old man presents to his General Practitioner with complaints of an erythematous scaling skin rash
affecting his scalp and the extensor surfaces of his arms and legs. It is noted that he has nail pitting.

Investigations:


What is the most likely diagnosis?

A Eczema

B Pityriasis versicolor

C Psoriasis

D Seborrhoeic dermatitis

E Tinea corporis

Explanation 

C Psoriasis

https://mypastest.pastest.com/Secure/TestMe/TimedBrowser/1098274#Top 1/3
8/25/22, 9:31 PM MyPastest

The erythematous scaling skin rash affecting the extensor surfaces and the scalp, accompanied by nail pitting,
is consistent with psoriasis. Psoriasis is associated with features of insulin resistance, including obesity and
type II diabetes, alcohol consumption and drugs such as lithium. Streptococcal infection and human
immunodeficiency virus/acquired immune deficiency syndrome are also known to worsen symptoms. First-line
topical therapies for psoriasis include corticosteroids, vitamin D analogues, dithranol and coal tar
preparations.

Image credit: By James Heilman, MD - CC BY-SA 3.0.

A Eczema

Eczema is most commonly seen on flexural skin surfaces and is characterised by erythema, usually without
the marked superficial skin scaling seen in this patient.

B Pityriasis versicolor

Pityriasis versicolor is characterised by multiple round erythematous patches with only minor skin scaling,
unlike the features this patient presented with. It is caused by Malassezia, a fungus found on normal skin.

D Seborrhoeic dermatitis

Seborrhoeic dermatitis is associated with a fine, erythematous scaling skin rash, which particularly affects the
face and scalp, and dandruff. It does not fit with this patient's presentations.

E Tinea corporis

Tinea corporis is characterised by an annular area of erythema and skin scaling with a central healed area. It
does not fit with the presentations seen in this patient.
73132

Rate this question:

Next Question

Previous Question Tag Question

Feedback End Review

Difficulty: Average

Peer Responses %

https://mypastest.pastest.com/Secure/TestMe/TimedBrowser/1098274#Top 2/3
8/25/22, 9:31 PM MyPastest

0:00:00/03:00:00

A 48-year-old woman who has rheumatoid arthritis and problems with a dry mouth and dry eyes presents to
her General Practitioner with weakness and lethargy which have steadily worsened over the past six months.
Her blood pressure is 145/90 mmHg and her heart rate is 78 bpm and regular. Her body mass index is 24
kg/m 2.

Investigation Result Normal value

Haemoglobin (Hb) 101 g/l 115–155 g/l

White cell count (WCC) 7.2 × 10 9/l 4–11 × 10 9/l

Platelets (PLT) 192 × 10 9/l 150–400 × 10 9/l

Sodium (Na +) 141 mmol/l 135–145 mmol/l

Potassium (K +) 3.0 mmol/l 3.5–5.0 mmol/l

Creatinine (Cr) 125 µmol/l 50–120 µmol/l

Bicarbonate (HCO 3 −) 17 mmol/l 23–29 mmol/l

Urine analysis Blood negative, protein 1+

What is the most likely diagnosis?

A Liddle syndrome

B Nephrogenic diabetes insipidus

C Renal tubular acidosis type 1

D Renal tubular acidosis type 2

E Renal tubular acidosis type 4

Explanation 

C Renal tubular acidosis type 1

https://mypastest.pastest.com/Secure/TestMe/TimedBrowser/1098274#Top 1/3
8/25/22, 9:31 PM MyPastest

This patient has a medical history consistent with Sjögren syndrome which is associated with distal renal
tubular acidosis. Distal renal tubular acidosis occurs because of an inability to produce acid urine in the distal
tubule. The biochemistry showing hypokalaemia and metabolic acidosis further support the diagnosis.
Urinary pH is above six and urinary calcium excretion is increased in patients with the condition.
Hypokalaemia is corrected with potassium supplementation as the first step in therapy. Over the long term,
bicarbonate supplementation is usually sufficient.

A Liddle syndrome

Liddle syndrome is an inherited syndrome which results in marked hypertension and hypokalaemia with
metabolic acidosis.

B Nephrogenic diabetes insipidus

Nephrogenic diabetes insipidus is associated with polyuria and polydipsia, not with hypokalaemia and
acidosis seen in this patient.

D Renal tubular acidosis type 2

Renal tubular acidosis type 2 is also known as proximal renal tubular acidosis. Causes include cystinosis,
amyloidosis and multiple myeloma.

E Renal tubular acidosis type 4

Renal tubular acidosis type 4 is associated with resistance to aldosterone action where hyperkalaemia and
acidosis occur.
73194

Rate this question:

Next Question

Previous Question Tag Question

Feedback End Review

Difficulty: Average

Peer Responses %

https://mypastest.pastest.com/Secure/TestMe/TimedBrowser/1098274#Top 2/3
8/25/22, 9:31 PM MyPastest

0:00:00/03:00:00

A 73-year-old man presents to his General Practitioner for review. He has chronic stable angina and New
York Heart Association (NYHA) class II heart failure, and currently uses a glyceryl trinitrate (GTN) inhaler
intermittently. He has asthma, for which he takes a salmeterol and fluticasone combination inhaler. His blood
pressure is 110/72 mmHg and his resting heart rate is 85 bpm and regular. His chest is clear and heart
sounds are normal.

What is the most appropriate intervention for this patient’s angina?

A Amlodipine

B Bisoprolol

C Isosorbide mononitrate

D Ivabradine

E Verapamil

Explanation 

D Ivabradine

This patient’s resting heart rate is above 75 bpm, which is associated with worsening symptoms of both
angina and heart failure. In this population, ivabradine is indicated if a beta-blocker is contraindicated (as
here) or fails to control heart rate adequately. The starting dose is 2.5 mg twice daily and it can be uptitrated
to 7.5 mg twice daily. Ivabradine acts to selectively inhibit I(f) channels within the cardiac pacemaker.
Although ivabradine has not been shown to have an impact on outcomes in patients with angina alone, it has
shown outcome benefits in patients, as with this patient, who also have heart failure.

A Amlodipine

Dihydropyridine calcium channel antagonists are less effective in treating angina because they do not reduce
heart rate. Indeed because of vasodilatation, resting heart rate may actually increase.

B Bisoprolol

https://mypastest.pastest.com/Secure/TestMe/TimedBrowser/1098274#Top 1/3
8/25/22, 9:31 PM MyPastest

Although bisoprolol is effective in managing symptoms of angina where there is elevated resting heart rate, it
is contraindicated because of this patient’s asthma.

C Isosorbide mononitrate

Isosorbide mononitrate should potentially be avoided here. It may drive symptoms of hypotension, and
tolerance often develops to nitrates after a few weeks of therapy.

E Verapamil

Although non-dihydropyridine calcium channel antagonists do improve symptoms of angina, they are not
associated with improved cardiovascular outcomes. This is in contrast to ivabradine which has shown an
outcome benefit in patients with baseline heart failure.
73116

Rate this question:

Next Question

Previous Question Tag Question

Feedback End Review

Difficulty: Average

Peer Responses %

Q. Answered Flagged

Q1

Q2

Q3

https://mypastest.pastest.com/Secure/TestMe/TimedBrowser/1098274#Top 2/3
8/25/22, 9:31 PM MyPastest

0:00:00/03:00:00

A 59-year-old woman with chronic renal impairment due to type II diabetes presents to her General
Practitioner for review. She has felt tired, with bony aches, over the past few months. Her blood pressure is
122/72 mmHg and her heart rate is 82 bpm and regular.

Investigation Result Normal value

Haemoglobin (Hb) 103 g/l 115–155 g/l

White cell count (WCC) 6.4 × 10 9/l 4–11 × 10 9/l

Platelets (PLT) 205 × 10 9/l 150–400 × 10 9/l

Sodium (Na +) 141 mmol/l 135–145 mmol/l

Potassium (K +) 4.9 mmol/l 3.5–5.0 mmol/l

Creatinine (Cr) 293 µmol/l 50–120 µmol/l

Calcium (Ca 2+) 2.11 mmol/l 10–300 mmol/l

Phosphate 1.7 mmol/l 0.8–1.5 mmol/l

What is the most likely cause of this patient’s presentation?

A Hypoparathyroidism

B Osteomalacia

C Paget’s disease of bone

D Primary hyperparathyroidism

E Secondary hyperparathyroidism

Explanation 

E Secondary hyperparathyroidism

https://mypastest.pastest.com/Secure/TestMe/TimedBrowser/1098274#Top 1/4
8/25/22, 9:31 PM MyPastest

This patient has renal impairment with a borderline low Calcium (Ca 2+) level and a significant elevation in
phosphate level. Although the parathyroid hormone level is not given, it is likely to be significantly elevated,
with secondary hyperparathyroidism as the underlying diagnosis. Treatment includes calcium
supplementation and correction of vitamin D deficiency, as well as restriction of dietary phosphate.
Calcimimetics, such as cinacalcet, are reserved for those with hyperparathyroidism which is refractory to other
treatments and in whom surgery is not an option.

A Hypoparathyroidism

Hypoparathyroidism leads to hypocalcaemia, rather than the low normal Ca 2+ levels seen here. It is not
associated with renal impairment.

B Osteomalacia

Osteomalacia is associated with hypocalcaemia, but not with the elevated phosphate level seen here, which
suggests secondary hyperparathyroidism is the underlying diagnosis.

C Paget’s disease of bone

Paget’s disease of bone leads to elevated alkaline phosphatase levels in the presence of a normal Ca 2+ level.
Involvement of a single joint, such as the hip, is common.

D Primary hyperparathyroidism

Primary hyperparathyroidism leads to hypercalcaemia in the presence of normal renal function.


Parathyroidectomy is the intervention of choice where the Ca 2+ level is significantly elevated.
73191

Rate this question:

Next Question

Previous Question Tag Question

Feedback End Review

Difficulty: Average

Peer Responses %

https://mypastest.pastest.com/Secure/TestMe/TimedBrowser/1098274#Top 2/4
8/25/22, 9:31 PM MyPastest

0:00:00/03:00:00

A 49-year-old schoolteacher presents with sensory loss affecting both feet and frequent tripping. He has a
loss of fine touch, proprioception and vibration sense affecting both feet. He has bilateral loss of ankle jerks
and there is a weakness of ankle dorsiflexion. Electromyography (EMG) shows positive sharp waves and
fibrillation potentials.
Investigations:

Investigation Result Normal value

Haemoglobin (Hb) 121 g/l 115–155 g/l

White cell count (WCC) 6.9 × 10 9/l 4–11 × 10 9/l

Platelets (PLT) 95 × 10 9/l 150–400 × 10 9/l

Sodium (Na +) 138 mmol/l 135–145 mmol/l

Potassium (K +) 3.9 mmol/l 3.5–5.0 mmol/l

Creatinine (Cr) 90 µmol/l 50–120 µmol/l

Glucose 7.1 mmol/l < 11 mmol/l

What is the most likely cause of this patient’s neuropathy?

A Alcoholic neuropathy

B Diabetic neuropathy

C Hereditary motor sensory neuropathy

D Lead neuropathy

E Thyroid neuropathy

Explanation 

A Alcoholic neuropathy

https://mypastest.pastest.com/Secure/TestMe/TimedBrowser/1098274#Top 1/3
8/25/22, 9:31 PM MyPastest

The anaemia and thrombocytopenia reported in this patient are consistent with chronic liver disease due to
alcoholism, as are the peripheral sensory loss and foot drop. Electromyography (EMG) in patients with
alcoholic neuropathy characteristically shows both positive sharp waves and fibrillation potentials. Alcoholic
cardiomyopathy may well improve with cessation of further alcohol consumption. It is therefore very
important to gain an accurate history with respect to this patient’s alcohol use.

B Diabetic neuropathy

This patient’s glucose level is in the normal range for a random sample, and diabetic neuropathy leads to
peripheral sensory loss, rather than the mixed sensory and motor deficiency seen in this patient.

C Hereditary motor sensory neuropathy

Hereditary motor and sensory neuropathy present with gradually progressive peripheral motor and sensory
loss, beginning in the intrinsic muscles of the feet. It would not fit with the anaemia and thrombocytopenia
seen here.

D Lead neuropathy

Lead poisoning characteristically leads to distal motor neuropathy. It also causes other significant symptoms,
including chronic abdominal pain and constipation, which are absent here.

E Thyroid neuropathy

Thyroid neuropathy is associated with sensory and motor axonal neuropathy. The clinical picture here,
including anaemia and low PLT count, is more suggestive of alcoholism and subsequent nerve dysfunction.
73176

Rate this question:

Next Question

Previous Question Tag Question

Feedback End Review

Difficulty: Average

Peer Responses %

https://mypastest.pastest.com/Secure/TestMe/TimedBrowser/1098274#Top 2/3
8/25/22, 9:32 PM MyPastest

0:00:00/03:00:00

A 74-year-old man is admitted to the Emergency Department with worsening confusion and a cough of five
days duration. His blood pressure is 135/82 mmHg and his heart rate is 85 bpm and regular. He has crackles
in the right lung base on auscultation. There is no ankle swelling.

Investigations:

Investigation Result Normal value

Haemoglobin (Hb) 136 g/l 115–155 g/l

White cell count (WCC) 14.1 × 10 9/l 4–11 × 10 9/l

Platelets (PLT) 199 × 10 9/l 150–400 × 10 9/l

Sodium (Na +) 128 mmol/l 135–145 mmol/l

Potassium (K +) 3.8 mmol/l 3.5–5.0 mmol/l

Creatinine (Cr) 99 µmol/l 50–120 µmol/l

What is the most appropriate next step?

A Demeclocycline

B Fluid restriction

C 0.45% saline

D 1.8% saline

E Tolvaptan

Explanation 

B Fluid restriction

This patient likely has a syndrome of inappropriate antidiuretic hormone secretion (SIADH) as a result of
pneumonia. His sodium (Na +) level is reduced; he appears euvolaemic, with normal-range blood pressure, and
his Cr level is normal. Fluid restriction is therefore the most appropriate initial intervention. In patients who

https://mypastest.pastest.com/Secure/TestMe/TimedBrowser/1098274#Top 1/3
8/25/22, 9:32 PM MyPastest

fail to increase their Na + level with fluid restriction, then demeclocycline or tolvaptan may be an option.
Hypertonic saline is only considered when there are life-threatening neurological sequelae of hyponatraemia.

A Demeclocycline

Demeclocycline leads to nephrogenic diabetes insipidus and increases free water excretion. It is considered in
patients whose hyponatraemia fails to respond to fluid restriction alone.

C 0.45% saline

This is hypotonic saline, which is not of value in increasing the serum Na+ level in patients with significant
hyponatraemia.

D 1.8% saline

If possible, 1.8% saline should be avoided and only considered if there is life-threatening hyponatraemia, eg
with significantly decreased consciousness or seizures. It should be given on the Intensive Therapy Unit,
aiming for a rise in Na + level of < 10 mmol/l over a 24-hour period.

E Tolvaptan

Tolvaptan is a vasopressin V2 receptor antagonist. It can be considered in patients with SIADH who are
resistant to fluid retention and/or demeclocycline.
73154

Rate this question:

Next Question

Previous Question Tag Question

Feedback End Review

Difficulty: Average

Peer Responses %

https://mypastest.pastest.com/Secure/TestMe/TimedBrowser/1098274#Top 2/3
8/25/22, 9:32 PM MyPastest

0:00:00/03:00:00

A 19-year-old man presents with fevers which peak each evening and are sometimes as high as 39 °C and
joint pains affecting the knees, wrists, ankles and hips, as well as an erythematous rash which affects his
chest and thighs at the time he has a fever. He also reports intermittent pleuritic chest pain.

Investigation Result Normal value

Haemoglobin (Hb) 129 g/l 115–155 g/l

White cell count (WCC) 13.1 × 10 9/l 4.0–11.0 × 10 9/l

Platelets (PLT) 410 × 10 9/l 150–400 × 10 9/l

Sodium (Na +) 143 mmol/l 135–145 mmol/l

Potassium (K +) 3.7 mmol/l 3.5–5.0 mmol/l

Creatinine (Cr) 95 µmol/l 50–120 µmol/l

Rheumatoid factor Negative

Erythrocyte sedimentation rate (ESR) 65 mm/hour < 10 mm/hour

What is the most likely diagnosis?

A Adult-onset Still’s disease

B Ankylosing spondylitis

C Polymyalgia rheumatica

D Reactive arthritis

E Rheumatoid arthritis

Explanation 

A Adult-onset Still’s disease

https://mypastest.pastest.com/Secure/TestMe/TimedBrowser/1098274#Top 1/4
8/25/22, 9:32 PM MyPastest

This patient has a number of features of Still’s disease, including periodic fevers, a typical pattern of joint
disease seen in Still’s disease, an erythematous rash, commonly described as being salmon pink in natural
appearance and a marked rise in erythrocyte sedimentation rate (ESR). There are no specific autoantibodies
which are positive in patients with Still’s disease and the negative rheumatoid factor here is a further pointer
towards the underlying diagnosis.

Major criteria for diagnosing Still’s disease include:

fever of at least 39 °C lasting at least one week


arthralgia lasting two weeks or longer
a non-itchy macular or maculopapular skin rash that is salmon-coloured in appearance and found over
the trunk or extremities during febrile episodes
leukocytosis (10 × 10 9 /l), with at least 80% neutrophils.

Corticosteroids are the initial intervention of choice for treating Still’s disease. Methotrexate and anti-tumour
necrosis factor (TNF) agents are also used.

B Ankylosing spondylitis

Ankylosing spondylitis can present in this age group but is more likely to be associated with lumbar and
sacroiliac joint pain and stiffness. It does not fit with the pattern of symptoms seen in this patient.

C Polymyalgia rheumatica

Polymyalgia rheumatica is associated with proximal muscle pain and stiffness and is commonly seen in the
elderly population.

D Reactive arthritis

Reactive arthritis is most commonly seen following an episode of sexually transmitted or gastrointestinal
infection and does not follow the symptom pattern seen in this patient.

E Rheumatoid arthritis

The intermittent fevers and rash, coupled with the pattern of joint involvement seen here, are more consistent
with a diagnosis of Still’s disease, especially in the presence of a negative rheumatoid factor.
73207

Rate this question:

Next Question

https://mypastest.pastest.com/Secure/TestMe/TimedBrowser/1098274#Top 2/4
8/25/22, 9:32 PM MyPastest

0:00:00/03:00:00

A 67-year-old man presents with a dry cough, night sweats and fatigue over the past six weeks. He has
chronic obstructive pulmonary disease (COPD) that is managed with triple inhaled therapy, and he is usually
admitted once per year to the hospital. His temperature is 37.7 °C. His blood pressure is 155/82 mmHg, and
his heart rate is 75 bpm and regular. There are coarse crackles over the right upper lobe on auscultation.

Investigation Result Normal value

Haemoglobin (Hb) 132 g/l 115–155 g/l

White cell count (WCC) 8.1 × 10 9/l 4–11 × 10 9/l

Platelets (PLT) 187 × 10 9/l 150–400 × 10 9/l

Sodium (Na +) 143 mmol/l 135–145 mmol/l

Potassium (K +) 4.3 mmol/l 3.5–5.0 mmol/l

Creatinine (Cr) 95 µmol/l 50–120 µmol/l

Sputum Acid- and alcohol-fast bacilli positive

Polymerase chain reaction (PCR) Negative for Mycobacterium tuberculosis

Chest X-ray Right upper lobe infiltration and cavitation

What is the most likely cause of this patient’s symptoms?

A Aspergillus fumigatus

B Mycobacterium avium

C Mycobacterium chimaera

D Mycobacterium leprae

E Mycobacterium scrofulaceum

Explanation 

B Mycobacterium avium

https://mypastest.pastest.com/Secure/TestMe/TimedBrowser/1098274#Top 1/3
8/25/22, 9:32 PM MyPastest

Mycobacterium avium is a cause of pulmonary mycobacterial infection, particularly in patients who have
underlying chronic lung disease, as in the case here. Upper lobe infiltration with cavitation is typical, and the
presence of acid- and alcohol-fast bacilli in the absence of M. tuberculosis on PCR testing fit with the
diagnosis. For those with macrolide-sensitive disease, a macrolide is usually combined with ethambutol and
rifampicin for several months. For macrolide-resistant disease, a regimen including a parenteral
aminoglycoside is required.

A Aspergillus fumigatus

On a chest X-ray, aspergillus fumigatus normally appears as a spherical mass within a previous cavity from
tuberculosis or bronchiectasis, which does not fit with this patient's results.

C Mycobacterium chimaera

Mycobacterium chimaera has low virulence and has primarily been associated with infection after prosthetic
cardiac valve insertion, making it unlikely in this patient.

D Mycobacterium leprae

Mycobacterium leprae is the cause of leprosy, rather than the cause of lung disease associated with
mycobacterial infection.

E Mycobacterium scrofulaceum

Mycobacterium scrofulaceum primarily causes superficial lymphadenitis, especially in children and patients
who are immunocompromised.
73162

Rate this question:

Next Question

Previous Question Tag Question

Feedback End Review

Difficulty: Average

Peer Responses %

https://mypastest.pastest.com/Secure/TestMe/TimedBrowser/1098274#Top 2/3
8/25/22, 9:33 PM MyPastest

0:00:00/03:00:00

A 22-year-old woman is reviewed on the Medical Admissions Ward by the on-call team 24 hours after an
overdose with an unknown quantity of paracetamol. She has continued to feel nauseated on the ward and is
complaining of abdominal pain.

Investigations:

Investigation Result Normal value

Haemoglobin (Hb) 102 g/l 115–155 g/l

White cell count (WCC) 7.1 × 10 9/l 4–11 × 10 9/l

Platelets (PLT) 73 × 10 9/l 150–400 × 10 9/l

Sodium (Na +) 142 mmol/l 135–145 mmol/l

Potassium (K +) 5.0 mmol/l 3.5–5.0 mmol/l

Creatinine (Cr) 132 µmol/l 50–120 µmol/l

Bicarbonate 18 mmol/l 23–29 mmol/l

Alanine aminotransferase (ALT) 2732 U/l 0–35 U/l

International normalised ratio (INR) 1.5 < 1.1

What is the most appropriate intervention?

A Acetylcysteine

B Fresh frozen plasma

C Prothrombin complex concentrate

D Sodium bicarbonate

E Vitamin K

Explanation 

A Acetylcysteine
https://mypastest.pastest.com/Secure/TestMe/TimedBrowser/1098274#Top 1/4
8/25/22, 9:33 PM MyPastest

Guidelines from the University of Birmingham suggest there is value in continuing acetylcysteine beyond the
24-hour point post-overdose where the international normalised ratio (INR) is elevated above 1.3, as here.
The marked elevation in alanine aminotransferase (ALT) level is also a pointer to significant liver damage. In
this situation, although evidence is limited, acetylcysteine is thought to have antioxidant properties and
reduce further liver injury.

A poor prognosis in paracetamol overdose is indicated by:

an INR of > 3.0


plasma creatinine (Cr) > 200 μmol/l
blood pH < 7.3
signs of encephalopathy (mental confusion, drowsiness, spatial disorientation, asterixis).

B Fresh frozen plasma

Fresh frozen plasma is used to reverse abnormal clotting in patients who are actively bleeding. In this case, it
is not indicated, given the INR is only slightly elevated at 1.5.

C Prothrombin complex concentrate

Prothrombin complex concentrate contains clotting factors VII, IX and X. It is used to reverse clotting
abnormalities associated with warfarin toxicity.

D Sodium bicarbonate

Sodium bicarbonate is not indicated, given there is relatively modest metabolic acidosis where the
bicarbonate level is only reduced to 18 mmol/l.

E Vitamin K

The INR is modestly prolonged and the concern here is not vitamin K deficiency, but deficient liver synthetic
function, including with respect to the production of liver-dependent clotting factors.
73126

Rate this question:

Next Question

Previous Question Tag Question

Feedback End Review

https://mypastest.pastest.com/Secure/TestMe/TimedBrowser/1098274#Top 2/4
8/25/22, 9:33 PM MyPastest

0:00:00/03:00:00

A 24-year-old woman presents to the Emergency Department because of left loin pain and swinging fevers.
She started amoxicillin three days earlier for symptoms of a urinary tract infection. Her temperature is 37.9 °C,
blood pressure 105/70 mmHg and heart rate 98 bpm and regular. There is left loin tenderness on abdominal
palpation.

Investigation Result Normal value

Haemoglobin (Hb) 132 g/l 115–155 g/l

White cell count (WCC) 14.8 × 10 9/l 4–11 × 10 9/l

Platelets (PLT) 209 × 10 9/l 150–400 × 10 9/l

Sodium (Na +) 1443 mmol/l 135–145 mmol/l

Potassium (K +) 3.9 mmol/l 3.5–5.0 mmol/l

Creatinine (Cr) 89 µmol/l 50–120 µmol/l

What is the most likely diagnosis?

A Diverticulitis

B Ectopic pregnancy

C Glandular fever

D Nephrolithiasis

E Pyelonephritis

Explanation 

E Pyelonephritis

There is significant Escherichia coli resistance to amoxicillin. Therefore, it is not surprising that this patient
presented only three days later with a probable ascending infection now affecting the left kidney. The pain
and swinging fevers are consistent with pyelonephritis. Cephalosporins, co-amoxiclav and trimethoprim are

https://mypastest.pastest.com/Secure/TestMe/TimedBrowser/1098274#Top 1/3
8/25/22, 9:33 PM MyPastest

all potential options for treatment but should be backed up by sensitivity testing. Intravenous gentamicin may
also be considered.

A Diverticulitis

Diverticulitis is associated with left-sided anterior abdominal pain consistent with colonic inflammation. The
patient is also in the wrong age group for diverticulitis.

B Ectopic pregnancy

Ectopic pregnancy is associated with lower quadrant pain. The position of the pain and fever in the context of
a recent urinary tract infection is more consistent with pyelonephritis.

C Glandular fever

Glandular fever is associated with symptoms of pharyngitis and left upper quadrant pain due to splenic
tenderness, rather than left loin pain.

D Nephrolithiasis

Nephrolithiasis is a possible cause of loin pain. However, the pain usually radiates to the groin when
associated with the passage of any stones along the ureter.
73188

Rate this question:

Next Question

Previous Question Tag Question

Feedback End Review

Difficulty: Average

Peer Responses %

https://mypastest.pastest.com/Secure/TestMe/TimedBrowser/1098274#Top 2/3
8/25/22, 9:34 PM MyPastest

0:00:00/03:00:00

A 27-year-old man is admitted to the Emergency Department a few days after returning from a hiking trip to
the North West United States. He has a severe headache, with muscle pains and fever, as well as a
maculopapular rash that has appeared on the wrists, forearms, ankles, palms and soles. His temperature is
39.5 °C. His blood pressure is 110/70 mmHg, and his heart rate is 95 bpm and regular. You note a small
necrotic insect bite on his left ankle.

What is the most likely diagnosis?

A Endemic tick typhus

B Rocky Mountain spotted fever

C Sennetsu fever

D Scrub typhus

E Tickborne encephalitis

Explanation 

B Rocky Mountain spotted fever

Rocky Mountain spotted fever can be considered a possible diagnosis in patients exposed to a tick bite in
almost any rural, forested areas of the United States, particularly in the North Western part of the country, in
spring. The typical presentation includes fever, myalgias and a rash, as seen here, and a tick bite can be
identified in around 70% of cases. Doxycycline is the intervention of choice and should be started even before
serology testing is available. It reduces mortality from around 20% to 5%.

A Endemic tick typhus

Endemic tick typhus is transmitted by rat flea bites. It often leads to relatively mild symptoms with fever and
myalgia and is usually rare in developed countries such as the United States.

C Sennetsu fever

https://mypastest.pastest.com/Secure/TestMe/TimedBrowser/1098274#Top 1/3
8/25/22, 9:34 PM MyPastest

Sennetsu fever is caused by Ehrlichia sennetsu and infection is thought to be caused by ingesting raw fish.
Cases are reported in Western Japan and Malaysia.

D Scrub typhus

Scrub typhus is caused by Orientia tsutsugamushi which is spread by larval mites known as chiggers. It is
found in South East Asia, Japan, India, Indonesia, China and Northern Australia.

E Tickborne encephalitis

Tickborne encephalitis typically leads to fever and muscle pains, although the rash fits better with Rocky
Mountain spotted fever, and a rash is not a prominent feature of tickborne encephalitis.
73161

Rate this question:

Next Question

Previous Question Tag Question

Feedback End Review

Difficulty: Average

Peer Responses %

Q. Answered Flagged

Q1

Q2

Q3

Q4

https://mypastest.pastest.com/Secure/TestMe/TimedBrowser/1098274#Top 2/3
8/25/22, 9:34 PM MyPastest

0:00:00/03:00:00

A 23-year-old man is admitted to the Emergency Department in a state of collapse from a local nightclub. He
is severely short of breath and confused and complains of chest pain. His blood pressure is 100/60 mmHg,
and his pulse is 110 bpm and regular. His skin has a blue discolouration. His chest is clear on auscultation.
Oxygen saturation measured on a finger probe is 86% on 60% inspired oxygen.

Investigation Result Normal value

Haemoglobin (Hb) 152 g/l 115–155 g/l

White cell count (WCC) 7.1 × 10 9/l 4–11 × 10 9/l

Platelets (PLT) 192 × 10 9/l 150–400 × 10 9/l

Sodium (Na +) 142 mmol/l 135–145 mmol/l

Potassium (K +) 3.9 mmol/l 3.5–5.0 mmol/l

Creatinine (Cr) 75 µmol/l 50–120 µmol/l

p a(O 2) 50 kPa 11–14.4 kPa

What is the most appropriate intervention?

A Alteplase

B Ascorbic acid

C Exchange transfusion

D Hyperbaric oxygen

E Methylene blue

Explanation 

E Methylene blue

https://mypastest.pastest.com/Secure/TestMe/TimedBrowser/1098274#Top 1/3
8/25/22, 9:34 PM MyPastest

Methylene blue is the accepted first-line intervention for patients with methaemoglobinaemia, the most likely
diagnosis here. It is caused by oxidation of the iron component of haemoglobin from Fe 2+ to Fe 3+ which more
avidly holds on to oxygen. The big pointers here are cyanosis and hypoxia measured on pulse oximetry
despite a normal pa(O 2). Unfortunately, amyl nitrate, which is a cause of methaemoglobinaemia, is readily
available in nightclub settings and may be responsible here. It is given at a dose of 1–2 mg/kg over a period of
five minutes.

A Alteplase

Although pulmonary embolism may present with chest pain, palpitations and hypoxia on pulse oximetry, the
pa(O 2) is not consistent with pulmonary embolism and fits better with methaemoglobinaemia.

B Ascorbic acid

Ascorbic acid is an alternative to methylene blue to treat methaemoglobinaemia. An intravenous dose of two
g of ascorbic acid is usually given.

C Exchange transfusion

Exchange transfusion, like hyperbaric oxygen, is a second- or third-line option in cases where methylene blue
is unsuccessful or contraindicated.

D Hyperbaric oxygen

Hyperbaric oxygen is an alternative intervention to methylene blue in patients in whom methylene blue is
contraindicated such as those with glucose-6-phosphate dehydrogenase (G6PD) deficiency.
73199

Rate this question:

Next Question

Previous Question Tag Question

Feedback End Review

Difficulty: Average

Peer Responses %

https://mypastest.pastest.com/Secure/TestMe/TimedBrowser/1098274#Top 2/3
8/25/22, 9:34 PM MyPastest

0:00:00/03:00:00

A 49-year-old Afro-Caribbean man has hypertension diagnosed by his General Practitioner, with an average
blood pressure of 155/92 mmHg on a 24-hour recording. He has had a normal echocardiogram (ECHO) and
his renal function is normal.

What is the most appropriate treatment for this patient’s hypertension?

A Amlodipine

B Bendroflumethiazide

C Bisoprolol

D Ramipril

E Valsartan

Explanation 

A Amlodipine

The National Institute for Health and Care Excellence (NICE) guidelines suggest offering a calcium channel
blocker (CCB) to adults starting step 1 antihypertensive treatment who:

are aged 55 or over and do not have type II diabetes or


are of Black African or Afro-Caribbean family origin and do not have type II diabetes (of any age).

This makes amlodipine, a long-acting dihydropyridine calcium channel antagonist, the most appropriate
choice. Where calcium channel antagonists are not tolerated due to peripheral oedema, then a thiazide-like
diuretic, such as indapamide, should be considered.

B Bendroflumethiazide

Like beta-blockers, thiazide diuretics are not recommended for the treatment of hypertension because they
carry a small negative impact on glucose tolerance. Thiazide-like diuretics which do not impair glucose
tolerance are preferred instead as an option at step two.

https://mypastest.pastest.com/Secure/TestMe/TimedBrowser/1098274#Top 1/3
8/25/22, 9:34 PM MyPastest

C Bisoprolol

Beta-blockers are not preferred for the initial treatment of hypertension. They have been shown in multiple
meta-analyses to carry a small, but significant, increased risk of type II diabetes.

D Ramipril

Angiotensin-converting enzyme (ACE) inhibitors are the preferred treatment for hypertension in patients with
type II diabetes and those aged under 55 years who are not of Afro-Caribbean origin.

E Valsartan

Angiotensin receptor blockers are considered an option at step two for control of blood pressure in Afro-
Caribbean patients. Calcium channel blockers are the preferred step one therapy.
73115

Rate this question:

Next Question

Previous Question Tag Question

Feedback End Review

Difficulty: Average

Peer Responses %

Q. Answered Flagged

Q1

Q2

https://mypastest.pastest.com/Secure/TestMe/TimedBrowser/1098274#Top 2/3
8/25/22, 9:34 PM MyPastest

0:00:00/03:00:00

A 74-year-old woman is started on 60 mg of prednisolone by her General Practitioner (GP) for severe
bitemporal headaches and pain in her jaw when she eats. She admits to severe tiredness and intermittent
night sweats over the past few months and one episode where she temporarily lost vision in her left eye. On
examination, temporal artery tenderness on palpation is noted.
Investigations checked by her GP are shown below:

Investigation Result Normal value

Haemoglobin (Hb) 109 g/l 115–155 g/l

White cell count (WCC) 9.2 × 10 9/l 4.0–11.0 × 10 9/l

Platelets (PLT) 301 × 10 9/l 150–400 × 10 9/l

Sodium (Na +) 142 mmol/l 135–145 mmol/l

Potassium (K +) 4.5 mmol/l 3.5–5.0 mmol/l

Creatinine (Cr) 132 µmol/l 50–120 µmol/l

Erythrocyte sedimentation rate (ESR) 92 mm/hour < 10 mm/hour

What is the most useful investigation?

A Autoimmune profile

B Carotid Doppler

C Computed tomography (CT) head

D Echocardiography

E Temporal artery biopsy

Explanation 

E Temporal artery biopsy

https://mypastest.pastest.com/Secure/TestMe/TimedBrowser/1098274#Top 1/3
8/25/22, 9:34 PM MyPastest

This is an elderly woman with temporal headaches, jaw claudication and a probable transient ischaemic
attack (TIA). Coupled with the marked rise in erythrocyte sedimentation rate, these symptoms are suggestive
of temporal arteritis. Quite rightly, the General Practitioner has started steroid therapy as it should not be
delayed pending investigations. The confirmatory investigation of choice is a temporal artery biopsy, although
multiple areas may need to be sampled because of the possibility of skip lesions.

A Autoimmune profile

Autoantibody and complement levels are normal in temporal arteritis, meaning that this is not a useful next
step of investigation.

B Carotid Doppler

If there were no signs of temporal arteritis, then carotid Doppler would be the most useful investigation.
Given there are clear signs of temporal arteritis, temporal artery biopsy is much more useful as a next step.

C Computed tomography (CT) head

A CT head may be of value in the event there is a permanent neurological deficit. As the patient has
recovered from the TIA, temporal artery biopsy is potentially more useful in confirming the underlying
diagnosis.

D Echocardiography

Although this patient has suffered a TIA, a more likely cause, in this case, is temporal arteritis, rather than any
intracardiac thrombus.
73206

Rate this question:

Next Question

Previous Question Tag Question

Feedback End Review

Difficulty: Average

Peer Responses %

https://mypastest.pastest.com/Secure/TestMe/TimedBrowser/1098274#Top 2/3
8/25/22, 9:34 PM MyPastest

0:00:00/03:00:00

A 40-year-old man is reviewed in the morning after admission to the Emergency Department following an
overdose of 50 × 500 mg paracetamol tablets.

Which of the following is most associated with completed suicide?

A Children in the same home

B Chronic low-level alcohol consumption

C Family support from Social Services

D Strong religious faith

E Unemployment

Explanation 

E Unemployment

Recent unemployment and other traumatic life events, such as divorce, are strongly associated with
completed suicide. The risk of completed suicide is 5–15 times higher in patients with previous mental illness.
Other factors include excessive alcohol consumption and binge drinking, drug use, single status and recent
admission to the prison service.

A Children in the same home

Having children in the home is a protective factor with respect to successful suicide. They act as a stimulus to
seek help and resolve the underlying cause of suicidal ideation.

B Chronic low-level alcohol consumption

Alcohol abuse, including excessive daily use and binge drinking, is most associated with completed suicide.
Any increased risk is much less in patients who indulge in chronic low-level, controlled alcohol consumption.

C Family support from Social Services

https://mypastest.pastest.com/Secure/TestMe/TimedBrowser/1098274#Top 1/3
8/25/22, 9:35 PM MyPastest

0:00:00/03:00:00

A 53-year-old woman who is undergoing haemodialysis presents to her General Practitioner complaining of
tiredness and lethargy. She is taking oral iron but feels it is not helping her. She looks pale. Her blood
pressure is 119/82 mmHg, and her heart rate is 84 bpm and regular.

Investigation Result Normal value

Haemoglobin (Hb) 82 g/l 115–155 g/l

White cell count (WCC) 6.1 × 10 9/l 4–11 × 10 9/l

Platelets (PLT) 190 × 10 9/l 150–400 × 10 9/l

Sodium (Na +) 142 mmol/l 135–145 mmol/l

Potassium (K +) 4.8 mmol/l 3.5–5.0 mmol/l

Creatinine (Cr) 625 µmol/l 50–120 µmol/l

Ferritin 21 ng/ml 10–300 ng/ml

What is the most appropriate next step?

A Continue oral ferrous sulfate

B Intravenous (IV) iron infusion

C Start erythropoietin

D Start roxadustat

E Switch to spatone iron

Explanation 

B Intravenous (IV) iron infusion

Despite oral iron, this patient’s ferritin level is still well below target. The National Institute for Health and
Care Excellence guidelines, therefore, recommend moving to IV iron as a next step. Most adults require
between 500 and 1000 mg of iron delivered in one or two doses, depending on the preparation. Only when

https://mypastest.pastest.com/Secure/TestMe/TimedBrowser/1098274#Top 1/3
8/25/22, 9:35 PM MyPastest

iron stores are replete should moving to erythropoietin be considered.

A Continue oral ferrous sulfate

Despite oral iron replacement, this patient’s ferritin level is still well below target. Continuing oral ferrous
sulfate is, therefore, not an option.

C Start erythropoietin

Erythropoietin is only indicated for the treatment of renal anaemia once iron stores are adequately
replenished either with oral or IV iron.

D Start roxadustat

Roxadustat is a hypoxia-inducible factor-1 activator, which acts in a similar way to erythropoietin, but it is an
oral medication. It is an option if the patient is still anaemic after iron replacement and do not tolerate or want
to inject erythropoietin.

E Switch to spatone iron

Spatone iron is often better tolerated than iron sulfate, although it contains a slightly lower level of elemental
iron. It is, therefore, not an option for this patient.
73189

Rate this question:

Next Question

Previous Question Tag Question

Feedback End Review

Difficulty: Average

Peer Responses %

https://mypastest.pastest.com/Secure/TestMe/TimedBrowser/1098274#Top 2/3
8/25/22, 9:35 PM MyPastest

0:00:00/03:00:00

A 52-year-old taxi driver presents to the Respiratory Clinic complaining of severe daytime sleepiness and
snoring at night. His wife mentions that he often stops breathing during the early hours of the morning and
seems to be choking. His body mass index is 35 kg/m 2. His Epworth sleepiness score is 18.

What is the most appropriate next intervention?

A Dapagliflozin

B Liraglutide

C Mandibular advancement device

D Modafinil

E Nocturnal continuous positive airway pressure

Explanation 

E Nocturnal continuous positive airway pressure

This patient’s Epworth sleepiness score is significantly elevated and he is obese. Coupled with the snoring,
this points to obstructive sleep apnoea as the most likely diagnosis. At this level of Epworth sleepiness score,
this patient has severe symptoms and requires intervention with nocturnal continuous positive airway
pressure, particularly if he plans to return to taxi driving, as currently, this would be dangerous for him and his
customers. Weight loss is, of course, also important. In milder diseases, a mandibular advancement device
may be of value.

0–5 – lower normal daytime sleepiness


6–10 – higher normal daytime sleepiness
11–12 – mild excessive daytime sleepiness
13–15 – moderate excessive daytime sleepiness
16–24 – severe excessive daytime sleepiness.

A Dapagliflozin

https://mypastest.pastest.com/Secure/TestMe/TimedBrowser/1098274#Top 1/3
8/25/22, 9:35 PM MyPastest

Dapagliflozin is an sodium-glucose cotransporter-2 inhibitor. It is associated with modest weight loss, its
main impact is on reducing cardiovascular risk in patients with heart failure, and progression in patients with
chronic renal impairment.

B Liraglutide

Liraglutide at a high dose is of value for weight loss. This is of course a long-term aim, although it will take
significant time to impact his obstructive sleep apnoea symptoms.

C Mandibular advancement device

Mandibular advancement devices do relieve airway obstruction, although they are only of value in patients
with much milder obstructive sleep apnoea than that seen here, (usual cut-off 11 in the sleepiness score).

D Modafinil

Modafinil is a central nervous system stimulant used in the treatment of narcolepsy, it is not recommended for
the treatment of obstructive sleep apnoea.
73198

Rate this question:

Next Question

Previous Question Tag Question

Feedback End Review

Difficulty: Average

Peer Responses %

Q. Answered Flagged

https://mypastest.pastest.com/Secure/TestMe/TimedBrowser/1098274#Top 2/3
8/25/22, 9:35 PM MyPastest

0:00:00/03:00:00

A 24-year-old man presents to the Emergency Department with a severe unremitting headache. He mentions
that he has been to see his General Practitioner three times over the past six months because of headaches
and he is increasingly tired. Examination reveals some bitemporal visual field loss.

The magnetic resonance imaging (MRI) brain is shown below (T1-weighted scan with gadolinium):


What is the most likely diagnosis?

A Arteriovenous (AV) malformation

B Cerebral lymphoma

C Glioblastoma

D Meningioma

E Schwannoma

Explanation 

D Meningioma

https://mypastest.pastest.com/Secure/TestMe/TimedBrowser/1098274#Top 1/3
8/25/22, 9:35 PM MyPastest

This patient's magnetic resonance imaging (MRI) is a T1 scan with gadolinium enhancement. It reveals
homogenous enhancement of a well-circumscribed tumour arising from the dura at the skull base. This fits
with the diagnosis of a meningioma. The position also potentially accounts for the visual field symptoms seen
in this patient. Given the position, surgical excision may be challenging, meaning that debulking surgery,
followed by radiotherapy, is a potential intervention.

Image credit: By Tdvorak, CC BY-SA 3.0

A Arteriovenous (AV) malformation

Arteriovenous (AV) malformations are best imaged with fast-phase magnetic resonance angiography. The
appearance here is more consistent with a meningioma, given the uniform contrast enhancement on T1
scanning with gadolinium.

B Cerebral lymphoma

Primary central nervous system (CNS) lymphomas do enhance on T1 scanning with contrast. However, the
position and the fact that the tumour arises from the dura make it much more likely to be a meningioma.

C Glioblastoma

Glioblastomas are rapidly growing CNS tumours. They show irregular contrast enhancement, often including
areas of significant necrosis. It does not fit with the position of the tumour or the relatively homogenous
enhancement seen here.

E Schwannoma

Schwannomas are benign encapsulated neoplasms of Schwann cells. They are most commonly seen affecting
the vestibular nerve.
73179

Rate this question:

Next Question

Previous Question Tag Question

Feedback End Review

Difficulty: Average

Peer Responses %

https://mypastest.pastest.com/Secure/TestMe/TimedBrowser/1098274#Top 2/3
8/25/22, 9:35 PM MyPastest

0:00:00/03:00:00

A 32-year-old man presents to the Emergency Department with drowsiness, confusion, severe headache and
worsening neck stiffness. He has a chronic cough and is known to be human immunodeficiency virus (HIV)-
positive. His temperature is 37.9 °C, blood pressure 110/72 mmHg and heart rate 90 bpm and regular. His
Glasgow Coma Scale (GCS) score is 13 and he has a bilateral sixth nerve palsy.

Investigation Result Normal value

Haemoglobin (Hb) 112 g/l 115–155 g/l

White cell count (WCC) 12.9 × 10 9/l 4–11 × 10 9/l

Platelets (PLT) 122 × 10 9/l 150–400 × 10 9/l

Sodium (Na +) 112 mmol/l 135–145 mmol/l

Potassium (K +) 4.5 mmol/l 3.5–5.0 mmol/l

Creatinine (Cr) 112 µmol/l 50–120 µmol/l

C-reactive protein (CRP) 79 mg/l < 10 mg/l

Computed tomography (CT) head with contrast Leptomeningeal enhancement

Cerebrospinal fluid (CSF) opening pressure 27 cmH 2O 7–18 cmH 2O

CSF protein 1.9 g /l 0.2–0.4 g/l

CSF microscopy Lymphocytic pleocytosis

What is the most likely diagnosis?

A Central nervous system lymphoma

B Cryptococcal meningitis

C Herpes simplex encephalitis

D Progressive multifocal leukoencephalopathy

E Tuberculous meningitis

https://mypastest.pastest.com/Secure/TestMe/TimedBrowser/1098274#Top 1/4
8/25/22, 9:35 PM MyPastest

Explanation 

E Tuberculous meningitis

The significant elevation in cerebrospinal fluid (CSF) protein level and lymphocyte predominance on
microscopy, along with elevated opening pressure, raise the possibility of tuberculous meningitis. The chronic
cough against a background of human immunodeficiency virus (HIV) infection further supports this as the
underlying diagnosis. Nucleic acid amplification testing on CSF is the optimal way to confirm the diagnosis.
Quadruple therapy with isoniazid, rifampicin, pyrazinamide and ethambutol is recommended and
prednisolone may also be of value. These antibacterial components can be remembered by the acronym RIPE.
Therapy of up to 12 months is likely to be required.

A Central nervous system lymphoma

Primary central nervous system lymphoma is usually associated with multiple areas of hyper-attenuation
with enhancement on contrast administration, which does not fit with the CT appearance reported here.

B Cryptococcal meningitis

Often, CT head is normal in cryptococcal meningitis which is often associated with a subacute presentation.
The chronic cough here raises the possibility of tuberculosis infection.

C Herpes simplex encephalitis

Herpes simplex encephalitis is associated with progressive memory loss, confusion and abnormal behaviour
and temporal lobe inflammation is seen on scanning.

D Progressive multifocal leukoencephalopathy

Progressive multifocal leukoencephalopathy occurs due to infection with JC virus. CT reveals asymmetric focal
areas of low attenuation involving the periventricular and subcortical white matter.
73167

Rate this question:

Next Question

Previous Question Tag Question

Feedback End Review

Difficulty: Average

https://mypastest.pastest.com/Secure/TestMe/TimedBrowser/1098274#Top 2/4
8/25/22, 9:35 PM MyPastest

0:00:00/03:00:00

A 39-year-old woman is admitted to the Emergency Department with haemoptysis. She has coughed up a
cup of fresh blood and reports lesser degrees of haemoptysis in the past 3–4 weeks. She also has sinusitis,
which she puts down to hay fever. Her blood pressure is 155/90 mmHg, and her heart rate is 95 bpm and
regular. There are inspiratory crackles on auscultation of the chest.

Investigation Result Normal value

Haemoglobin (Hb) 92 g/l 115–155 g/l

White cell count (WCC) 9.9 × 10 9/l 4–11 × 10 9/l

Platelets (PLT) 299 × 10 9/l 150–400 × 10 9/l

Sodium (Na +) 142 mmol/l 135–145 mmol/l

Potassium (K +) 5.9 mmol/l 3.5–5.0 mmol/l

Creatinine (Cr) 321 µmol/l 50–120 µmol/l

Chest X-ray (CXR) Patchy interstitial shadowing through both lung fields

Urine Blood 3+, protein 2+

Which autoantibody is most likely to be found here?

A Anti-mitochondrial

B Anti-Ro

C Anti-smooth muscle

D Classic antineutrophil cytoplasmic antibodies (c-ANCA)

E Perinuclear antineutrophil cytoplasmic antibodies (p-ANCA)

Explanation 

D Classic antineutrophil cytoplasmic antibodies (c-ANCA)

https://mypastest.pastest.com/Secure/TestMe/TimedBrowser/1098274#Top 1/3
8/25/22, 9:35 PM MyPastest

This patient has haemoptysis, sinusitis and disordered renal function. The urine results raise the possibility of
renal vasculitis, putting granulomatosis with polyangiitis as the most likely diagnosis here. Classic
antineutrophil cytoplasmic antibodies (c-ANCA) is positive in around 80–90% of patients. Biopsy of affected
tissue, which can include nasal mucosa, lung or renal tissue, is the main way to confirm the diagnosis.
Corticosteroids and cyclophosphamide remain the mainstay of remission induction therapy.

A Anti-mitochondrial

Anti-mitochondrial antibodies are most strongly associated with primary biliary cirrhosis, not with
granulomatosis with polyangiitis.

B Anti-Ro

Anti-Ro antibodies are found in patients with systemic lupus erythematosus. They are a cause of neonatal
heart block in pregnant women.

C Anti-smooth muscle

Anti-smooth muscle antibodies are characteristic of autoimmune hepatitis. They are not strongly related to
granulomatosis with polyangiitis, the likely diagnosis in this patient.

E Perinuclear antineutrophil cytoplasmic antibodies (p-ANCA)

Typically, p-ANCA is found in approximately 10–15% of cases of granulomatosis with polyangiitis, the most
likely diagnosis here. Most patients (80–90%) are c-ANCA positive.
73195

Rate this question:

Next Question

Previous Question Tag Question

Feedback End Review

Difficulty: Average

Peer Responses %

https://mypastest.pastest.com/Secure/TestMe/TimedBrowser/1098274#Top 2/3
8/25/22, 9:36 PM MyPastest

0:00:00/03:00:00

A 59-year-old man, who smokes 20 cigarettes per day and has had dull right-sided chest ache and shortness
of breath for the past four months, is referred to the Respiratory Clinic. His General Practitioner noted an
abnormal chest X-ray and sent him for a computed tomography (CT) chest.

Investigations:


What is the most likely diagnosis?

A Chylothorax

B Right lower lobe collapse

C Right lower lobe consolidation

D Right pleural effusion

E Right-sided pleural plaque

Explanation 

D Right pleural effusion

https://mypastest.pastest.com/Secure/TestMe/TimedBrowser/1098274#Top 1/3
8/25/22, 9:36 PM MyPastest

This patient has a fluid collection affecting the posterior aspect of the right lung while he is lying down in the
computed tomography (CT) scanner. This is consistent with a right pleural effusion, which may be due to an
underlying malignancy, given his history of smoking. Drainage under ultrasound or CT guidance, with pleural
biopsy, is the next most appropriate step in investigation and treatment.
Image credit: By Hellerhoff , CC BY-SA 3.0.

A Chylothorax

Chylothorax can be a cause of massive fluid accumulation within the lung of near-water density. In this
patient’s age group, it would, however, be expected to occur following surgical disruption or trauma to the
thoracic duct.

B Right lower lobe collapse

Right lower lobe collapse is potentially associated with elevation of the right hemidiaphragm. What is seen
on the CT scan here is not diaphragmatic elevation, but fluid consistent with a pleural effusion.

C Right lower lobe consolidation

Consolidation will appear as non-homogenous areas of increased density within the right lower lobe of the
lung consistent with infection, rather than the homogenous area with a fluid level here, which is due to a
pleural effusion.

E Right-sided pleural plaque

Pleural plaques appear as a thickening affecting the pleura. The collection affecting the posterior aspect of
the right lung with a fluid level is a pleural effusion.
73202

Rate this question:

Next Question

Previous Question Tag Question

Feedback End Review

Difficulty: Average

Peer Responses %

https://mypastest.pastest.com/Secure/TestMe/TimedBrowser/1098274#Top 2/3
8/25/22, 9:36 PM MyPastest

0:00:00/03:00:00

A 71-year-old man is reviewed in the Transient Ischaemic Attack Clinic after a 30-minute episode of left-
sided facial weakness. He admits to weight loss, night sweats and intermittent fevers over the past six
months. His blood pressure is 123/73 mmHg, and his heart rate is 85 bpm and regular. There is moderate
splenomegaly.

Investigation Result Normal value

Haemoglobin (Hb) 115 g/l 115–155 g/l

White cell count (WCC) 7.9 × 10 9/l 4–11 × 10 9/l

Platelets (PLT) 301 × 10 9/l 150–400 × 10 9/l

Sodium (Na +) 143 mmol/l 135–145 mmol/l

Erythrocyte sedimentation rate (ESR) 95 mm/hour < 10 mm/hour

Potassium (K +) 4.5 mmol/l 3.5–5.0 mmol/l

Creatinine (Cr) 167 µmol/l 50–120 µmol/l

Immunoglobulin electrophoresis Immunoglobulin M (IgM) paraprotein band

What is the most likely diagnosis?

A Brucellosis

B Chronic lymphocytic leukaemia

C Chronic myeloid leukaemia

D Parvovirus B19 infection

E Waldenström’s macroglobulinemia

Explanation 

E Waldenström’s macroglobulinemia

https://mypastest.pastest.com/Secure/TestMe/TimedBrowser/1098274#Top 1/3
8/25/22, 9:36 PM MyPastest

This patient has features of hyperviscosity, including the recent transient ischaemic attack, anaemia and an
immunoglobulin M (IgM) paraprotein band. These features fit with a diagnosis of Waldenström’s
macroglobulinemia. Infiltration of the bone marrow with lymphoplasmacytoid lymphoma cells is diagnostic of
the condition and a bone marrow biopsy is a logical step in confirming the diagnosis. Rituximab-based
combination chemotherapy regimens are the usual intervention of choice. Chlorambucil can be used for
particularly frail elderly patients.

A Brucellosis

Brucella infection can lead to anaemia, decreased PLT count and a fall in WCC. Infection usually occurs after
exposure to farm animals and it is considered a rare condition.

B Chronic lymphocytic leukaemia

Chronic lymphocytic leukaemia leads to marked splenomegaly and there is usually a large increase in WCC
where lymphocytes predominate.

C Chronic myeloid leukaemia

Chronic myeloid leukaemia leads to splenomegaly and the WCC is usually elevated to a level above 25 ×
10 9/l. It also would not be expected to lead to the formation of an IgM paraprotein band.

D Parvovirus B19 infection

Parvovirus B19 infection leads to an aplastic crisis in patients with chronic haemolytic anaemia such as those
with sickle cell disease and hereditary spherocytosis.
73158

Rate this question:

Next Question

Previous Question Tag Question

Feedback End Review

Difficulty: Average

Peer Responses %

https://mypastest.pastest.com/Secure/TestMe/TimedBrowser/1098274#Top 2/3
8/25/22, 9:36 PM MyPastest

0:00:00/03:00:00

A 34-year-old man is referred to the Endocrine Clinic with weight gain, hypertension and type II diabetes.

Investigations:

Investigation Result Normal value

Haemoglobin (Hb) 138 g/l 115–155 g/l

White cell count (WCC) 7.2 × 10 9/l 4–11 × 10 9/l

Platelets (PLT) 209 × 10 9/l 150–400 × 10 9/l

Sodium (Na +) 144 mmol/l 135–145 mmol/l

Potassium (K +) 3.4 mmol/l 3.5–5.0 mmol/l

Bicarbonate (HCO 3 −) 34 mmol/l 23–30 mmol/l

Creatinine (Cr) 95 µmol/l 50–120 µmol/l

Glucose 14.5 mmol/l < 11.1 mmol/l

Adrenocorticotropic hormone (ACTH) 65 ng/l 0–50 ng/l

24-hour urinary cortisol 280 nmol/l < 140 nmol

Magnetic resonance imaging (MRI) No pituitary adenoma seen

What is the most likely cause of this patient’s biochemistry results?

A Adenocarcinoma of the bronchus

B Bronchial carcinoid

C Colon cancer

D Exocrine pancreatic cancer

E Papillary thyroid cancer

Explanation 

https://mypastest.pastest.com/Secure/TestMe/TimedBrowser/1098274#Top 1/3
8/25/22, 9:36 PM MyPastest

B Bronchial carcinoid

Bronchial carcinoid tumours are known to secrete adrenocorticotropic hormone (ACTH) and may lead to the
development of Cushing syndrome in younger patients. Small cell carcinomas of the bronchus are recognised
as the most common cause. Other rarer causes include endocrine tumours of the pancreas and sarcomas in
children. Workup for a bronchial tumour, including computed tomography, is a potential next step.

A Adenocarcinoma of the bronchus

Small cell bronchial carcinoma, not adenocarcinoma, is responsible for the development of Cushing syndrome
because of ectopic ACTH secretion.

C Colon cancer

Colon cancers are formed from glandular tissue and are not hormone-secreting tumours.

D Exocrine pancreatic cancer

Endocrine, rather than exocrine, pancreatic cancers may produce ectopic ACTH. This is extremely rare, with
insulin being the most common hormone they produce.

E Papillary thyroid cancer

Medullary thyroid cancer, not papillary thyroid cancer, is a rare cause of ectopic ACTH secretion, leading to
Cushing syndrome, the likely diagnosis here.
73137

Rate this question:

Next Question

Previous Question Tag Question

Feedback End Review

Difficulty: Average

Peer Responses %

https://mypastest.pastest.com/Secure/TestMe/TimedBrowser/1098274#Top 2/3
8/25/22, 9:37 PM MyPastest

0:00:00/03:00:00

A 27-year-old woman is referred to the Emergency Department with a rapidly enlarging mole on her left
forearm. She says the lesion has been bleeding on contact and is painful and itching.

Investigations:


What is the most likely diagnosis?

A Actinic keratosis

B Keratoacanthoma

C Lentigo simplex

D Malignant melanoma

E Papillomatous dermal nevus

Explanation 

D Malignant melanoma

The lesion this patient presented with is consistent with a nodular melanoma. The lesion has grown rapidly, is
raised and has an irregular edge, with some heterogeneity in pigmentation. This is suggestive of a malignant
melanoma that requires urgent wide-margin excision. Prognosis is closely related to the depth of the lesion.

https://mypastest.pastest.com/Secure/TestMe/TimedBrowser/1098274#Top 1/3
8/25/22, 9:37 PM MyPastest

Image credit: By DermNetNZ - StatPearls Publishing LLC, Contributed by DermNetNZ- Creative Commons
Attribution 4.0 International License

A Actinic keratosis

Actinic keratoses occur due to sun exposure, appearing initially as rough areas of skin. Later they become
erythematous, with some skin scaling.

B Keratoacanthoma

Keratoacanthomas resemble squamous cell carcinomas. They are nodular lesions with a central keratotic,
necrotic area.

C Lentigo simplex

Lentigo simplex lesions are not raised. They are usually brown in colour and contain increased numbers of
melanocytes, present from birth. They have a defined edge and are not malignant.

E Papillomatous dermal nevus

Papillomatous dermal lesions are pigmented and have structural similarity to warts. They are benign lesions.
73133

Rate this question:

Next Question

Previous Question Tag Question

Feedback End Review

Difficulty: Average

Peer Responses %

https://mypastest.pastest.com/Secure/TestMe/TimedBrowser/1098274#Top 2/3
8/25/22, 9:37 PM MyPastest

0:00:00/03:00:00

A 35-year-old woman has loss of appetite, early satiety, dyspepsia, nausea, bloating and flatulence. She
underwent a small bowel resection for Crohn’s disease three years earlier but has been well up until the last
three months. Her abdomen is mildly distended, but soft, and there is no significant tenderness. A hydrogen
breath test shows an increase of over 20 parts per million in exhaled hydrogen after the test.
What is the most appropriate intervention?

A Amoxicillin

B Budesonide

C Mesalamine

D Prednisolone

E Rifaximin

Explanation 

E Rifaximin

The symptoms reported here and the significant rise in exhaled hydrogen are consistent with a diagnosis of
bacterial overgrowth syndrome where there are abnormal levels of bacteria within the small bowel. A drink
containing sugars is given as part of the test and they are metabolised by the bacteria into excess exhaled
hydrogen. Rifaximin, tetracyclines and metronidazole are all used as potential treatments for bacterial
overgrowth.

A Amoxicillin

This patient has symptoms of bacterial overgrowth syndrome, potentially due to previous bowel resection.
Tetracyclines, rifaximin or metronidazole, not amoxicillin, are recommended interventions.

B Budesonide

https://mypastest.pastest.com/Secure/TestMe/TimedBrowser/1098274#Top 1/3
8/25/22, 9:37 PM MyPastest

Budesonide is used in modified-release form for the treatment of active Crohn’s disease. There is no strong
evidence this patient’s Crohn’s disease is active and the hydrogen breath test fits with bacterial overgrowth
syndrome.

C Mesalamine

Mesalamine is a 5-aminosalicylic acid compound used to maintain Crohn’s disease remission. It has no utility
in the treatment of bacterial overgrowth syndrome, the diagnosis here.

D Prednisolone

Prednisolone is of most value in treating an exacerbation of inflammatory bowel disease. The hydrogen
breath test suggests bacterial overgrowth which is, in fact, the cause of the patient’s symptoms.
73144

Rate this question:

Next Question

Previous Question Tag Question

Feedback End Review

Difficulty: Average

Peer Responses %

Q. Answered Flagged

Q1

Q2

Q3

https://mypastest.pastest.com/Secure/TestMe/TimedBrowser/1098274#Top 2/3
8/25/22, 9:37 PM MyPastest

0:00:00/03:00:00

A 31-year-old drug abuser is brought to the Emergency Department by her partner. She has been confined to
bed for the past 12 hours. She has a fever, facial spasms and neck stiffness and is unable to swallow properly.
Her temperature is 37.9 °C, blood pressure 123/82 mmHg and heart rate 89 bpm. An injection wound is
noted on her left foot, surrounded by erythema.
Investigations:

Investigation Result Normal value

Haemoglobin (Hb) 102 g/l 115–155 g/l

White cell count (WCC) 13.1 × 10 9/l 4–11 × 10 9/l

Platelets (PLT) 95 × 10 9/l 150–400 × 10 9/l

Sodium (Na +) 142 mmol/l 135–145 mmol/l

Potassium (K +) 3.9 mmol/l 3.5–5.0 mmol/l

Creatinine (Cr) 90 µmol/l 50–120 µmol/l

What is the most likely diagnosis?

A Botulism

B Guillain–Barré syndrome

C Miller Fisher syndrome

D Myasthenia gravis

E Tetanus

Explanation 

E Tetanus

The most likely diagnosis here is tetanus as a result of infection via the injection wound. It fits with the fever
and spastic muscle weakness affecting central function, including swallowing. Early diagnosis is crucial, with
time to administration of the tetanus antitoxin closely linked to prognosis. Most patients are considered for
https://mypastest.pastest.com/Secure/TestMe/TimedBrowser/1098274#Top 1/3
8/25/22, 9:37 PM MyPastest

prophylactic sedation, intubation and ventilation. Metronidazole may be of additional value in the clearance of
tetanus infection.

A Botulism

Botulism does occur as a result of injection drug abuse. However, it leads to flaccid paralysis, rather than to
the increased tone and muscle spasms seen in this patient.

B Guillain–Barré syndrome

Guillain–Barré syndrome is associated with ascending flaccid paralysis, which often presents after a viral
infection.

C Miller Fisher syndrome

Miller Fischer syndrome is a variant of Guillain–Barré syndrome, which is associated with progressive flaccid
muscle weakness that begins centrally.

D Myasthenia gravis

Myasthenia gravis is associated with slowly progressive, predominantly proximal muscle weakness and the
presence of anti-acetylcholine receptor antibodies.
73175

Rate this question:

Next Question

Previous Question Tag Question

Feedback End Review

Difficulty: Average

Peer Responses %

https://mypastest.pastest.com/Secure/TestMe/TimedBrowser/1098274#Top 2/3
8/25/22, 9:37 PM MyPastest

0:00:00/03:00:00

A 28-year-old man is referred to the Respiratory Clinic because of recurrent lower respiratory tract infections.
A computed tomography (CT) scan of his chest is taken.

Investigations:


What is the most likely cause of this patient’s symptoms?

A Asthma

B Cigarette smoking

C Pertussis infection as a child

D Primary ciliary dysmotility

E Selective immunoglobulin G (IgG) deficiency

Explanation 

C Pertussis infection as a child

This patient’s computed tomography (CT) shows thickened, dilated bronchi, with some displaying the signet
ring sign, with a prominent pulmonary artery at the edge of the bronchial ring looking like a signet ring.
Around 40% of cases of bronchiectasis are said to develop a post-viral or bacterial infection, which can
https://mypastest.pastest.com/Secure/TestMe/TimedBrowser/1098274#Top 1/3
8/25/22, 9:37 PM MyPastest

include pertussis, influenza and herpes zoster, to name but three examples. Other causes of bronchiectasis
are rarer, although, in up to half of adults, no formal cause is identified. Patients suffering repeated infections
may be considered for antibiotic rotations.

Image credit: By Mcgfowler, CC BY-SA 3.0.

A Asthma

Asthma is a less likely cause of bronchiectasis than childhood viral infections. Bronchiectasis may be
considered in patients with asthma who get recurrent unusual lower respiratory tract infections.

B Cigarette smoking

Cigarette smoking is a risk factor for worsening bronchiectasis. However, in this case, the primary cause of the
disease is likely to be childhood infection.

D Primary ciliary dysmotility

Congenital conditions as a whole, including cystic fibrosis, alpha-1-antitrypsin deficiency, primary ciliary
dysmotility and others, only make up 3–4% of the total causes of bronchiectasis.

E Selective immunoglobulin G (IgG) deficiency

Around 10–15% of the population have selective IgG4 deficiency. However, immunodeficiency of all types
only accounts for between 1% and 8% of cases of bronchiectasis, which is the diagnosis here.
73201

Rate this question:

Next Question

Previous Question Tag Question

Feedback End Review

Difficulty: Average

Peer Responses %

https://mypastest.pastest.com/Secure/TestMe/TimedBrowser/1098274#Top 2/3
8/25/22, 9:37 PM MyPastest

0:00:00/03:00:00

A 27-year-old man presents to the Emergency Department with central chest pain. The pain is worse when
he breathes in and is made worse by lying down. He tells you he has been ill with symptoms of a cold over
the past few days.

His electrocardiogram (ECG) is shown below:


What is the most likely diagnosis?

A Acute pericarditis

B Myocarditis

C Non-ST-segment elevation myocardial infarction

D ST-segment elevation myocardial infarction

E Unstable angina

Explanation 

A Acute pericarditis

This patient’s central chest pain, which is worse on lying flat, coupled with concave ST-segment elevation, is
very typical of acute pericarditis, which is seen after infection with a number of respiratory viruses, including
influenza and Coxsackie B. Non-steroidals are the intervention of choice, with colchicine also potentially
indicated to reduce the risk of chronic pericarditis.
Image source: CC BY-SA 4.0, https://commons.wikimedia.org/w/index.php?curid=40647847

https://mypastest.pastest.com/Secure/TestMe/TimedBrowser/1098274#Top 1/3
8/25/22, 9:37 PM MyPastest

B Myocarditis

The ECG changes in myocarditis may be a mix of those seen in pericarditis and myocardial infarction. More
significant symptoms and signs may be expected in myocarditis such as shortness of breath and tachycardia.

C Non-ST-segment elevation myocardial infarction

Non-ST-segment elevation myocardial infarction is associated with ischaemic chest pain, and T wave
changes, such as T wave inversion or ST-segment depression would be expected.

D ST-segment elevation myocardial infarction

ST-segment elevation myocardial infarction is associated with ischaemic, rather than pericarditic, chest pain
and convex, rather than concave, ST-segment elevation.

E Unstable angina

The pain described here is more consistent with pericarditis than with ischaemic cardiac chest pain. The ECG
also shows concave, rather than convex, ST-segment changes.
73111

Rate this question:

Next Question

Previous Question Tag Question

Feedback End Review

Difficulty: Average

Peer Responses %

https://mypastest.pastest.com/Secure/TestMe/TimedBrowser/1098274#Top 2/3
8/25/22, 9:37 PM MyPastest

0:00:00/03:00:00

A 45-year-old woman presents to her General Practitioner with weight gain, lethargy, menorrhagia and a
goitre. Examination reveals a blood pressure of 150/90 mmHg and a body mass index of 29 kg/m 2.

Investigation Result Normal value

Thyroid-stimulating hormone (TSH) 17.1 U/ml 0.5–4.2 U/ml

High-density lipoprotein (HDL) cholesterol 0.91 mmol/l > 0.9 mmol/l

Low-density lipoprotein (LDL) cholesterol 3.5 mmol/l < 3.0 mmol/l

Triglycerides 3.4 mmol/l < 2.3 mmol/l

What is the most appropriate next step for managing this patient’s lipids?

A Atorvastatin

B Fenofibrate

C Thyroxine and fenofibrate

D Thyroxine and atorvastatin

E Thyroxine, then review lipids in three months

Explanation 

E Thyroxine, then review lipids in three months

Hypothyroidism is associated with the pattern of dyslipidaemia seen here, with a rise in both triglyceride and
low-density lipoprotein cholesterol levels. Lipids may well normalise following a period of thyroxine
treatment. Given there are well-described adverse effects of both statins and fibrates, it is appropriate to wait
for three months and then reassess.

A Atorvastatin

This patient may have mixed hyperlipidaemia before she acquired hypothyroidism. However, lipid-lowering
therapy should be started only after lipids are reviewed following a period of thyroxine treatment.
https://mypastest.pastest.com/Secure/TestMe/TimedBrowser/1098274#Top 1/3
8/25/22, 9:37 PM MyPastest

B Fenofibrate

Fenofibrate is effective in reducing triglycerides. However, its impact on outcomes is unproven. It should only
be considered after a period of thyroxine treatment.

C Thyroxine and fenofibrate

Thyroxine may normalise this patient’s lipids. Lipid-lowering therapy should therefore not be considered until
a period of thyroxine treatment has passed.

D Thyroxine and atorvastatin

Atorvastatin should only be considered after lipids are retested following three months of thyroxine therapy.
They may well have normalised once the hypothyroidism is corrected.
73138

Rate this question:

Next Question

Previous Question Tag Question

Feedback End Review

Difficulty: Average

Peer Responses %

Q. Answered Flagged

Q1

Q2

https://mypastest.pastest.com/Secure/TestMe/TimedBrowser/1098274#Top 2/3
8/25/22, 9:37 PM MyPastest

0:00:00/03:00:00

A 74-year-old woman who was previously independent is reviewed on the Orthogeriatric Ward following a
fractured neck of femur. Although she was previously well, she is confused and deemed not to have capacity.

Which of the following is the most important with respect to proceeding to surgery?

A Judgement of the Court of Protection

B Judgement of the hospital solicitor

C Next of kin consent

D Proceeding in the best interests of the patient

E Verbal consent from the patient even though she is confused

Explanation 

D Proceeding in the best interests of the patient

Unless an advance directive has been issued, the treating medical team make a decision to proceed with
therapy according to the patient’s best interest. The team should take into account any prior religious beliefs
of the patient and how these may impact on treatment, trying to involve the patient in the decision, if
possible, and deciding whether it is appropriate to wait until the patient may regain capacity. Obviously, in the
case of a fractured neck of femur, surgery should be done as soon as possible because of the risk of venous
thromboembolism.

A Judgement of the Court of Protection

Where there is a serious dispute about what is in a patient’s best interests, referral to the Court of Protection
to make a decision can be undertaken. In this case, the decision should be relatively straightforward.

B Judgement of the hospital solicitor

Judgement of the hospital solicitor should not be required. They are not medically qualified, although they
may take expert testimony, and the clinical team should be able to decide on intervention.

https://mypastest.pastest.com/Secure/TestMe/TimedBrowser/1098274#Top 1/3
8/25/22, 9:37 PM MyPastest

C Next of kin consent

Next of kin consent carries much less weight than acting in the patient’s best interests, because the medical
team are much more qualified to weigh the pros and cons of intervention, compared to a relative.

E Verbal consent from the patient even though she is confused

Unfortunately, verbal consent from a patient who is deemed not to have capacity does not carry any weight
with respect to proceeding to surgery.
73152

Rate this question:

Next Question

Previous Question Tag Question

Feedback End Review

Difficulty: Average

Peer Responses %

Q. Answered Flagged

Q1

Q2

Q3

Q4

Q5

https://mypastest.pastest.com/Secure/TestMe/TimedBrowser/1098274#Top 2/3
8/25/22, 9:38 PM MyPastest

0:00:00/03:00:00

A 54-year-old woman is referred to the Renal Clinic because of deteriorating renal function that her General
Practitioner has been monitoring over the past few months. She admits to taking Xi Xin, a Chinese herb which
is designed to increase energy. Her blood pressure is 155/90 mmHg. Her abdomen is soft and non-tender,
with no palpable masses. Her body mass index is 24 kg/m 2.

Investigation Result Normal value

Haemoglobin (Hb) 98 g/l 115–155 g/l

White cell count (WCC) 6.9 × 10 9/l 4–11 × 10 9/l

Platelets (PLT) 191 × 10 9/l 150–400 × 10 9/l

Sodium (Na +) 142 mmol/l 135–145 mmol/l

Potassium (K +) 5.0 mmol/l 3.5–5.0 mmol/l

Creatinine (Cr) 235 µmol/l 50–120 µmol/l

Urine protein : creatinine ratio 15 mg/mmol < 3 mg/mmol

What is the most likely diagnosis?

A Acute interstitial nephritis

B Chronic interstitial nephritis

C Hypertensive nephropathy

D Membranous glomerulonephritis

E Minimal change glomerulonephritis

Explanation 

B Chronic interstitial nephritis

https://mypastest.pastest.com/Secure/TestMe/TimedBrowser/1098274#Top 1/3
8/25/22, 9:38 PM MyPastest

Xi Xin is Chinese wild ginger, which is recognised as a cause of chronic interstitial nephritis. This fits with the
presentation of anaemia, abnormal creatinine (Cr) and moderately elevated urinary protein. Nephrotoxicity is
thought to be due to aristolochic acid which is present in the herb, with extensive interstitial nephritis as a
prominent feature. Corticosteroids may slow the rate of loss of renal function. Other causes of chronic
interstitial nephritis include the use of 5-aminosalicylic acid (5-ASA) compounds and phenacetin (a now
discontinued analgesic).

A Acute interstitial nephritis

Acute interstitial nephritis is associated with a rapid deterioration in renal function and a raised eosinophil
count. It is seen in association with agents such as beta-lactam antibiotics.

C Hypertensive nephropathy

Blood pressure is only moderately elevated and this patient has not been noted as being hypertensive before.
Given the use of Xi Xin, chronic interstitial nephritis is a more likely diagnosis.

D Membranous glomerulonephritis

Membranous glomerulonephritis may be idiopathic or linked to underlying inflammatory disease or


malignancy. It is usually associated with more marked proteinuria than that seen in this patient.

E Minimal change glomerulonephritis

Minimal change glomerulonephritis occurs predominantly in children and young adults and is associated with
a more marked elevation in proteinuria than that seen here. It usually responds to corticosteroid therapy.
73127

Rate this question:

Next Question

Previous Question Tag Question

Feedback End Review

Difficulty: Average

Peer Responses %

https://mypastest.pastest.com/Secure/TestMe/TimedBrowser/1098274#Top 2/3
8/25/22, 9:38 PM MyPastest

0:00:00/03:00:00

A 24-year-old man presents to the Emergency Department with a painless ulcer on the glans penis. He
mentions having unprotected sexual intercourse with two women during a recent trip to Thailand.

What is the most likely diagnosis?

A Bartonella infection

B Chlamydia

C Gonorrhoea

D Herpes simplex

E Syphilis

Explanation 

E Syphilis

This patient has a single painless ulcer on the glans penis after unprotected sexual intercourse. This is most
consistent with syphilis infection. The polymerase chain reaction of material from the surface of the ulcer, if
available, is the optimal investigation. The treponemal enzyme immunoassay and chemiluminescent assay are
standard recommended tests. Screening for other sexually transmitted infections and human
immunodeficiency virus is essential. A single dose of intramuscular penicillin is the recommended treatment.

A Bartonella infection

Bartonella infection is the cause of cat-scratch fever, which is usually self-limiting in patients who are not
immunocompromised.

B Chlamydia

Chlamydia is associated with symptoms of urethritis, but not with the single painless ulcer seen in this
patient.

C Gonorrhoea
https://mypastest.pastest.com/Secure/TestMe/TimedBrowser/1098274#Top 1/3
8/25/22, 9:38 PM MyPastest

Gonorrhoea is a cause of urethritis and penile discharge. It is not a cause of painless ulceration on the glans
penis. Ceftriaxone 1 g IM is the intervention of choice for gonorrhoea infection.

D Herpes simplex

Herpes simplex is a cause of urethritis and multiple painful ulcers. It does not, therefore, fit with the clinical
picture seen in this patient.
73165

Rate this question:

Next Question

Previous Question Tag Question

Feedback End Review

Difficulty: Average

Peer Responses %

Q. Answered Flagged

Q1

Q2

Q3

Q4

Q5

Q6

https://mypastest.pastest.com/Secure/TestMe/TimedBrowser/1098274#Top 2/3
8/25/22, 9:38 PM MyPastest

0:00:00/03:00:00

A 74-year-old man who is receiving end-of-life care for small cell lung cancer with cerebral metastases
presents to the Oncology Ward. He is asleep, but you note he has excessive respiratory secretions, which
seem to be causing him distress.

What is the most appropriate intervention?

A Diazepam

B Donepezil

C Glycopyrronium

D Hydrocortisone

E Morphine sulfate

Explanation 

C Glycopyrronium

Glycopyrronium, like hyoscine, is an anticholinergic agent. Both are thought to have value in controlling
excessive respiratory secretions in end-of-life care and are most often given subcutaneously. A number of
small studies have been conducted comparing glycopyrronium to hyoscine, but as yet, neither drug appears to
have a clear advantage. A dose of 200 μg every four hours is usually given to control respiratory secretions.

A Diazepam

Diazepam is primarily used to manage agitation in end-of-life care. This patient’s primary problem is
excessive respiratory secretions, for which an anticholinergic agent is the most appropriate intervention.

B Donepezil

Donepezil is an acetylcholinesterase inhibitor given for the treatment of Alzheimer’s dementia. It does not
have a role in the treatment of excessive respiratory secretions.

D Hydrocortisone
https://mypastest.pastest.com/Secure/TestMe/TimedBrowser/1098274#Top 1/3
8/25/22, 9:38 PM MyPastest

Corticosteroids are of no value in controlling excess respiratory secretions in this situation. They may be of
value in palliative care in controlling other symptoms, specifically as an adjunct in controlling nausea and
vomiting and, in some cases, for cerebral oedema.

E Morphine sulfate

It appears pain relief is not the most important need for this patient and may precipitate significant respiratory
depression. An anticholinergic agent is the preferred next step.
73182

Rate this question:

Next Question

Previous Question Tag Question

Feedback End Review

Difficulty: Average

Peer Responses %

Q. Answered Flagged

Q1

Q2

Q3

Q4

Q5

Q6

https://mypastest.pastest.com/Secure/TestMe/TimedBrowser/1098274#Top 2/3
8/25/22, 9:38 PM MyPastest

0:00:00/03:00:00

A 40-year-old woman presents to her General Practitioner for advice. She underwent chemotherapy and
radiotherapy for Hodgkin’s lymphoma with mediastinal involvement 15 years ago.

Which tumour has the highest incidence post-Hodgkin’s treatment?

A Bladder carcinoma

B Breast carcinoma

C Bronchial carcinoma

D Non-Hodgkin’s lymphoma

E Renal carcinoma

Explanation 

B Breast carcinoma

The incidence of breast cancer is highest in women who undergo treatment for Hodgkin’s lymphoma at
16.6%, and the standardised incidence ratio is 4.7. The risk of breast cancer in patients undergoing treatment
for Hodgkin’s lymphoma appears to be higher with treatment as a child or as a younger adult.

A Bladder carcinoma

The incidence of bladder cancer post-Hodgkin’s treatment is low at 0.6% over 30 years, with a standardised
incidence ratio of 4.1.

C Bronchial carcinoma

The 30-year incidence of bronchial carcinoma is very significantly increased post-treatment for Hodgkin’s
lymphoma at 6.4. This is the same as the increase in the standardised incidence ratio.

D Non-Hodgkin’s lymphoma

https://mypastest.pastest.com/Secure/TestMe/TimedBrowser/1098274#Top 1/3
8/25/22, 9:38 PM MyPastest

The 30-year incidence of non-Hodgkin’s lymphoma is 3.7%, and the standardised incidence ratio is massively
increased to 13.4.

E Renal carcinoma

The 30-year cumulative incidence of renal cancer is 0.4% post-Hodgkin’s lymphoma treatment, and the
standardised incidence ratio is 2.3.
73181

Rate this question:

Next Question

Previous Question Tag Question

Feedback End Review

Difficulty: Average

Peer Responses %

Q. Answered Flagged

Q1

Q2

Q3

Q4

Q5

Q6

Q7

https://mypastest.pastest.com/Secure/TestMe/TimedBrowser/1098274#Top 2/3
8/25/22, 9:38 PM MyPastest

0:00:00/03:00:00

A 54-year-old man is brought to the Emergency Department following a collapse at the local airport. His
blood pressure is 90/60 mmHg and his heart rate is 110 bpm and regular. His oxygen saturation is 90% on
60% inspired oxygen.

Computed tomography (CT):


What is the most likely diagnosis?

A Acute myocardial infarction

B Dissecting thoracic aneurysm

C Myocarditis

D Pulmonary embolism

E Pulmonary artery dissection

Explanation 

D Pulmonary embolism

The computed tomography (CT) pulmonary angiogram clearly demonstrates a filling defect at the bifurcation
of the pulmonary artery. This is consistent with a large pulmonary embolism that may have been due to a
plane journey. At this stage, given the patient’s state of circulatory collapse and hypoxia, thrombolysis is the
most appropriate next step.

https://mypastest.pastest.com/Secure/TestMe/TimedBrowser/1098274#Top 1/3
8/25/22, 9:38 PM MyPastest

Image credit: By James Heilman, MD -, CC BY-SA 3.0.

A Acute myocardial infarction

Acute myocardial infarction may be a cause of the collapse. However, it would not account for the degree of
hypoxia seen here nor for the changes seen on CT pulmonary angiography.

B Dissecting thoracic aneurysm

The patient's CT shows the pulmonary arteries, not the aorta and its branches. A filling defect can be
observed, consistent with pulmonary embolism.

C Myocarditis

Myocarditis presents with progressively worsening heart failure and would not fit with the CT findings of a
pulmonary artery clot seen here.

E Pulmonary artery dissection

Pulmonary artery dissection is extremely rare and usually seen as a result of chronic pulmonary hypertension.
The CT demonstrates an intimal flap, rather than the filling defect seen here consistent with a clot.
73204

Rate this question:

Next Question

Previous Question Tag Question

Feedback End Review

Difficulty: Average

Peer Responses %

https://mypastest.pastest.com/Secure/TestMe/TimedBrowser/1098274#Top 2/3
8/25/22, 9:39 PM MyPastest

0:00:00/03:00:00

A 72-year-old woman is referred to the Dermatology Clinic with a blistering rash which has slowly developed
over the past six months. She tells you the blisters are itchy and predominantly occur over the body and both
lower limbs. The oral cavity is relatively spared.


What is the most likely diagnosis?

A Dermatitis herpetiformis

B Erythema marginatum

C Erythema nodosum

D Pemphigoid

https://mypastest.pastest.com/Secure/TestMe/TimedBrowser/1098274#Top 1/4
8/25/22, 9:39 PM MyPastest

E Pemphigus

Explanation 

D Pemphigoid

Bullous pemphigoid fits with the rash seen here, affecting the body and lower limbs, with relative sparing of
the oral cavity. It is caused by the production of immunoglobulin G (IgG) and immunoglobulin E (IgE)
autoantibodies against hemi-desmosomal proteins known as BP180 and BP230. This leads to neutrophil
chemotaxis and degradation of the basement membrane zone. It is the most common autoimmune blistering
disease. Corticosteroids are the mainstay of therapy.
Image credit: By Mohammad2018 - Own work, CC BY-SA 4.0,

A Dermatitis herpetiformis

Dermatitis herpetiformis is associated with gluten enteropathy and is characterised by the development of
small vesicles over the buttocks and posterior thighs initially.

B Erythema marginatum

Erythema marginatum is characterised by annular erythematous lesions which occur on the torso and flexural
surfaces of the limbs.

C Erythema nodosum

Erythema nodosum leads to raised nodular erythematous lesions on both shins. It is associated with
tuberculosis, sarcoidosis and pregnancy, among a range of other conditions.

E Pemphigus

Pemphigus is associated with a bullous rash like that seen here. However, there are almost always significant
blistering lesions within the mouth, which are not a feature in this patient.
73128

Rate this question:

Next Question

Previous Question Tag Question

https://mypastest.pastest.com/Secure/TestMe/TimedBrowser/1098274#Top 2/4
8/25/22, 9:39 PM MyPastest

0:00:00/03:00:00

A 24-year-old man is brought to the Emergency Department in an agitated state. He says that everyone
given the coronavirus vaccine is being controlled by the government and that the jab is a mechanism to insert
thoughts into his and everyone else’s minds. He also believes that because he has been vaccinated, he is
transmitting his thoughts to the police via a 5G mast. He has tried to cut the skin away from his arm where he
was vaccinated and is carrying a quantify of marijuana.

What is the most likely diagnosis?

A Anankastic personality disorder

B Bipolar disorder

C Borderline personality disorder

D Conversion disorder

E Schizophrenia

Explanation 

E Schizophrenia

This patient's ideation that the government is controlling the population via coronavirus vaccination and using
it for thought broadcasting and thought insertion is highly suggestive of an acute presentation with
schizophrenia. Other symptoms that may be elicited in acute schizophrenia include primary delusional
perceptions, eg ‘there is a red car at the end of the road so I know the government is watching me’. Atypical
antipsychotics are the mainstay of initial therapy for acute schizophrenia, with agents such as risperidone as
the preferred treatment.

A Anankastic personality disorder

An anankastic personality disorder leads to emotional and behavioural constraints, including inflexibility and
risk avoidance. Patients with anankastic traits also rigidly adhere to rules and aim for perfection in every
activity they undertake.

B Bipolar disorder

https://mypastest.pastest.com/Secure/TestMe/TimedBrowser/1098274#Top 1/3
8/25/22, 9:39 PM MyPastest

Bipolar disorder is more likely to lead to delusions of grandeur and self-importance, rather than the picture
seen here with thought broadcasting and thought insertion.

C Borderline personality disorder

Borderline personality disorder involves frantic efforts to avoid perceived or real abandonment, intense and
unstable personal relationships, a tendency to act rashly and frequent episodes of self-harm.

D Conversion disorder

Conversion disorder is manifested by a neurological deficit that cannot be explained by medical evaluation
and is triggered by an extreme psychological stressor.
73186

Rate this question:

Next Question

Previous Question Tag Question

Feedback End Review

Difficulty: Average

Peer Responses %

Q. Answered Flagged

Q1

Q2

Q3

Q4

https://mypastest.pastest.com/Secure/TestMe/TimedBrowser/1098274#Top 2/3
8/25/22, 9:39 PM MyPastest

0:00:00/03:00:00

A 28-year-old woman who has had two previous admissions to the Intensive Therapy Unit with acute severe
asthma is reviewed 30 minutes after admission to the Emergency Department with severe shortness of
breath. She has received back-to-back salbutamol and ipratropium nebulisers, as well as intravenous
hydrocortisone. Her blood pressure is 90/60 mmHg and her heart rate is 115 bpm and regular. Her
respiratory rate is 32 breaths per minute and she looks tired. There is bilateral wheeze on auscultation. Her
oxygen saturation on 60% inspired oxygen is 91%. She is unable to perform a peak flow.

What is the most appropriate intervention?

A Aminophylline

B Bilevel positive airway pressure ventilation

C Co-amoxiclav

D Intubation and mechanical ventilation

E Magnesium

Explanation 

D Intubation and mechanical ventilation

There are a number of features of life-threatening asthma here, including hypotension, inability to perform a
peak flow, and very low oxygen saturation despite 60% inspired oxygen. The patient is at imminent risk of
respiratory arrest and progression to mechanical ventilation is advised. Specific relevant features, in this case,
include oxygen saturation <92%, peak flow below 33% of best or predicted and hypotension.

A Aminophylline

Aminophylline will not be of benefit here, given the patient is adequately treated with beta-agonist therapy.
The only realistic option is to progress to mechanical ventilation.

B Bilevel positive airway pressure ventilation

https://mypastest.pastest.com/Secure/TestMe/TimedBrowser/1098274#Top 1/3
8/25/22, 9:39 PM MyPastest

Bilevel positive airway pressure ventilation is recommended for the management of chronic obstructive
pulmonary disease. It is not recommended in the management of asthma where progression to mechanical
ventilation is preferred.

C Co-amoxiclav

Although antibiotics may be of value in some cases of asthma exacerbation, they will not have an impact in
the short term and the majority of exacerbations occur as a post-viral phenomenon.

E Magnesium

Magnesium is recommended for acute severe asthma. However, this patient is at imminent risk of respiratory
arrest and should be intubated and ventilated.
73200

Rate this question:

Next Question

Previous Question Tag Question

Feedback End Review

Difficulty: Average

Peer Responses %

Q. Answered Flagged

Q1

Q2

Q3

https://mypastest.pastest.com/Secure/TestMe/TimedBrowser/1098274#Top 2/3
8/25/22, 9:39 PM MyPastest

0:00:00/03:00:00

A 37-year-old man who has left-sided cluster headaches presents to the Emergency Department with
worsening symptoms. As well as indomethacin to cope with attacks, he has been taking regular paracetamol
and codeine almost every day. His main complaint is morning headaches that last for 2–3 hours each
morning. Neurological examination is normal.
What is the most appropriate next step?

A Addition of amitriptyline

B Addition of sumatriptan

C Addition of verapamil

D Withdrawal of codeine

E Withdrawal of codeine and paracetamol

Explanation 

E Withdrawal of codeine and paracetamol

This pattern of symptoms fits with a diagnosis of medication overuse headache with headaches occurring
almost every morning for a number of hours. The most appropriate intervention is the withdrawal of this
patient’s regular analgesics (codeine and paracetamol). Often patients report they have to take analgesia to
get rid of the daily headache, which sets up a vicious circle of medication overuse. Common triggers for
headaches, such as smoking, stress and lack of sleep, should also be avoided, if possible.

A Addition of amitriptyline

Amitriptyline is useful for migraine prophylaxis. It does not have a role in the treatment of medication overuse
headaches.

B Addition of sumatriptan

Triptans are best used as an acute intervention for migraine. A number of delivery options are available,
including intranasal and sublingual preparations.

https://mypastest.pastest.com/Secure/TestMe/TimedBrowser/1098274#Top 1/3
8/25/22, 9:39 PM MyPastest

C Addition of verapamil

Verapamil is an alternative intervention for migraine prophylaxis. It does not have a role in managing this
patient’s symptoms which are suggestive of overuse of analgesics.

D Withdrawal of codeine

Although the withdrawal of codeine alone may improve this patient’s symptoms, ideally both medications
should be discontinued from regular use.
73172

Rate this question:

Next Question

Previous Question Tag Question

Feedback End Review

Difficulty: Average

Peer Responses %

Q. Answered Flagged

Q1

Q2

Q3

Q4

Q5

https://mypastest.pastest.com/Secure/TestMe/TimedBrowser/1098274#Top 2/3
8/25/22, 9:39 PM MyPastest

0:00:00/03:00:00

A 55-year-old man who has hypertension and obesity presents to the Emergency Department with acute
pain and swelling affecting his right knee. He says he has had lesser episodes in the past, but he is now
unable to flex the knee properly or walk on it.

Investigation Result Normal value

Haemoglobin (Hb) 141 g/l 115–155 g/l

White cell count (WCC) 9.2 × 10 9/l 4.0–11.0 × 10 9/l

Platelets (PLT) 198 × 10 9/l 150–400 × 10 9/l

Sodium (Na +) 143 mmol/l 135–145 mmol/l

Potassium (K +) 4.4 mmol/l 3.5–5.0 mmol/l

Creatinine (Cr) 95 µmol/l 50–120 µmol/l

Uric acid 450 µmol/l 200–430 µmol/l

Positively birefringent crystals,


Knee aspirate
elevated neutrophils, normal glucose

What is the most likely diagnosis?

A Gout

B Osteoarthritis

C Prepatellar bursitis

D Pseudogout

E Septic arthritis

Explanation 

D Pseudogout

https://mypastest.pastest.com/Secure/TestMe/TimedBrowser/1098274#Top 1/3
8/25/22, 9:39 PM MyPastest

Monoarticular arthritis of acute onset, coupled with positively birefringent crystals in the aspirate, neutrophils
and a normal synovial fluid glucose level, raises the possibility of pseudogout. Although the uric acid level is a
little above the normal range, gout is associated with negatively birefringent crystals. Pseudogout can be
treated acutely with non-steroidal anti-inflammatory agents. Over the longer term, it is often managed with
colchicine.

A Gout

Gout is associated with monoarthritis, although negatively birefringent, rather than positively birefringent,
crystals are seen in this patient.

B Osteoarthritis

Osteoarthritis is not likely to present acutely, affecting only one joint. Some degree of arthritis affecting both
knees would be expected.

C Prepatellar bursitis

Prepatellar bursitis is associated with anterior pain and swelling over the prepatellar bursa, rather than
affecting the knee itself.

E Septic arthritis

Although elevated neutrophils may be seen in patients with septic arthritis, the normal glucose level and the
positively birefringent crystals are a stronger pointer towards pseudogout as the underlying diagnosis.
73209

Rate this question:

Next Question

Previous Question Tag Question

Feedback End Review

Difficulty: Average

Peer Responses %

https://mypastest.pastest.com/Secure/TestMe/TimedBrowser/1098274#Top 2/3
8/25/22, 9:39 PM MyPastest

0:00:00/03:00:00

A 34-year-old man is referred with a skin rash, joint pains and worsening renal function over the past three
months. His blood pressure is 155/90 mmHg, and his heart rate is 75 bpm and regular. There is evidence of
synovitis and a palpable purpuric rash.

Investigation Result Normal value

Haemoglobin (Hb) 113 g/l 115–155 g/l

White cell count (WCC) 7.9 × 10 9/l 4–11 × 10 9/l

Platelets (PLT) 110 × 10 9/l 150–400 × 10 9/l

Sodium (Na +) 143 mmol/l 135–145 mmol/l

Potassium (K +) 4.5 mmol/l 3.5–5.0 mmol/l

Creatinine (Cr) 189 µmol/l 50–120 µmol/l

Rheumatoid factor Positive

Urine Blood 2+, protein 1+

What is the most likely cause of this patient’s symptoms?

A Anti-glomerular basement membrane (GBM) disease

B Chronic lymphocytic leukaemia

C Eosinophilic granulomatosis with polyangiitis

D Hepatitis B

E Hepatitis C

Explanation 

E Hepatitis C

https://mypastest.pastest.com/Secure/TestMe/TimedBrowser/1098274#Top 1/3
8/25/22, 9:39 PM MyPastest

This patient has arthritis, a palpable purpuric rash and evidence of renal vasculitis. The platelets (PLT) count
is also below the lower limit of normal. With a positive rheumatoid factor, these features make mixed
cryoglobulinaemia the most likely diagnosis. Mixed cryoglobulinaemia makes up around 75% of all cases, and
hepatitis C is now recognised as the cause of > 90% of cases of the disease. Measurement of liver function,
clotting and hepatitis C ribonucleic acid (RNA) are obvious next steps in assessing the patient. Treatment of
underlying viral infection can lead to the resolution of cryoglobulinaemia.

A Anti-glomerular basement membrane (GBM) disease

Anti-GBM disease is associated with renal vasculitis, although a positive rheumatoid factor would be
unexpected. The more chronic nature of symptoms and low PLT count also raises the possibility of liver
disease as the underlying cause.

B Chronic lymphocytic leukaemia

Chronic lymphocytic leukaemia is associated with splenomegaly and marked lymphocytosis, neither of which
are seen in this patient.

C Eosinophilic granulomatosis with polyangiitis

Eosinophilic granulomatosis with polyangiitis is associated with poorly controlled symptoms of asthma and
features of vasculitis. There are no symptoms of asthma reported in this patient.

D Hepatitis B

Hepatitis B is very rarely associated with cryoglobulinaemia, the diagnosis here. In contrast, hepatitis C is the
most common cause of mixed cryoglobulinaemia.
73157

Rate this question:

Next Question

Previous Question Tag Question

Feedback End Review

Difficulty: Average

Peer Responses %

https://mypastest.pastest.com/Secure/TestMe/TimedBrowser/1098274#Top 2/3
8/25/22, 9:39 PM MyPastest

0:00:00/03:00:00

A 19-year-old woman who has primary amenorrhoea presents to her General Practitioner for review. She is
concerned as to whether she is developing normally. She is 1.85 m in height and has a body mass index of 22
kg/m 2. Her breast development is normal. She has virtually absent pubic and axillary hair. Her external
genitalia looks normal. Her testosterone level is 6.0 nmol/l (normal values 0.5–2.4 nmol/l).
What is the most likely diagnosis?

A Androgen insensitivity syndrome

B 17-hydroxylase deficiency

C 21-hydroxylase deficiency

D Klinefelter syndrome

E Turner syndrome

Explanation 

A Androgen insensitivity syndrome

This patient is tall, with normal external genitalia, but with virtually absent pubic and axillary hair. In the
presence of primary amenorrhoea and a marked elevation in testosterone level, androgen insensitivity
syndrome is the most likely diagnosis. Karyotyping to confirm an XY karyotype is the most appropriate next
step. Ultrasound of the abdomen and pelvis is likely to reveal bilateral undescended testes, which should be
removed because of malignant potential.

B 17-hydroxylase deficiency

17-hydroxylase deficiency is associated with hypertension in women, but not with virilisation. It is associated
with delayed puberty.

C 21-hydroxylase deficiency

21-hydroxylase deficiency is associated with classical congenital adrenal hyperplasia. This leads to
symptoms of salt wasting in childhood and hyperandrogenism with clitoromegaly.

https://mypastest.pastest.com/Secure/TestMe/TimedBrowser/1098274#Top 1/3
8/25/22, 9:39 PM MyPastest

D Klinefelter syndrome

Klinefelter syndrome is associated with male external genitalia and a low testosterone level compared to the
normal range for men.

E Turner syndrome

Turner syndrome is associated with an XO karyotype. Short stature is a feature of the syndrome and
testosterone level is not elevated.
73134

Rate this question:

Next Question

Previous Question Tag Question

Feedback End Review

Difficulty: Average

Peer Responses %

Q. Answered Flagged

Q1

Q2

Q3

Q4

Q5

https://mypastest.pastest.com/Secure/TestMe/TimedBrowser/1098274#Top 2/3
8/25/22, 9:40 PM MyPastest

0:00:00/03:00:00

A 32-year-old woman presents to the Endocrine Clinic for review, following switching of her antiepileptic
medication from carbamazepine to levetiracetam. A past history of Hashimoto’s thyroiditis is noted, for which
she takes 150 μg of thyroxine daily. She has felt more active since the switch in drug therapy and his General
Practitioner is concerned about her thyroid function results.

Investigation Result Normal value

Thyroid-stimulating hormone (TSH) 0.05 mU/l 0.27–4.2 mU/l

Free thyroxine (T4) 28 pmol/l 12–24 pmol/l

What is the most likely cause of the patient’s thyroid function tests?

A End of carbamazepine P450 induction

B End of carbamazepine P450 inhibition

C Increased gastrointestinal absorption of thyroxine

D Increased conversion of thyroxine to T3

E Levetiracetam-related P450 inhibition

Explanation 

A End of carbamazepine P450 induction

Carbamazepine is a potent P450 enzyme inducer, which increases the metabolism of steroid hormones and
thyroxine. For this reason, in patients taking exogenous T4, their requirements may fall significantly a few
weeks after carbamazepine is discontinued. Levetiracetam is not an inhibitor of any P450 enzymes and is only
a mild inducer of CYP2B6 and 3A4. This accounts for the patient having mild thyrotoxicosis when the
medication is switched.

B End of carbamazepine P450 inhibition

Carbamazepine is an inducer of CYP450, not an inhibitor. This means that it decreases levels of steroid
hormones and T4 while the patient is on therapy.

https://mypastest.pastest.com/Secure/TestMe/TimedBrowser/1098274#Top 1/3
8/25/22, 9:40 PM MyPastest

C Increased gastrointestinal absorption of thyroxine

Carbamazepine does not affect the gastrointestinal absorption of T4. Drugs recognised to affect the
absorption of T4 include antacids, proton pump inhibitors, calcium salts, cimetidine, oral iron, sucralfate,
colestipol, polystyrene sulphonate resin and colestyramine.

D Increased conversion of thyroxine to T3

Carbamazepine has no impact on the conversion of thyroxine to T3. It reduces levels of T4 by driving
increased metabolism of the drug.

E Levetiracetam-related P450 inhibition

Levetiracetam does not inhibit any CYP450 enzymes. It is a mild inducer of CYP2B6 and 3A4 so it is unlikely
to be the cause in this patient.
73120

Rate this question:

Next Question

Previous Question Tag Question

Feedback End Review

Difficulty: Average

Peer Responses %

Q. Answered Flagged

Q1

Q2

https://mypastest.pastest.com/Secure/TestMe/TimedBrowser/1098274#Top 2/3
8/25/22, 9:40 PM MyPastest

0:00:00/03:00:00

A 24-year-old woman who is undergoing induction chemotherapy for acute lymphoblastic leukaemia
presents to her General Practitioner for review. She has become increasingly unwell over the past 12 hours
with nausea, vomiting and dehydration. Her blood pressure is 105/72 mmHg and her heart rate is 95 bpm
and regular. She looks dehydrated.

Investigation Result Normal value

Sodium (Na +) 143 mmol/l 135–145 mmol/l

Potassium (K +) 5.9 mmol/l 3.5–5.0 mmol/l

Creatinine (Cr) 311 µmol/l 50–120 µmol/l

Calcium 2.00 mmol/l 2.1–2.65 mmol/l

Urate 0.78 mmol/l 0.16–0.43 mmol/l

Phosphate 2.1 mmol/l 0.8–1.5 mmol/l

Urine Protein negative, blood 1+

What is the most likely cause of this patient’s symptoms?

A Acute tubular necrosis

B Renal vein thrombosis

C Rhabdomyolysis

D Tumour lysis syndrome

E Urinary sepsis

Explanation 

D Tumour lysis syndrome

https://mypastest.pastest.com/Secure/TestMe/TimedBrowser/1098274#Top 1/3
8/25/22, 9:40 PM MyPastest

This patient is undergoing induction chemotherapy for acute lymphoblastic leukaemia. This induces massive
cell death very quickly. This, in turn, can result in a rapid rise in uric acid levels, leading to acute urate
nephropathy and consequent hyperkalaemia, hyperphosphataemia and hypocalcaemia. For patients at high
risk of tumour lysis syndrome, the use of recombinant urate oxidase as prophylaxis significantly reduces its
incidence.

A Acute tubular necrosis

Acute tubular necrosis can present with hyperphosphataemia, hypocalcaemia and hyperkalaemia. However, it
is the significant increase in the uric acid level here that is a strong pointer towards the diagnosis of tumour
lysis syndrome and acute urate nephropathy.

B Renal vein thrombosis

Renal vein thrombosis leads to acute renal impairment. However, it is associated with a dull ache on the
affected side and a marked rise in urinary protein excretion.

C Rhabdomyolysis

Rhabdomyolysis does lead to the electrolyte imbalance seen here. However, it is not associated with
induction chemotherapy. In addition, marked dipstick-positive haematuria would be expected due to the
presence of urinary myoglobin.

E Urinary sepsis

Although urinary sepsis can lead to an acute deterioration in renal function, the constellation seen here with
hypocalcaemia, hyperuricaemia, hyperkalaemia and hyperphosphataemia is most consistent with tumour lysis
syndrome.
73180

Rate this question:

Next Question

Previous Question Tag Question

Feedback End Review

Difficulty: Average

Peer Responses %

https://mypastest.pastest.com/Secure/TestMe/TimedBrowser/1098274#Top 2/3
8/25/22, 9:40 PM MyPastest

0:00:00/03:00:00

A 45-year-old woman presents to the Emergency Department with severe frontal headache, nausea and
vomiting. She is under investigation by the rheumatologist for sclerodactyly, peripheral calcinosis and
multiple telangiectasias, particularly clustered around the mouth. Around ten days earlier, she was diagnosed
with pulmonary fibrosis and started on high-dose corticosteroids by the Respiratory Team. Her blood
pressure is 179/110 mmHg, and her heart rate is 90 bpm and regular. She has grade III hypertensive
retinopathy on fundoscopy.

Investigation Result Normal value

Haemoglobin (Hb) 109 g/l 115–155 g/l

White cell count (WCC) 8.9 × 10 9/l 4–11 × 10 9/l

Platelets (PLT) 191 × 10 9/l 150–400 × 10 9/l

Sodium (Na +) 144 mmol/l 135–145 mmol/l

Potassium (K +) 4.9 mmol/l 3.5–5.0 mmol/l

Creatinine (Cr) 314 µmol/l 50–120 µmol/l

What is the most appropriate way to treat this patient’s hypertension?

A Amlodipine

B Indapamide

C Metoprolol

D Ramipril

E Sildenafil

Explanation 

D Ramipril

https://mypastest.pastest.com/Secure/TestMe/TimedBrowser/1098274#Top 1/3
8/25/22, 9:40 PM MyPastest

This patient has scleroderma renal crisis which may well have been precipitated by the use of corticosteroids
for pulmonary fibrosis. These can result in a rapid increase in blood pressure over as short as a few days, and
a significant deterioration in renal function. Angiotensin-converting enzyme (ACE) inhibitors are the
cornerstone of the management of blood pressure in this condition. Second-line agents could include a
calcium channel antagonist, such as amlodipine, which may also help relieve any symptoms of Raynaud’s. If
possible, the dose of corticosteroids should also be tapered as soon as is practical.

A Amlodipine

Amlodipine is a second-line agent in the management of scleroderma renal crisis. Angiotensin-converting


enzyme (ACE) inhibitors have been shown to have the most impact on outcomes and are the standard first-
line intervention to control blood pressure and reduce the risk of further renal deterioration.

B Indapamide

Indapamide is a thiazide-like diuretic. It may have a role as a second- or third-line agent in combination with
ramipril and/or a calcium channel antagonist to control hypertension.

C Metoprolol

Beta blockade is not recommended in the treatment of patients with systemic sclerosis because it can worsen
peripheral circulation significantly and exacerbate symptoms of Raynaud’s.

E Sildenafil

Sildenafil is one possible intervention for secondary pulmonary hypertension associated with systemic
sclerosis, the likely underlying diagnosis.
73187

Rate this question:

Next Question

Previous Question Tag Question

Feedback End Review

Difficulty: Average

Peer Responses %

https://mypastest.pastest.com/Secure/TestMe/TimedBrowser/1098274#Top 2/3
8/25/22, 9:44 PM MyPastest

0:00:00/03:00:00

A 59-year-old man who had a left-sided squamous cell lung cancer resected a year ago presents to his
General Practitioner with a skin lesion over his sternum. He has a 1-cm nodule that has grown rapidly and
bleeds easily on minimal contact. Skin metastasis is suspected.

What is the most appropriate intervention?

A Cisplatin-based chemotherapy

B Local radiotherapy

C Photodynamic therapy

D Topical imiquimod

E Topical isotretinoin

Explanation 

B Local radiotherapy

Local radiotherapy is the intervention of choice for this patient. It can lead to rapid resolution of the nodular
tumour before skin breakdown and ulceration occur. Excision is also possible, although often lesions on the
sternum leave little margin for successful excision.

A Cisplatin-based chemotherapy

Cisplatin-based chemotherapy is not very effective in the treatment of squamous cell carcinoma of the
bronchus and local radiotherapy is more appropriate.

C Photodynamic therapy

Unfortunately, photodynamic therapy is not very effective in the treatment of metastatic squamous cell
carcinoma of the bronchus. It is most useful for other types of metastatic skin cancer.

D Topical imiquimod

https://mypastest.pastest.com/Secure/TestMe/TimedBrowser/1098274#Top 1/3
8/25/22, 9:44 PM MyPastest

Topical imiquimod induces cell apoptosis. It is most useful as a treatment for metastatic malignant melanoma
and would not be appropriate for this patient.

E Topical isotretinoin

Topical isotretinoin is best known as a treatment for acne. It is not effective with respect to the clearance of
metastatic squamous cell carcinoma.
73184

Rate this question:

Next Question

Previous Question Tag Question

Feedback End Review

Difficulty: Average

Peer Responses %

Q. Answered Flagged

Q1

Q2

Q3

Q4

Q5

Q6

Q7

https://mypastest.pastest.com/Secure/TestMe/TimedBrowser/1098274#Top 2/3
8/25/22, 9:44 PM MyPastest

0:00:00/03:00:00

A 72-year-old woman is referred to the Haematology Clinic after a loss of appetite, gradual weight loss and
two episodes of herpes zoster infection in the past three months. Her General Practitioner has noted a highly
abnormal white cell count. Abdominal examination reveals massive splenomegaly. Her body mass index is 22
kg/m 2.

Investigation Result Normal value

Haemoglobin (Hb) 90 g/l 115–155 g/l

White cell count (WCC) 747.2 × 10 9/l 4–11 × 10 9/l

Lymphocytes 41.0 × 10 9/l 1–4 × 10 9/l

Platelets (PLT) 110 × 10 9/l 150–400 × 10 9/l

Sodium (Na +) 142 mmol/l 135–145 mmol/l

Potassium (K +) 45.0 mmol/l 3.5–5.0 mmol/l

Creatinine (Cr) 132 µmol/l 50–120 µmol/l

What is the most useful next investigation?

A Abdominal ultrasound scanning

B Immunophenotyping

C Immunoglobulin electrophoresis

D Lymph node biopsy

E TP53 gene testing

Explanation 

B Immunophenotyping

https://mypastest.pastest.com/Secure/TestMe/TimedBrowser/1098274#Top 1/3
8/25/22, 9:44 PM MyPastest

Marked elevation in the white cell count with a lymphocytic predominance is consistent with chronic
lymphocytic leukaemia, in which impaired viral immunity is common (leading to the recurrent herpes zoster
infection seen here) and there is massive splenomegaly. Immunophenotyping is the usual next investigation,
which confirms the presence of clonal B lymphocytes expressing CD5, CD19, CD20 and CD23. Typically,
TP53 gene testing is done because TP53 deletion implies the removal of a tumour suppressor gene, which
signifies a poor prognosis.

A Abdominal ultrasound scanning

Abdominal ultrasound scanning will merely confirm the presence of splenomegaly. It will not help confirm the
diagnosis of chronic lymphocytic leukaemia.

C Immunoglobulin electrophoresis

Immunoglobulin electrophoresis is not useful in establishing the diagnosis but may be considered in patients
who suffer recurrent viral infections.

D Lymph node biopsy

Lymph node biopsy is indicated in chronic lymphocytic leukaemia for patients who have rapidly enlarging
peripheral lymph nodes, which implies progression to aggressive lymphoma.

E TP53 gene testing

It would be a poor prognosis in underlying chronic lymphocytic leukaemia as TP53 gene testing is useful
because deletion of TP53 implies removal of an important tumour suppressor gene.
73159

Rate this question:

Next Question

Previous Question Tag Question

Feedback End Review

Difficulty: Average

Peer Responses %

https://mypastest.pastest.com/Secure/TestMe/TimedBrowser/1098274#Top 2/3
8/25/22, 9:44 PM MyPastest

0:00:00/03:00:00

A 40-year-old man who works on a building site presents to the Emergency Department with severe right-
sided pleuritic chest pain and shortness of breath. He smokes 30 cigarettes per day. His oxygen saturation is
91% on 60% inhaled oxygen. He has no breath sounds on auscultation of the right-hand side of the chest.

A chest X-ray is taken:


What is the most appropriate next step?

A Admit for 24-hour administration of oxygen

B Air aspiration, then admit for 24-hour administration of oxygen

C Chest drain insertion

D Observe for four hours, then discharge

E Repeat air aspiration

Explanation 

C Chest drain insertion

https://mypastest.pastest.com/Secure/TestMe/TimedBrowser/1098274#Top 1/3
8/25/22, 9:44 PM MyPastest

Given the size of this patient’s right-sided pneumothorax (almost complete collapse of the right lung) and the
fact that he is a heavy smoker, insertion of a formal chest drain is the only appropriate way to manage this
patient. Air aspiration is highly unlikely to be successful. The chest drain would normally stop bubbling after
48 hours in position. After this time, continued bubbling would be considered as a persistent air leak.
Image credit: CC BY-NC-SA 2.0

A Admit for 24-hour administration of oxygen

This patient has a very large right-sided pneumothorax, with air almost completely replacing the right lung
field. A chest drain is the only appropriate intervention.

B Air aspiration, then admit for 24-hour administration of oxygen

Given the size of this patient’s pneumothorax, air aspiration is highly unlikely to be effective. He is also a
heavy smoker, which means a secondary pneumothorax in the presence of chest disease cannot be
completely excluded.

D Observe for four hours, then discharge

This patient has a large pneumothorax, and observing him for four hours is not appropriate. Insertion of a
chest drain is the most appropriate next step.

E Repeat air aspiration

Air aspiration should not be attempted more than once, after which it should be followed by insertion of a
chest drain. It is unlikely to be successful here due to the size of the pneumothorax.
73203

Rate this question:

Next Question

Previous Question Tag Question

Feedback End Review

Difficulty: Average

Peer Responses %

https://mypastest.pastest.com/Secure/TestMe/TimedBrowser/1098274#Top 2/3
8/25/22, 9:45 PM MyPastest

0:00:00/03:00:00

A 43-year-old bar owner is referred to the Endocrine Clinic with tiredness and erectile dysfunction. He also
has problems with swelling and stiffness affecting his right knee. His blood pressure is 122/86 mmHg and his
heart rate is 67 bpm. He looks tanned and has spider naevi over his upper body. His body mass index is 27
kg/m 2.

Investigation Result Normal value

Haemoglobin (Hb) 132 g/l 115–155 g/l

White cell count (WCC) 7.1 × 10 9/l 4.0–11.0 × 10 9/l

Platelets (PLT) 75 × 10 9/l 150–400 × 10 9/l

Sodium (Na +) 142 mmol/l 135–145 mmol/l

Potassium (K +) 4.1 mmol/l 3.5–5.0 mmol/l

Creatinine (Cr) 95 µmol/l 50–120 µmol/l

Alanine aminotransferase (ALT) 95 U/l 1–40 U/l

Alkaline phosphatase 245 U/l 30–130 U/l

Fasting glucose 6.3 mmol/l < 7 mmol/l

What is the most likely underlying cause of this patient’s symptoms?

A Addison’s disease

B Alcoholic cirrhosis

C Cushing syndrome

D Haemochromatosis

E Wilson’s disease

Explanation 

D Haemochromatosis

https://mypastest.pastest.com/Secure/TestMe/TimedBrowser/1098274#Top 1/3
8/25/22, 9:45 PM MyPastest

Haemochromatosis, also known as bronze diabetes, is the most likely diagnosis here, with increased skin
pigmentation, likely pseudogout, abnormal liver function tests and impaired glucose tolerance, all features of
the disease. It is caused by abnormal accumulation of iron and at least two gene mutations which predispose
to the condition – C282Y and H63D have been identified. Venesection is the intervention of choice to manage
the condition.

A Addison’s disease

Although Addison’s disease is associated with increased skin pigmentation, other features such as
hyponatraemia, hyperkalaemia and weight loss, as well as hypoglycaemia, are absent here.

B Alcoholic cirrhosis

Alcoholic cirrhosis may explain some of this patient’s abnormal liver function, especially as he is a bar owner.
The other features, including skin pigmentation and likely pseudogout, are, however, stronger pointers
towards haemochromatosis.

C Cushing syndrome

Cushing syndrome may be associated with abnormal liver function tests and abnormal glucose tolerance.
However, the marked weight gain seen in Cushing syndrome is absent in this patient.

E Wilson’s disease

Although Wilson’s disease is associated with abnormal liver function tests, it presents more commonly with
psychiatric disturbance and movement disorder, rather than the features here which are typical of
haemochromatosis.
73208

Rate this question:

Next Question

Previous Question Tag Question

Feedback End Review

Difficulty: Average

Peer Responses %

https://mypastest.pastest.com/Secure/TestMe/TimedBrowser/1098274#Top 2/3
8/25/22, 9:52 PM MyPastest

0:00:00/03:00:00

A 57-year-old woman presents to her General Practitioner with increasing drowsiness and ataxia over the
past few days. She has also had nausea and vomiting. She recently split from her partner. She has epilepsy,
type II diabetes and chronic renal failure. Medications include phenytoin, ramipril, amlodipine and sitagliptin.
She is drowsy and has obvious nystagmus on examination. She finds it difficult to follow your commands.
Investigation:

Investigation Result Normal value

Haemoglobin (Hb) 105 g/l 115–155 g/l

White cell count (WCC) 7.9 × 10 9/l 4–11 × 10 9/l

Platelets (PLT) 182 × 10 9/l 150–400 × 10 9/l

Sodium (Na +) 143 mmol/l 135–145 mmol/l

Potassium (K +) 4.9 mmol/l 3.5–5.0 mmol/l

Bicarbonate 18 mmol/l 22–29 mmol/l

Creatinine (Cr) 120 µmol/l 50–120 µmol/l

Phenytoin 48 µg/ml 10–20 µg/ml

What is the most appropriate next step?

A Activated charcoal

B Calcium chloride

C Diazepam

D Haemodialysis

E Sodium bicarbonate

Explanation 

A Activated charcoal

https://mypastest.pastest.com/Secure/TestMe/TimedBrowser/1098274#Top 1/3
8/25/22, 9:52 PM MyPastest

Repeat gastrointestinal decontamination with activated charcoal is the intervention of choice for phenytoin
overdose, the most likely cause of this patient’s symptoms. Although the patient also has renal failure, which
raises the proportion of unbound phenytoin, the level of phenytoin measured suggests an intentional
overdose after the split from her partner. Other measures such as haemodialysis are thought to be ineffective.
Where there are phenytoin-related seizures, benzodiazepines are the intervention of choice.

B Calcium chloride

Calcium chloride is not a first-line intervention in the treatment of phenytoin overdose. It is used to treat the
symptoms of hypocalcemia, arrhythmias and hypermagnesemia. In this case, activated charcoal is the most
appropriate next step.

C Diazepam

Benzodiazepines are the intervention of choice for seizures associated with phenytoin toxicity. They may, of
course, lead to further respiratory depression necessitating ventilatory support.

D Haemodialysis

Haemodialysis has traditionally been thought to be ineffective in the treatment of phenytoin overdose
because the drug is highly protein-bound.

E Sodium bicarbonate

Although acidosis can increase the proportion of unbound phenytoin and worsen toxicity, the bicarbonate
suggests a relatively mild acidosis at only 18 mmol/l. Sodium bicarbonate can be considered when there is
QRS widening and ventricular arrhythmias.
73119

Rate this question:

Next Question

Previous Question Tag Question

Feedback End Review

Difficulty: Average

Peer Responses %

https://mypastest.pastest.com/Secure/TestMe/TimedBrowser/1098274#Top 2/3
8/25/22, 9:52 PM MyPastest

0:00:00/03:00:00

A 59-year-old man is currently undergoing investigation for possible small cell lung cancer. Over the past
few weeks, he has also begun to complain of proximal muscle weakness, particularly affecting his lower
limbs. He also has symptoms of presyncope. Examination reveals proximal muscle weakness, and he has a
waddling gait and difficulty with raising his arms.
What is the most useful investigation?

A Anti-acetylcholine receptor antibodies

B Anti-mitochondrial antibodies

C Anti-smooth muscle antibodies

D Anti-voltage-gated calcium channel antibodies

E Anti-Yo antibodies

Explanation 

D Anti-voltage-gated calcium channel antibodies

This patient has Lambert–Eaton myasthenic syndrome, which is strongly associated with small cell carcinoma
of the bronchus, and anti-voltage-gated calcium channel antibodies are found in over 90% of patients with
the condition. The mainstay of therapy is 3,4-diaminopyridine, which significantly improves muscle strength.
Symptoms also improve once treatment for the underlying cancer is initiated. Immunosuppression may also
be of value in some patients.

A Anti-acetylcholine receptor antibodies

Anti-acetylcholine receptor antibodies are associated with myasthenia gravis. The presence of an underlying
bronchial carcinoma suggests that Lambert–Eaton syndrome is actually the diagnosis.

B Anti-mitochondrial antibodies

Anti-mitochondrial antibodies are primarily associated with primary biliary cirrhosis, where it is found in over
90% of patients.

https://mypastest.pastest.com/Secure/TestMe/TimedBrowser/1098274#Top 1/3
8/25/22, 9:52 PM MyPastest

C Anti-smooth muscle antibodies

Anti-smooth muscle antibodies are not associated with Lambert–Eaton syndrome. They are, in fact, most
strongly associated with autoimmune hepatitis.

E Anti-Yo antibodies

Anti-Yo antibodies are associated with paraneoplastic cerebellar degeneration, which is not suggested by the
symptoms reported by this patient.
73174

Rate this question:

Next Question

Previous Question Tag Question

Feedback End Review

Difficulty: Average

Peer Responses %

Q. Answered Flagged

Q1

Q2

Q3

Q4

Q5

https://mypastest.pastest.com/Secure/TestMe/TimedBrowser/1098274#Top 2/3
8/25/22, 9:52 PM MyPastest

0:00:00/03:00:00

A 19-year-old student presents to the Emergency Department with frank haematuria over the past 12 hours.
He has a respiratory tract infection, which began three days ago. His blood pressure is 115/82 mmHg, and his
heart rate is 75 bpm and regular. His abdomen is soft and non-tender.

Investigation Result Normal value

Haemoglobin (Hb) 82 g/l 115–155 g/l

White cell count (WCC) 6.1 × 10 9/l 4–11 × 10 9/l

Platelets (PLT) 190 × 10 9/l 150–400 × 10 9/l

Sodium (Na +) 142 mmol/l 135–145 mmol/l

Potassium (K +) 4.8 mmol/l 3.5–5.0 mmol/l

Creatinine (Cr) 95 µmol/l 50–120 µmol/l

Urine Blood 3+, protein+

What is the most likely diagnosis?

A Granulomatosis with polyangiitis

B Immunoglobulin A (IgA) nephropathy

C Membranous nephropathy

D Minimal change disease

E Post-streptococcal glomerulonephritis

Explanation 

B Immunoglobulin A (IgA) nephropathy

Immunoglobulin A (IgA) nephropathy is the most likely diagnosis of this patient, with frank haematuria
occurring in association with, or shortly after, a respiratory tract infection. It is characterised by increased
circulating levels of IgA with galactose-deficient hinge-region O-glycans. Antibodies form against this

https://mypastest.pastest.com/Secure/TestMe/TimedBrowser/1098274#Top 1/3
8/25/22, 9:52 PM MyPastest

abnormal IgA, leading to immune complex formation. Control of blood pressure with the use of angiotensin-
converting enzyme inhibitors in patients who experience hypertension is the cornerstone of management. The
role of corticosteroids in reducing progression to end-stage renal failure is controversial because of their
benefit-risk profile. A modified-release form of budesonide has recently become available, which appears to
reduce proteinuria with an acceptable side-effect profile.

A Granulomatosis with polyangiitis

Granulomatosis with polyangiitis is associated with sinusitis and pulmonary and renal vasculitis, resulting in
episodes of haemoptysis and haematuria.

C Membranous nephropathy

Membranous nephropathy often has an insidious onset, presenting with nephrotic range proteinuria. It may be
idiopathic; it is also seen in association with underlying lung cancer.

D Minimal change disease

Minimal change disease is associated with marked proteinuria and tends to occur in children and young
adults. It responds well to corticosteroids.

E Post-streptococcal glomerulonephritis

Post-streptococcal glomerulonephritis is associated with haematuria. However, it tends to present 1–2 weeks
after a streptococcal throat infection, rather than concomitantly or very shortly after a respiratory tract
infection, as seen here.
73196

Rate this question:

Next Question

Previous Question Tag Question

Feedback End Review

Difficulty: Average

Peer Responses %

https://mypastest.pastest.com/Secure/TestMe/TimedBrowser/1098274#Top 2/3
8/25/22, 9:52 PM MyPastest

0:00:00/03:00:00

A 23-year-old woman presents to her General Practitioner with complaints of hirsutism and acne. She also
has irregular heavy periods which occur every 2–3 months. Her blood pressure is 145/85 mmHg, and her
heart rate is 75 bpm and regular. Her body mass index is 29 kg/m 2. She has facial and upper chest acne and
increased hair over her moustache and beard line.
Investigations:

Investigation Result Normal value

Haemoglobin (Hb) 132 g/l 115–155 g/l

White cell count (WCC) 8.3 × 10 9/l 4–11 × 10 9/l

Platelets (PLT) 191 × 10 9/l 150–400 × 10 9/l

Sodium (Na +) 142 mmol/l 135–145 mmol/l

Potassium (K +) 3.5 mmol/l 3.5–5.0 mmol/l

Creatinine (Cr) 85 µmol/l 50–120 µmol/l

Testosterone 3.2 nmol/l 0.2–1.7 nmol/l

Luteinising hormone
2.3 < 2.0
(LH) : follicle-stimulating hormone (FSH) ratio

Prolactin 720 mIU/l < 500 mIU/l

Cortisol after low-dose dexamethasone 65 nmol/l < 50 nmol/l

What is the most likely diagnosis?

A Androgen-secreting adrenal tumour

B Cushing’s disease

C Polycystic ovarian syndrome

D Prolactinoma

E 21-hydroxylase deficiency

https://mypastest.pastest.com/Secure/TestMe/TimedBrowser/1098274#Top 1/4
8/25/22, 9:52 PM MyPastest

Explanation 

C Polycystic ovarian syndrome

This patient is overweight, with an elevated testosterone level and a luteinising hormone (LH) : follicle-
stimulating hormone (FSH) ratio of above 2.0. She has irregular periods, acne and hirsutism. Taken together,
these symptoms are consistent with a diagnosis of polycystic ovarian syndrome. The prolactin level is only
mildly elevated, as is the cortisol post-dexamethasone suppression test, raising the possibility that these
results are related to stress. Weight loss can improve symptoms of hyperandrogenism and fertility. Other
options include electrolysis for excess facial hair and the contraceptive pill to regularise the menstrual cycle.

A Androgen-secreting adrenal tumour

An androgen-secreting adrenal tumour can lead to symptoms of virilisation, although a much greater rise in
testosterone level than that seen here would be expected.

B Cushing’s disease

Cushing’s disease occurs because of an underlying adrenocorticotropic hormone (ACTH)-producing pituitary


tumour. In this case, other features such as weight gain, more marked hypertension and hyperglycaemia
would be expected.

D Prolactinoma

There is only a very slight elevation in prolactin levels, which is consistent with another explanation for the
rise seen here such as stress.

E 21-hydroxylase deficiency

A deficiency in 21-hydroxylase is behind congenital adrenal hyperplasia. This is associated with virilisation,
potentially including clitoromegaly. A more marked elevation in testosterone level would be expected versus
that seen here.
73140

Rate this question:

Next Question

Previous Question Tag Question

Feedback End Review

https://mypastest.pastest.com/Secure/TestMe/TimedBrowser/1098274#Top 2/4
8/25/22, 9:52 PM MyPastest

0:00:00/03:00:00

A 33-year-old woman is admitted to the Emergency Department overnight for intravenous fluids due to
severe dehydration. She is suspected of having a Campylobacter infection.

Which of the following is the most important complication of Campylobacter infection?

A Ascending colangitis

B Guillain–Barré syndrome

C Membranous glomerulonephritis

D Myasthenia gravis

E Rheumatoid arthritis

Explanation 

B Guillain–Barré syndrome

Guillain–Barré syndrome is associated with both Campylobacter gastroenteritis and sexually transmitted
infection. It presents with ascending flaccid weakness and decreased reflexes and may also be associated
with neuropathic pain. Respiratory failure is of major concern and patients should be monitored for
deterioration in forced vital capacity which may require ventilation on the Intensive Therapy Unit. Treatment is
with immunoglobulin infusion or plasma exchange.

A Ascending colangitis

Recurrent cholecystitis, rather than ascending cholangitis, may be associated with Campylobacter infection.

C Membranous glomerulonephritis

Campylobacter may lead to the development of haemolytic uraemic syndrome, rather than membranous
glomerulonephritis.

D Myasthenia gravis

https://mypastest.pastest.com/Secure/TestMe/TimedBrowser/1098274#Top 1/3
8/25/22, 9:52 PM MyPastest

Guillain–Barré syndrome, not myasthenia gravis, is associated with Campylobacter. Myasthenia gravis is
associated with progressive weakness, which usually begins centrally.

E Rheumatoid arthritis

Campylobacter is well recognised as a trigger for reactive arthritis, not for rheumatoid arthritis. The other
well-described trigger for reactive arthritis is sexually transmitted infection.
73148

Rate this question:

Next Question

Previous Question Tag Question

Feedback End Review

Difficulty: Average

Peer Responses %

Q. Answered Flagged

Q1

Q2

Q3

Q4

Q5

Q6

Q7

https://mypastest.pastest.com/Secure/TestMe/TimedBrowser/1098274#Top 2/3
8/25/22, 9:53 PM MyPastest

0:00:00/03:00:00

A 59-year-old man presents to the Emergency Department due to a left leg deep vein thrombosis (DVT) after
a colectomy for a Dukes’ B colon cancer some three weeks earlier. Liver function is normal.

What is the most appropriate long-term treatment for this patient’s DVT?

A Apixaban

B Dalteparin

C Lepirudin

D Rivaroxaban

E Warfarin

Explanation 

A Apixaban

Oral direct anticoagulants, such as apixaban, have proven to reduce the risk of venous thromboembolism in
patients with malignancy when compared to low-molecular-weight heparin. Rates of bleeding are slightly
higher with direct anticoagulants than with low-molecular-weight heparin. However, efficacy is significantly
higher with direct anticoagulants and the bleeding risk appears highest for those with upper gastrointestinal
cancers.

B Dalteparin

Dalteparin is a low-molecular-weight heparin. It is slightly less effective, compared to direct oral


anticoagulants, but may still be used as first-line prophylaxis in patients with upper gastrointestinal cancers.

C Lepirudin

Lepirudin is a direct thrombin inhibitor. It is less well studied in prophylaxis against thrombosis in malignancy,
compared to low-molecular-weight heparin or agents such as apixaban.

D Rivaroxaban
https://mypastest.pastest.com/Secure/TestMe/TimedBrowser/1098274#Top 1/3
8/25/22, 9:53 PM MyPastest

Although rivaroxaban is effective in preventing thrombosis in patients with underlying malignancy, across
clinical trials for this indication, it had a larger increase in secondary haemorrhage, compared to apixaban.

E Warfarin

Warfarin is considered a second-line option for thromboembolism prophylaxis in cancer in the event that low-
molecular-weight heparin or direct anticoagulants are considered unsuitable or contraindicated.
73160

Rate this question:

Next Question

Previous Question Tag Question

Feedback End Review

Difficulty: Average

Peer Responses %

Q. Answered Flagged

Q1

Q2

Q3

Q4

Q5

Q6

https://mypastest.pastest.com/Secure/TestMe/TimedBrowser/1098274#Top 2/3
8/25/22, 9:53 PM MyPastest

0:00:00/03:00:00

A 34-year-old man who has ulcerative colitis presents to his General Practitioner, complaining of tiredness
and itching which have progressively worsened over the past 4–5 months. His bowel symptoms are stable; he
takes regular mesalamine and opens his bowels three times per day. There are scratch marks over the upper
body. His body mass index is 22 kg/m 2.

Investigation Result Normal value

Haemoglobin (Hb) 120 g/l 115–155 g/l

White cell count (WCC) 8.1 × 10 9/l 4–11 × 10 9/l

Platelets (PLT) 84× 10 9/l 150–400 × 10 9/l

Sodium (Na +) 140 mmol/l 135–145 mmol/l

Potassium (K +) 3.9 mmol/l 3.5–5.0 mmol/l

Creatinine (Cr) 90 µmol/l 50–120 µmol/l

Alanine transaminase (ALT) 95 U/l 1–40 U/l

Alkaline phosphatase 355 U/l 30–130 U/l

Bilirubin 18 µmol/l 5–17 µmol/l

Perinuclear anti-neutrophil cytoplasmic


antibodies (p-ANCA) and Positive
antinuclear antibodies (ANA)

What is the most likely diagnosis?

A Autoimmune hepatitis

B Cholangiocarcinoma

C Mesalamine-related hepatotoxicity

D Primary biliary cholangitis

E Primary sclerosing cholangitis

https://mypastest.pastest.com/Secure/TestMe/TimedBrowser/1098274#Top 1/4
8/25/22, 9:53 PM MyPastest

Explanation 

E Primary sclerosing cholangitis

Over 90% of patients with primary sclerosing cholangitis have underlying inflammatory bowel disease and
present with slowly progressive obstructive liver dysfunction because of intra- and extra-hepatic bile duct
stricture formation. Both positive perinuclear anti-neutrophil cytoplasmic antibodies and antinuclear
antibodies may be seen. Magnetic resonance cholangiopancreatography is the investigation of choice and
may be followed by a liver biopsy to help stage the degree of liver fibrosis. Ursodeoxycholic acid may be of
value when instigated early.

A Autoimmune hepatitis

Autoimmune hepatitis presents with hepatocellular dysfunction and is associated with anti-smooth muscle
antibodies.

B Cholangiocarcinoma

Cholangiocarcinoma presents with rapidly worsening obstructive liver function tests. It can occur after long-
term primary sclerosing cholangitis.

C Mesalamine-related hepatotoxicity

It is unusual for mesalamine-related hepatotoxicity to develop a long time after starting therapy. It presents
most often with an obstructive picture on liver function testing 2–3 weeks after treatment is commenced.

D Primary biliary cholangitis

Primary biliary cholangitis is not associated with inflammatory bowel disease and is not associated with p-
ANCA. It is associated with anti-mitochondrial antibodies. It does present with obstructive liver function tests.
73151

Rate this question:

Next Question

Previous Question Tag Question

Feedback End Review

Difficulty: Average

https://mypastest.pastest.com/Secure/TestMe/TimedBrowser/1098274#Top 2/4
8/25/22, 9:56 PM MyPastest

0:00:00/03:00:00

A 64-year-old man with chronic obstructive pulmonary disease (COPD) is reviewed in the Respiratory Clinic
after a third exacerbation in the past year. He has stopped smoking five years earlier. When he was first
diagnosed, he showed some reversibility after a trial of oral prednisolone. He is currently managed with triple
inhaled therapy but is still markedly short of breath and finds it difficult to walk 200 m to the bus stop. His
blood pressure is 123/82 mmHg, and his heart rate is 67 bpm and regular. His chest is hyperexpanded and he
has a quiet wheeze on auscultation.

Investigations:

Investigation Result Normal value

Haemoglobin (Hb) 139 g/l 115–155 g/l

White cell count (WCC) 7.3 × 10 9/l 4–11 × 10 9/l

Eosinophils 0.6 ×10 9/l 0.02–0.5 ×10 9/l

Platelets (PLT) 192 × 10 9/l 150–400 × 10 9/l

Sodium (Na +) 142 mmol/l 135–145 mmol/l

Potassium (K +) 3.7 mmol/l 3.5–5.0 mmol/l

Creatinine (Cr) 95 µmol/l 50–120 µmol/l

Forced expiratory volume in one second (FEV1) 50% of predicted

Forced vital capacity (FVC) 82% of predicted

What is the most useful next intervention?

A Mepolizumab

B Montelukast

C Prednisolone

D Roflumilast

E Theophylline

https://mypastest.pastest.com/Secure/TestMe/TimedBrowser/1098274#Top 1/4
8/25/22, 9:56 PM MyPastest

Explanation 

A Mepolizumab

Mepolizumab is an anti-interleukin 5 (IL5) monoclonal antibody that depletes eosinophils. This patient’s
chronic obstructive pulmonary disease (COPD) is a special case that is expected to benefit from eosinophil
depletion, given that he has an elevated eosinophil count and a reversible component to his airway
obstruction. The mepolizumab phase 3 study showed that in patients with elevated eosinophil counts,
mepolizumab is associated with an 18% reduction in moderate or severe COPD exacerbations, compared to
no significant impact across the patient population as a whole.

B Montelukast

Montelukast is a leukotriene receptor antagonist. It is most useful in the management of uncontrolled asthma.
It does not have a role in the treatment of COPD.

C Prednisolone

Oral steroids are not preferred in the long-term management of COPD because of the negative impact on
weight, glucose tolerance and bone mineral density.

D Roflumilast

Roflumilast is a phosphodiesterase type 4 (PDE4) inhibitor, useful in patients with COPD that is not
controlled on triple inhaled therapy. However, here an anti-eosinophil agent is preferred, given the
reversibility and elevated eosinophil count.

E Theophylline

Theophylline tablets have limited impact on patients with COPD who are already treated with adequate
beta-agonist therapy. They also increase the risk of atrial fibrillation.
73197

Rate this question:

Next Question

Previous Question Tag Question

Feedback End Review

Difficulty: Average

https://mypastest.pastest.com/Secure/TestMe/TimedBrowser/1098274#Top 2/4
8/25/22, 9:56 PM MyPastest

0:00:00/03:00:00

A 19-year-old man presents to the Dermatology Clinic, complaining of multiple macular lesions which have
developed over his upper body in the past 1–2 weeks. The lesions are mildly erythematous and have fine
superficial skin scaling over their surface.

Investigations:


What is the most likely diagnosis?

A Erythrasma

B Melasma

C Pityriasis rosea

D Pityriasis versicolor

https://mypastest.pastest.com/Secure/TestMe/TimedBrowser/1098274#Top 1/4
8/25/22, 9:56 PM MyPastest

E Tinea corporis

Explanation 

D Pityriasis versicolor

Pityriasis versicolor is caused by proliferation of the Malassezia fungus, which is part of normal skin flora. It is
seen more often in hot and humid climates and during summer. The lesions fluoresce under Wood’s lamp, as
seen on the patient's Wood's lamp. It is treated with topical imidazoles, either as a cream or as a shampoo
preparation.

Image credit: By Warfieldian - CC BY-SA 3.0, https://commons.wikimedia.org/w/index.php?curid=28382856.

A Erythrasma

Erythrasma is caused by Corynebacterium infection which occurs in skinfolds. It fluoresces coral pink in colour
under Wood’s lamp.

B Melasma

Melasma is increased skin pigmentation which is commonly seen in association with pregnancy. It actually
absorbs more of Wood’s light and appears darker.

C Pityriasis rosea

Pityriasis rosea occurs as a salmon pink or red larger herald patch, which is followed by a generalised
erythematous rash a few days later. It may be triggered by a viral infection, vaccination or drugs, including
angiotensin-converting enzyme inhibitors.

E Tinea corporis

Tinea corporis usually appears as a single patch of affected skin, rather than as multiple areas as seen here,
although it does fluoresce under Wood’s lamp.
73130

Rate this question:

Next Question

Previous Question Tag Question

Feedback
https://mypastest.pastest.com/Secure/TestMe/TimedBrowser/1098274#Top 2/4
8/25/22, 9:56 PM MyPastest

0:00:00/03:00:00

A 19-year-old student is reviewed on the Acute Medical Ward following a direct current (DC) cardioversion.
He collapsed in a nightclub and on arrival in the Emergency Department, he had a narrow complex
tachycardia, with a heart rate of 180 bpm. His blood pressure was 90/50 mmHg, and it is now 120/80 mmHg.
His heart rate is 72 bpm.
His current electrocardiogram (ECG) is shown below:


What is the most likely cause of this patient’s tachycardia?

A Brugada syndrome

B Hypertrophic obstructive cardiomyopathy

C Long QT syndrome

D Lown–Ganong–Levine syndrome

E Wolff–Parkinson–White syndrome

Explanation 

E Wolff–Parkinson–White syndrome

https://mypastest.pastest.com/Secure/TestMe/TimedBrowser/1098274#Top 1/3
8/25/22, 9:56 PM MyPastest

This patient’s resting electrocardiogram (ECG) shows a short PR interval (< 120 ms), and a slurred upstroke
to the QRS consistent with a delta wave. The delta wave is negative in the inferior leads, which wrongly
raises the possibility of an inferior myocardial infarction. The episode of supraventricular tachycardia reported
here is typical of the episodes seen in Wolff–Parkinson–White syndrome, the underlying diagnosis. To avoid
long-term antiarrhythmic therapy, patients are increasingly referred for electrophysiological studies and
radiofrequency ablation, which carries a 95% success rate. Flecainide and sotalol are both used for long-term
drug therapy to prevent episodes of arrhythmia.

Image credit: By Michael Rosengarten BEng, MD.McGill, Michael Rosengarten BEng, MD.McGill, CC BY-SA
3.0

A Brugada syndrome

Brugada syndrome is associated with a right bundle branch block pattern on this patient's ECG, and sudden
death with arrhythmias and cardiac arrest often occurs during sleep.

B Hypertrophic obstructive cardiomyopathy

Hypertrophic obstructive cardiomyopathy leads to the features of left ventricular hypertrophy on this patient's
ECG, rather than the short PR interval and delta waves seen in this patient.

C Long QT syndrome

The QT interval is not prolonged here, and the PR interval is markedly reduced. With the delta wave seen on
this patient's ECG, the clinical picture is consistent with Wolff–Parkinson–White syndrome.

D Lown–Ganong–Levine syndrome

Lown–Ganong–Levine syndrome is associated with episodes of supraventricular tachycardia and a short PR


interval, although the QRS complex is normal.
73114

Rate this question:

Next Question

Previous Question Tag Question

Feedback End Review

Difficulty: Average

Peer Responses %

https://mypastest.pastest.com/Secure/TestMe/TimedBrowser/1098274#Top 2/3
8/25/22, 9:56 PM MyPastest

0:00:00/03:00:00

A 58-year-old man presents to the Oncology Clinic for review. He has been diagnosed with a stage IIB
squamous cell carcinoma of the bronchus, with limited involvement of the chest wall.

What is the most appropriate next step?

A Conventional radiotherapy and chemotherapy

B Immunotherapy

C Palliative care

D Stereotactic radiotherapy

E Surgery with chest wall resection

Explanation 

E Surgery with chest wall resection

Although this patient has chest wall involvement, he has stage IIB disease, meaning that resection and cure
are still a possibility. Although surgery is challenging, it can result in a potential 40% survival rate at five
years, although this falls to 12% in the event lymph node involvement is detected at surgery (this is stage IIIA
disease). Progression to surgery is contingent upon adequate lung function assessed preoperatively.

A Conventional radiotherapy and chemotherapy

Conventional chemotherapy and radiotherapy should be offered to patients with non-small cell lung cancer
who have an irresectable tumour. Although the chest wall is involved, surgery is still an option here.

B Immunotherapy

Checkpoint inhibitors are currently a potential option for patients with metastatic non-small cell lung cancer.
In this patient, surgical excision should still be explored as a potential option.

C Palliative care

https://mypastest.pastest.com/Secure/TestMe/TimedBrowser/1098274#Top 1/3
8/25/22, 9:56 PM MyPastest

Palliative care alone is not appropriate for this patient, given there is potential for surgical resection.

D Stereotactic radiotherapy

Stereotactic radiotherapy is indicated for patients with stage 1 non-small cell lung cancer in whom surgery is
inappropriate. This patient still potentially has surgical resection as an option.
73183

Rate this question:

Next Question

Previous Question Tag Question

Feedback End Review

Difficulty: Average

Peer Responses %

Q. Answered Flagged

Q1

Q2

Q3

Q4

Q5

Q6

Q7

https://mypastest.pastest.com/Secure/TestMe/TimedBrowser/1098274#Top 2/3
8/25/22, 9:57 PM MyPastest

0:00:00/03:00:00

A 63-year-old man presents to his General Practitioner complaining of indigestion. He says this is particularly
bad when he lies flat and he occasionally gets food and acid coming up into his mouth. He has not lost any
weight. His abdomen is soft and non-tender, and his body mass index is 25 kg/m 2.

A chest X-ray is performed:


What is the most likely diagnosis?

A Achalasia

B Dilated cardiomyopathy

C Hiatus hernia

D Oesophageal carcinoma

E Phrenic nerve palsy

Explanation 

https://mypastest.pastest.com/Secure/TestMe/TimedBrowser/1098274#Top 1/3
8/25/22, 9:57 PM MyPastest

C Hiatus hernia

This patient’s symptoms are consistent with a hiatus hernia, and the chest X-ray reveals abdominal contents
that have herniated into the chest cavity through the oesophageal hiatus. Given this patient’s level of
symptoms, laparoscopic fundoplication is likely to be required to deal with symptoms.

Image credit: By James Heilman, MD - CC BY-SA 4.0,


https://commons.wikimedia.org/wiki/File:Hiatus_Hernia_on_X-Ray.jpg

A Achalasia

Achalasia is associated with oesophageal dilatation and not the presence of abdominal viscera within the
chest cavity. For achalasia, intervention with oesophageal dilatation or botulinum toxin may be considered.

B Dilated cardiomyopathy

Dilated cardiomyopathy is associated with cardiac enlargement. It does not fit with the double shadow seen
here, which is more consistent with a hiatus hernia.

D Oesophageal carcinoma

The chest X-ray shows a large shadow due to the presence of abdominal contents within the chest cavity.
This is consistent with a hiatus hernia, rather than with an oesophageal carcinoma.

E Phrenic nerve palsy

Phrenic nerve palsy leads to elevation of the hemidiaphragm on the affected side. The clinical picture in this
patient, with the appearance of a double shadow, is more consistent with a hiatus hernia.
73143

Rate this question:

Next Question

Previous Question Tag Question

Feedback End Review

Difficulty: Average

Peer Responses %

https://mypastest.pastest.com/Secure/TestMe/TimedBrowser/1098274#Top 2/3
8/25/22, 9:57 PM MyPastest

0:00:00/03:00:00

A 78-year-old man is admitted to the Emergency Department with severe headache, neck stiffness and
confusion which has worsened significantly over the past 12 hours. He has just returned from a holiday to
Northern France. His temperature is 38.2 °C, blood pressure 110/70 mmHg and heart rate 92 bpm and
regular. His Glasgow Coma Scale (GCS) score is 13. There is marked neck stiffness and photophobia.

Investigation Result Normal value

Haemoglobin (Hb) 142 g/l 115–155 g/l

White cell count (WCC) 11.9 × 10 9/l 4–11 × 10 9/l

Platelets (PLT) 192 × 10 9/l 150–400 × 10 9/l

Sodium (Na +) 134 mmol/l 135–145 mmol/l

Potassium (K +) 4.3 mmol/l 3.5–5.0 mmol/l

Creatinine (Cr) 105 µmol/l 50–120 µmol/l

C-reactive protein (CRP) 185 mg/l < 10 mg/l

Plasma glucose 5.7 mmol/l < 11.1 mmol/l

Cerebrospinal fluid (CSF) glucose 3.1 mmol /l

CSF protein 1.5 g/l 0.2–0.4 g/l

CSF microscopy Lymphocytic pleocytosis

What is the most likely diagnosis?

A Enterovirus meningitis

B Herpes simplex encephalitis

C Listeria meningitis

D Meningococcal meningitis

E Tuberculous meningitis

https://mypastest.pastest.com/Secure/TestMe/TimedBrowser/1098274#Top 1/4
8/25/22, 9:57 PM MyPastest

Explanation 

C Listeria meningitis

Listeria meningitis is associated with exposure from unpasteurised dairy products and occurs with increasing
frequency in children, pregnant women and the elderly. The recent trip to Northern France suggests possible
exposure to unpasteurised cheeses. The cerebrospinal fluid (CSF) protein level is significantly increased, the
glucose level is significantly reduced and there is lymphocytosis – this is highly suspicious for the diagnosis.
Intravenous amoxicillin and gentamicin in combination is the intervention of choice.

A Enterovirus meningitis

Enterovirus meningitis leads to symptoms of headache and neck stiffness, but not the marked reduction in
glucose level and elevation in protein level seen in this patient.

B Herpes simplex encephalitis

Herpes simplex encephalitis would not be expected to lead to the marked elevation in protein level seen here
or the level of glucose reduction. It is also more likely to lead to temporal lobe inflammation, memory loss and
behavioural changes, compared to the typical symptoms of meningitis seen here.

D Meningococcal meningitis

Meningococcal meningitis usually leads to CSF neutrophilia, rather than the lymphocytosis seen here. Other
features such as a typical meningococcal rash are also absent.

E Tuberculous meningitis

Although tuberculous meningitis does lead to elevated CSF protein level and lymphocytosis, the presentation
here is acute and the CSF glucose level is significantly reduced, pointing towards an alternative cause.
73169

Rate this question:

Next Question

Previous Question Tag Question

Feedback End Review

Difficulty: Average

https://mypastest.pastest.com/Secure/TestMe/TimedBrowser/1098274#Top 2/4
8/25/22, 9:57 PM MyPastest

0:00:00/03:00:00

A 72-year-old woman presents to the Urology Clinic for review. She has been prescribed oxybutynin for
overactive bladder symptoms but is unable to tolerate it due to worsening confusion.

What is the most appropriate alternative to oxybutynin?

A Doxazosin

B Finasteride

C Intermittent self-catheterisation

D Mirabegron

E Tolterodine

Explanation 

D Mirabegron

Mirabegron is a beta-3 adrenergic receptor agonist which causes relaxation of the detrusor muscle and
subsequent increase in bladder capacity. It does not have the same liability as anticholinergic agents with
respect to confusion, making it the preferred option here. It may cause hypertension in patients with poorly
controlled blood pressure, meaning that it should be monitored intermittently after starting therapy.

A Doxazosin

Doxazosin is an alpha-adrenergic inhibitor. It can be used in the treatment of hypertension and symptom
management for patients with benign prostatic hypertrophy.

B Finasteride

Finasteride is a 5-alpha reductase inhibitor. It is used in the treatment of benign prostatic hypertrophy, rather
than overactive bladder.

C Intermittent self-catheterisation

https://mypastest.pastest.com/Secure/TestMe/TimedBrowser/1098274#Top 1/3
8/25/22, 9:57 PM MyPastest

Intermittent self-catheterisation is the best option for patients with chronic urinary retention. In this case, with
an overactive bladder, an alternative medical intervention with mirabegron is preferred.

E Tolterodine

Tolterodine is an M2 and M3 muscarinic receptor antagonist used in the treatment of overactive bladder. It
does still cause hallucinations and confusion in some patients. For this reason, mirabegron is the preferred
option.
73153

Rate this question:

Next Question

Previous Question Tag Question

Feedback End Review

Difficulty: Average

Peer Responses %

Q. Answered Flagged

Q1

Q2

Q3

Q4

Q5

Q6

https://mypastest.pastest.com/Secure/TestMe/TimedBrowser/1098274#Top 2/3
8/25/22, 9:57 PM MyPastest

0:00:00/03:00:00

A 78-year-old man, who has been admitted with left lower lobe pneumonia and a non-ST-segment elevation
myocardial infarction (NSTEMI) 36 hours ago, presents to the Emergency Department. He had a prolonged
period of hypotension on admission, which resolved with fluid loading. His blood pressure is 139/82 mmHg,
and his heart rate is 79 bpm and regular. He has a central venous pressure (CVP) line inserted, which is
currently measuring 14 cmH 2O. There is minor pitting oedema of both ankles. He has passed 30 ml of urine in
the past two hours.

Investigation Result Normal value

Haemoglobin (Hb) 137 g/l 115–155 g/l

White cell count (WCC) 14.1 × 10 9/l 4–11 × 10 9/l

Platelets (PLT) 191 × 10 9/l 150–400 × 10 9/l

Sodium (Na +) 142 mmol/l 135–145 mmol/l

Potassium (K +) 5.2 mmol/l 3.5–5.0 mmol/l

Creatinine (Cr) 279 µmol/l (up from 169 on admission) 50–120 µmol/l

Urine Blood negative, granular casts seen

What is the most likely cause of this patient’s renal impairment?

A Acute interstitial nephritis

B Acute tubular necrosis

C Acute urinary retention

D Renal artery occlusion

E Renal vein thrombosis

Explanation 

B Acute tubular necrosis

https://mypastest.pastest.com/Secure/TestMe/TimedBrowser/1098274#Top 1/3
8/25/22, 9:57 PM MyPastest

Acute tubular necrosis is suggested by reduced urine output a few hours after a period of hypotension. This
could be as a result of acute sepsis, eg urinary tract infection or pneumonia, or due to pump failure, eg
transient low blood pressure related to myocardial ischaemia. Urinary casts seen on urinalysis are consistent
with tubular necrosis. Although furosemide and dopamine are used as therapies in acute tubular necrosis,
there is no strong evidence to support their use. Support with dialysis may well be required in the event a
further deterioration in renal function is seen. Any drugs that are potentially nephrotoxic should be
discontinued.

A Acute interstitial nephritis

Acute interstitial nephritis is associated with some antibiotic therapies. However, features that would be
expected, including eosinophilia, are absent in this patient.

C Acute urinary retention

Acute urinary retention is a possible alternative cause for reduced urine output, although abdominal pain due
to bladder distension would be expected and the period of hypotension is a better pointer towards acute
tubular necrosis.

D Renal artery occlusion

Like renal vein thrombosis, renal artery occlusion presents with a significant increase in proteinuria, as well as
with pain over the affected kidney.

E Renal vein thrombosis

Renal vein thrombosis is associated with a marked increase in urinary protein excretion and often a dull ache
over the affected kidney.
73193

Rate this question:

Next Question

Previous Question Tag Question

Feedback End Review

Difficulty: Average

Peer Responses %

https://mypastest.pastest.com/Secure/TestMe/TimedBrowser/1098274#Top 2/3
8/25/22, 9:57 PM MyPastest

0:00:00/03:00:00

A 54-year-old man who is taking long-term sertraline for depression is brought to the Emergency
Department. He has recently started tramadol for back pain. He is agitated and confused and has an obvious
tremor. His blood pressure is 170/90 mmHg and his heart rate is 100 bpm and regular. His temperature is
38.9 °C. He has bilateral dilated pupils. He also has increased muscle tone, hyperreflexia and spontaneous
clonus.

What is the most likely diagnosis?

A Delirium tremens

B Neuroleptic malignant syndrome

C Serotonin syndrome

D Tramadol withdrawal

E Viral encephalitis

Explanation 

C Serotonin syndrome

When tramadol is prescribed in patients who are chronically using selective serotonin reuptake inhibitors,
such as sertraline, or a serotonin and noradrenaline reuptake inhibitor, it can precipitate an acute rise in
serotonin levels. This leads to the development of serotonin syndrome, as here, with mydriasis, confusion,
hypertension, pyrexia, increased muscle tone, reflexes and spontaneous clonus. Both agents should be
stopped, and measures to control fever and manage fluid balance are essential. In severe cases,
cyproheptadine can be of value.

A Delirium tremens

Delirium tremens can present with disorientation and agitation with sympathetic activation. However, the
myoclonus seen here would be very unusual.

B Neuroleptic malignant syndrome

https://mypastest.pastest.com/Secure/TestMe/TimedBrowser/1098274#Top 1/3
8/25/22, 9:57 PM MyPastest

Neuroleptic malignant syndrome is related to the use of traditional and atypical antipsychotics such as
haloperidol or risperidone.

D Tramadol withdrawal

Tramadol withdrawal may lead to hyperactivity and sweating. However, it would not be expected to lead to
pyrexia, increased muscle tone and clonus, as seen in this patient.

E Viral encephalitis

Given the degree of pyrexia and increased tone, and the recent prescription of tramadol, serotonin syndrome
is the most likely cause of this patient’s symptoms.
73121

Rate this question:

Next Question

Previous Question Tag Question

Feedback End Review

Difficulty: Average

Peer Responses %

Q. Answered Flagged

Q1

Q2

Q3

Q4

https://mypastest.pastest.com/Secure/TestMe/TimedBrowser/1098274#Top 2/3
8/25/22, 9:57 PM MyPastest

0:00:00/03:00:00

A 56-year-old man who has been diagnosed with Barrett’s oesophagus returns to his General Practitioner six
months after commencing high-dose proton pump inhibitor therapy and a surveillance endoscopy. His biopsy
shows persistent areas of low-grade dysplasia.

What is the most appropriate next step?

A Add domperidone

B Continue high-dose proton pump inhibitor

C Oesophagectomy

D Photodynamic therapy

E Radiofrequency ablation

Explanation 

E Radiofrequency ablation

Studies have shown that low-grade dysplasia is still associated with relatively high rates of progression to
either high-grade dysplasia or oesophageal cancer, even when patients are treated with high-dose proton
pump inhibitors. Over three years, 26.5% of patients who continued on high-dose proton pump inhibitors only
progressed to high-grade dysplasia or cancer, compared to 1% when they were treated with radiofrequency
ablation. This has driven radiofrequency ablation to be recommended by the British Society of
Gastroenterology.

A Add domperidone

Unfortunately, adding a prokinetic antiemetic does not reduce mortality from cancer in Barrett’s oesophagus.
It is not of value here.

B Continue high-dose proton pump inhibitor

This is inappropriate because data suggest that progression to high-grade dysplasia or oesophageal cancer is
approximately 27% over three years.

https://mypastest.pastest.com/Secure/TestMe/TimedBrowser/1098274#Top 1/3
8/25/22, 9:57 PM MyPastest

C Oesophagectomy

Oesophagectomy is only recommended for high-grade dysplasia of the oesophageal mucosa. Low-grade
dysplasia is managed with radiofrequency ablation in the first instance.

D Photodynamic therapy

Photodynamic therapy is thought to be less effective than radiofrequency ablation in treating Barrett’s
oesophagus.
73145

Rate this question:

Next Question

Previous Question Tag Question

Feedback End Review

Difficulty: Average

Peer Responses %

Q. Answered Flagged

Q1

Q2

Q3

Q4

Q5

https://mypastest.pastest.com/Secure/TestMe/TimedBrowser/1098274#Top 2/3
8/25/22, 9:58 PM MyPastest

0:00:00/03:00:00

A 72-year-old man who has known ischaemic heart disease collapses and is brought by ambulance to the
Emergency Department. His blood pressure is 70/50 mmHg and he has a rapid tachycardia. The cardiac
monitor shows monomorphic ventricular tachycardia (VT) with a ventricular rate of 210.

What is the most appropriate next step?

A Intravenous (IV) amiodarone

B IV lidocaine

C IV magnesium

D Synchronised cardioversion

E Unsynchronised defibrillation

Explanation 

D Synchronised cardioversion

This patient has VT with a reduced, but maintained, cardiac output (blood pressure of 70/50 mmHg). He is
too unstable for treatment with IV antiarrhythmics, leaving synchronised direct current (DC) cardioversion as
the only option. The intervention carries a risk of R-on-T phenomenon, which can lead to deterioration to
ventricular fibrillation (VF). Other interventions such as IV magnesium should be considered post-
cardioversion.
Image credit: By Associate Professor Frank Gaillard, CC BY-SA 3.0.

A Intravenous (IV) amiodarone

Amiodarone is effective for chemical cardioversion of VT, but in this case, given the significant hypotension,
synchronised cardioversion is indicated.

B IV lidocaine

Although lidocaine is a membrane stabiliser that can be used to treat VT, this patient is unstable and requires
synchronised cardioversion as the next step.

https://mypastest.pastest.com/Secure/TestMe/TimedBrowser/1098274#Top 1/3
8/25/22, 9:58 PM MyPastest

C IV magnesium

Magnesium sulfate infusion may be an effective intervention for VT, even when serum magnesium is in the
normal range. As such, it may well be considered as a next step post-cardioversion. A bolus of up to 4 g may
be given.

E Unsynchronised defibrillation

Unsynchronised defibrillation is used for pulseless VT or VF. In this case, with a measurable blood pressure,
synchronised cardioversion is the most appropriate next step.
73109

Rate this question:

Next Question

Previous Question Tag Question

Feedback End Review

Difficulty: Average

Peer Responses %

Q. Answered Flagged

Q1

Q2

Q3

Q4

Q5

https://mypastest.pastest.com/Secure/TestMe/TimedBrowser/1098274#Top 2/3
8/25/22, 9:58 PM MyPastest

0:00:00/03:00:00

A 19-year-old man with learning difficulties is referred to the Endocrine Clinic due to a lack of energy, muscle
weakness and intermittent pins and needles in his hands. On examination, it is noted that he has a short
stature and a round face. His fifth digits appear abnormally shortened on each side of the hands and feet.

Investigations:

Investigation Result Normal value

Calcium (Ca 2+) 1.97 mmol/l 2.1–2.65 mmol/l

Phosphate (PO 4 −) 1.7 mmol/l 0.8–1.5 mmol/l

Parathyroid hormone (PTH) 18.2 pmol/l 1.6–6.9 pmol/l

What is the most likely diagnosis?

A Hypoparathyroidism

B Osteomalacia

C Primary hyperparathyroidism

D Pseudohypoparathyroidism

E Secondary hyperparathyroidism

Explanation 

D Pseudohypoparathyroidism

This patient has shortened fifth digits, a picture of biochemical hypoparathyroidism and an elevated
parathyroid hormone (PTH) level. This is consistent with PTH resistance, which is inherited and caused by
PTH resistance at the level of the PTH receptor. 1,25-hydroxy vitamin D is the standard intervention for
pseudohypoparathyroidism.

A Hypoparathyroidism

https://mypastest.pastest.com/Secure/TestMe/TimedBrowser/1098274#Top 1/3
8/25/22, 9:58 PM MyPastest

Hypoparathyroidism presents with the biochemical picture here, apart from the elevated PTH level, which is
consistent with pseudohypoparathyroidism.

B Osteomalacia

Osteomalacia is not associated with the shortened fifth digits seen here, which are consistent with
pseudohypoparathyroidism.

C Primary hyperparathyroidism

Primary hyperparathyroidism is caused by parathyroid hyperplasia or a parathyroid adenoma and is


characterised by hypercalcaemia and an elevated PTH level, without significant renal impairment.

E Secondary hyperparathyroidism

Secondary hyperparathyroidism is seen in patients with abnormal renal function and occurs in response to
defective hydroxylation of vitamin D and abnormalities of renal calcium and phosphate handling.
73136

Rate this question:

Next Question

Previous Question Tag Question

Feedback End Review

Difficulty: Average

Peer Responses %

Q. Answered Flagged

Q1

https://mypastest.pastest.com/Secure/TestMe/TimedBrowser/1098274#Top 2/3
8/25/22, 9:58 PM MyPastest

0:00:00/03:00:00

A 62-year-old man is brought to the Emergency Department by his partner because he has become
increasingly drowsy and confused and appears to be using his left arm less. He drinks seven litres of strong
beer per day, as well as whisky.

His computed tomography (CT) head is shown below:


What is the likely underlying diagnosis?

A Cerebral abscess

B Extradural haematoma

C Meningioma

D Subarachnoid haemorrhage

E Subdural haematoma

Explanation 

E Subdural haematoma

https://mypastest.pastest.com/Secure/TestMe/TimedBrowser/1098274#Top 1/3
8/25/22, 9:58 PM MyPastest

Subdural haematomas are much more common in patients with alcoholism, and the subacute presentation
with left-sided symptoms and confusion fits with the diagnosis. Computed tomography (CT) shows a rim of
an isodense subdural haematoma consistent with an injury occurring at least a few days earlier. The scan
findings and the patient’s symptoms should be discussed with Neurosurgery to decide whether an operative
intervention is appropriate. This usually depends on the premorbid state, the severity of symptoms and the
degree of midline shift/compression of brain tissue noted on the scan.

Image credit: CC BY-NC-SA 2.0

A Cerebral abscess

A cerebral abscess is more likely to be rounded, compared to the rim of haemorrhage seen here. In addition,
there are no other features to suggest an infective cause for this patient’s symptoms.

B Extradural haematoma

Extradural haematomas are most commonly seen as a result of damage to the middle meningeal artery. They
present soon after the injury with symptoms related to compression of the surrounding brain tissue.

C Meningioma

Meningiomas are well-defined extra-axial lesions that are clearly originating from the dura. They may show
cystic degeneration and there may be oedema of the surrounding brain tissue.

D Subarachnoid haemorrhage

Subarachnoid haemorrhage leads to acute symptoms of headache and neck stiffness. CT also often
demonstrates free blood within the subarachnoid space, and ventricular extension occurs in 25% of cases.
The lateralisation and position of the bleed here is much more consistent with a subdural haematoma.
73178

Rate this question:

Next Question

Previous Question Tag Question

Feedback End Review

Difficulty: Average

Peer Responses %

https://mypastest.pastest.com/Secure/TestMe/TimedBrowser/1098274#Top 2/3
8/25/22, 9:58 PM MyPastest

0:00:00/03:00:00

A 45-year-old man has a large upper gastrointestinal haemorrhage and is admitted to the Emergency
Department. According to the ambulance staff, the toilet was filled with fresh blood. He is a known alcoholic
who has hepatitis B and drinks one litre of whisky per day. His blood pressure is 100/60 mmHg and his heart
rate is 120 bpm. He is tender in the epigastrium. There are multiple spider naevi over his upper body.
Investigations:

Investigation Result Normal value

Haemoglobin (Hb) 91 g/l 115–155 g/l

White cell count (WCC) 6.7 × 10 9/l 4–11 × 10 9/l

Platelets (PLT) 52 × 10 9/l 150–400 × 10 9/l

Sodium (Na +) 143 mmol/l 135–145 mmol/l

Potassium (K +) 5.1 mmol/l 3.5–5.0 mmol/l

Creatinine (Cr) 99 µmol/l 50–120 µmol/l

What is the most useful next step?

A Factor VIIa

B Pantoprazole

C Platelet transfusion

D Propranolol

E Terlipressin

Explanation 

E Terlipressin

Terlipressin is a vasopressin analogue which has been proven to have an impact on outcomes in patients with
variceal haemorrhage. Given this patient is a known alcoholic with signs of chronic liver disease, this is the
most likely cause of his presentation. It reduces the portal blood pressure and contracts the oesophageal
https://mypastest.pastest.com/Secure/TestMe/TimedBrowser/1098274#Top 1/3
8/25/22, 9:58 PM MyPastest

smooth muscle, leading to subsequent compression of oesophageal varices. It should be given before
endoscopy and can be continued for a few days afterwards. Over the longer term, beta blockade is the main
intervention for portal hypertension.

A Factor VIIa

Recombinant factor VII is not recommended to control bleeding from oesophageal varices. Terlipressin is the
first-line agent shown to most improve prognosis.

B Pantoprazole

Pantoprazole is a proton pump inhibitor. It is of value when used in the acute treatment of non-variceal
bleeding.

C Platelet transfusion

Platelet transfusion is recommended in the treatment of upper gastrointestinal bleeding only when the PLT
count falls below 40. This man is likely to have chronically reduced platelets.

D Propranolol

Propranolol is recommended for chronic prophylaxis against varices in patients with portal hypertension.
When used acutely here, it may lead to circulatory compromise.
73147

Rate this question:

Next Question

Previous Question Tag Question

Feedback End Review

Difficulty: Average

Peer Responses %

https://mypastest.pastest.com/Secure/TestMe/TimedBrowser/1098274#Top 2/3
8/25/22, 10:00 PM MyPastest

0:00:00/03:00:00

A 19-year-old man presents to his General Practitioner six months after diagnosis with type I diabetes. It is
calculated by body weight that he requires approximately 48 units of insulin per day.

What is the most appropriate insulin dose for breakfast?

A Four units

B Eight units

C 12 units

D 16 units

E 24 units

Explanation 

B Eight units

This is the appropriate breakfast insulin dose. A usual starting point for bolus insulin requirements is to divide
the total daily insulin requirements by two, giving 24 units for breakfast, lunch and dinner. Dividing this by
three gives a breakfast dose of eight units.

A Four units

A dose of four units is likely to represent underdosing of around 50% for mealtime insulin. This risks
persistently elevated blood glucose levels.

C 12 units

A dose of 12 units represents a significant excess to the likely breakfast requirements, given that the daily
bolus of 24 units is usually divided into three doses. Unless the patient eats a much larger meal first thing in
the morning, this is likely to represent a risk of hypoglycaemia.

D 16 units

https://mypastest.pastest.com/Secure/TestMe/TimedBrowser/1098274#Top 1/3
8/25/22, 10:00 PM MyPastest

A dose of 16 units represents the insulin dose for two main meals, rather than one, given that 24 units should
be the total bolus dose administered in a day.

E 24 units

Approximately 50% of the total daily insulin dose is usually given as a bolus component. However, this is
divided across the three main meals of the day, meaning that the most appropriate insulin dose is eight,
rather than 24, units.
73135

Rate this question:

Next Question

Previous Question Tag Question

Feedback End Review

Difficulty: Average

Peer Responses %

Q. Answered Flagged

Q1

Q2

Q3

Q4

Q5

Q6

https://mypastest.pastest.com/Secure/TestMe/TimedBrowser/1098274#Top 2/3
8/25/22, 10:00 PM MyPastest

0:00:00/03:00:00

A 43-year-old woman presents to the Emergency Department with severe neck pain and occipital headache a
few hours after a prolonged trip to the hairdressers. She also has vertigo, diplopia, nausea and vomiting. Her
blood pressure is 155/90 mmHg. Her heart rate is 85 bpm. Neurological examination reveals abnormal
sensations affecting the left side of the face.
What is the most likely diagnosis?

A Carotid artery dissection

B Ramsay Hunt syndrome

C Subarachnoid haemorrhage

D Vertebral artery dissection

E Whiplash

Explanation 

D Vertebral artery dissection

Vertebral artery dissection is increasingly recognised as a cause of stroke in patients under 45 years. It
presents with severe occipital headache and neck pain, and focal neurological signs may develop some hours
after the original neurological insult. It can occur as a result of excessive neck extension, eg related to a
prolonged trip to the hair salon.
A range of symptoms can occur, including:

ipsilateral facial pain and numbness


dysarthria or hoarseness
contralateral loss of pain and temperature sensation affecting the trunk and limbs
ipsilateral loss of taste
hiccups
vertigo
nausea and vomiting
diplopia
dysphagia.
https://mypastest.pastest.com/Secure/TestMe/TimedBrowser/1098274#Top 1/3
8/25/22, 10:00 PM MyPastest

Magnetic resonance (MR) angiography is the most sensitive way to confirm the diagnosis. Computed
tomography (CT) is thought to be less sensitive with respect to identifying a dissection, compared to the
anterior cerebral circulation.

A Carotid artery dissection

Carotid dissection is characteristically associated with a partial Horner syndrome, motor and sensory loss on
the same side of the body and speech disturbance.

B Ramsay Hunt syndrome

Ramsay Hunt syndrome is paralysis of the facial nerve related to herpes zoster infection affecting the ear.
This patient did not present with the features associated with Ramsay Hunt syndrome.

C Subarachnoid haemorrhage

Vertebral artery dissection is more likely here than subarachnoid haemorrhage, given the young age of the
patient and the clear history of prolonged neck extension.

E Whiplash

Whiplash is a cause of neck pain and posterior headache, although it would not fit here with abnormal
sensation affecting one side of the face.
73177

Rate this question:

Next Question

Previous Question Tag Question

Feedback End Review

Difficulty: Average

Peer Responses %

https://mypastest.pastest.com/Secure/TestMe/TimedBrowser/1098274#Top 2/3
8/25/22, 10:00 PM MyPastest

0:00:00/03:00:00

A 45-year-old woman is admitted to the Emergency Department claiming she has been unable to get out of
her chair for three days. She tells you she has become increasingly weak over the past four months and has
only been climbing the stairs once per day. She has had problems with shortness of breath, for which her
General Practitioner has prescribed a salbutamol inhaler, but with no improvement. Her blood pressure is
122/82 mmHg and her heart rate is 65 bpm and regular. There is no facial rash. She has marked proximal
muscle weakness affecting her shoulders and hips.

Investigation Result Normal value

Haemoglobin (Hb) 133 g/l 115–155 g/l

White cell count (WCC) 7.1 × 10 9/l 4.0–11.0 × 10 9/l

Platelets (PLT) 180 × 10 9/l 150–400 × 10 9/l

Sodium (Na +) 142 mmol/l 135–145 mmol/l

Potassium (K +) 3.9 mmol/l 3.5–5.0 mmol/l

Creatinine (Cr) 75 µmol/l 50–120 µmol/l

Creatine kinase (CK) 1921 U/l 40–320 U/l

Erythrocyte sedimentation rate (ESR) 71 mm/hour < 10 mm/hour

Chest X-ray Bilateral pulmonary infiltrates

What is the most likely diagnosis?

A Dermatomyositis

B Myasthenia gravis

C Osteomalacia

D Polymyalgia rheumatica

E Polymyositis

Explanation 

https://mypastest.pastest.com/Secure/TestMe/TimedBrowser/1098274#Top 1/3
8/25/22, 10:00 PM MyPastest

E Polymyositis

The progressive proximal muscle weakness seen in this patient in the absence of a rash, but with a significant
rise in creatine kinase (CK) level, raises the possibility of polymyositis. The shortness of breath and pulmonary
infiltrates may indicate pulmonary fibrosis, which is associated with anti-Jo1 antibodies, found in around 20%
of patients and indicating a poor prognosis. High-dose corticosteroids at a dose of 1 mg/kg are usually
instigated, with tapering as symptoms improve. Azathioprine is a typical steroid-sparing agent.

A Dermatomyositis

Although the slowly progressive nature of this patient’s symptoms, coupled with proximal muscle weakness
and raised ESR, could point towards dermatomyositis, the absence of a facial rash makes polymyositis a more
likely cause of her symptoms.

B Myasthenia gravis

Myasthenia gravis is associated with ascending flaccid paralysis, and not with elevated CK levels, which
therefore counts against this being the diagnosis.

C Osteomalacia

Osteomalacia is a cause of proximal myopathy, although it is due to low vitamin D levels, most commonly
associated with calcium at the lower end of the normal range and not with a significant rise in CK to the levels
seen here.

D Polymyalgia rheumatica

Polymyalgia is associated with pain and stiffness affecting proximal muscles in an older age group than that
seen here, without the very large rise in CK level.
73205

Rate this question:

Next Question

Previous Question Tag Question

Feedback End Review

Difficulty: Average

Peer Responses %

https://mypastest.pastest.com/Secure/TestMe/TimedBrowser/1098274#Top 2/3
8/25/22, 10:00 PM MyPastest

0:00:00/03:00:00

A 45-year-old woman presents to her General Practitioner (GP) with complaints of palpitations, shortness of
breath and night sweats, which have increased in frequency over the last four months. She has also suffered
transient weakness of the left side of her face, which, she was told by her GP, was due to migraine. Her blood
pressure is 132/82 mmHg and her heart rate 74 bpm and regular. There is a mid-diastolic murmur.
Her echocardiogram (ECHO) is shown below:


What is the most likely diagnosis?

A Intra-atrial thrombus

B Left atrial myxoma

C Left atrial rhabdomyoma

D Papillary fibroelastoma

E Subacute bacterial endocarditis

Explanation 

B Left atrial myxoma

https://mypastest.pastest.com/Secure/TestMe/TimedBrowser/1098274#Top 1/3
8/25/22, 10:00 PM MyPastest

This patient has a ball-shaped mass, with some ECHO heterogeneity, which is situated within the left atrium.
Given the patient’s age and chronicity of symptoms, this fits with a diagnosis of left atrial myxoma. Patients
may present with symptoms of cardiac insufficiency, including palpitations, or suffer embolic phenomena,
which may have accounted for this patient’s transient facial weakness. Surgery to excise the myxoma is
indicated.

Image source:
https://commons.wikimedia.org/wiki/File:Myxoma_in_left_atrium_E00632_(CardioNetworks_ECHOpedia).jpg

A Intra-atrial thrombus

Intra-atrial thrombus may appear as a heterogeneous mass within the atrium. However, it would be very
unlikely to have the rounded tumour-like appearance seen here, which is much more characteristic of atrial
myxoma.

C Left atrial rhabdomyoma

Rhabdomyomas are the most common cardiac tumours seen in children. More than 80% disappear in the first
few years of life and are most commonly associated with tuberous sclerosis.

D Papillary fibroelastoma

Papillary fibroelastomas are more likely seen in older adults, usually as small, pedunculated tumours
attached either to the aortic valve or to the mitral valve.

E Subacute bacterial endocarditis

Subacute bacterial endocarditis is associated with vegetations which arise from the valve itself. In this ECHO,
there is a ball-shaped mass within the left atrium itself, more consistent with a myxoma.
73112

Rate this question:

Next Question

Previous Question Tag Question

Feedback End Review

Difficulty: Average

Peer Responses %

https://mypastest.pastest.com/Secure/TestMe/TimedBrowser/1098274#Top 2/3
8/25/22, 10:00 PM MyPastest

0:00:00/03:00:00

A 38-year-old woman presents to her General Practitioner with worsening shortness of breath and
palpitations and now she cannot climb the stairs. She has recently completed the third round of
chemotherapy for breast cancer.

Investigations:

Investigation Result Normal value

Haemoglobin (Hb) 101 g/l 115–155 g/l

White cell count (WCC) 7.0 × 10 9/l 4–11 × 10 9/l

Platelets (PLT) 172 × 10 9/l 150–400 × 10 9/l

Sodium (Na +) 133 mmol/l 135–145 mmol/l

Potassium (K +) 4.0 mmol/l 3.5–5.0 mmol/l

Creatinine (Cr) 99 µmol/l 50–120 µmol/l

Chest X-ray Evidence of left ventricular failure

Electrocardiogram (ECG) Atrial fibrillation ventricular rate 110 bpm

What is the most likely cause of this patient’s symptoms?

A Carboplatin-related cardiomyopathy

B Checkpoint inhibitor-induced myocarditis

C Ischaemic cardiomyopathy

D Trastuzumab-related cardiomyopathy

E Viral myocarditis

Explanation 

D Trastuzumab-related cardiomyopathy

https://mypastest.pastest.com/Secure/TestMe/TimedBrowser/1098274#Top 1/3
8/25/22, 10:00 PM MyPastest

Trastuzumab is a human epidermal growth factor receptor 2 (HER2) antagonist used in the treatment of
breast cancers which can express the HER2 receptor. Typically, HER2 expression is thought to be protective
against the development of dilative cardiomyopathy, the condition here which has led to palpitations and
symptoms of heart failure. All patients taking trastuzumab require baseline cardiac assessment, including
history and physical examination, electrocardiography, echocardiography and/or multigated acquisition scan
or magnetic resonance imaging. This should be repeated every three months on therapy and every six months
once therapy is completed until 24 months have passed.

A Carboplatin-related cardiomyopathy

Platinum-based chemotherapy has been shown, according to a Cochrane meta-analysis, not to be effective in
the treatment of breast cancer. The main concern with platinum-based chemotherapy is also renal tubular
toxicity, rather than cardiotoxicity.

B Checkpoint inhibitor-induced myocarditis

Checkpoint inhibitor-induced myocarditis does occur. However, it is a rare event and other sequelae such as
autoimmune thyroid disease and type I diabetes are much more common.

C Ischaemic cardiomyopathy

Ischaemic cardiomyopathy is much less likely to be seen than other causes of heart failure, given that this
patient is young and has no reported cardiovascular risk factors.

E Viral myocarditis

There is no prodromal illness to suggest underlying viral myocarditis, and the proximity to chemotherapy
suggests trastuzumab-related cardiomyopathy is a more likely cause.
73125

Rate this question:

Next Question

Previous Question Tag Question

Feedback End Review

Difficulty: Average

Peer Responses %

https://mypastest.pastest.com/Secure/TestMe/TimedBrowser/1098274#Top 2/3
8/25/22, 10:00 PM MyPastest

0:00:00/03:00:00

A 73-year-old man presents to the Emergency Department, complaining of easy bruising and bleeding
around his gums when he cleans his teeth. He is taking warfarin for left lower limb deep vein thrombosis. His
international normalised ratio (INR) is measured at 6.0.

What is the most appropriate next step?

A 1 mg intravenous (IV) vitamin K

B 5 units IV fresh frozen plasma

C 15 μg/kg IV factor VII

D 15 mg IV protamine sulphate

E 25 units/kg IV prothrombin complex concentrate (PCC)

Explanation 

A 1 mg intravenous (IV) vitamin K

Where there is non-major bleeding, as here, and the international normalised ratio (INR) is above 5, reversal
with warfarin is recommended. The dose can be tailored from 1 to 3 mg, and warfarin restarted once the INR
has fallen to below 5. Where there is major life-threatening bleeding or urgent surgical intervention is
required, then prothrombin complex concentrate (PCC) should be used, in addition to vitamin K.

B 5 units IV fresh frozen plasma

Fresh frozen plasma is used for reversal of warfarin therapy only when acute intervention for severe
haemorrhage or urgent surgery is required and PCC is unavailable.

C 15 μg/kg IV factor VII

Factor VII only partially reverses the effects of warfarin on the INR. Three-factor PCC is preferred in the event
that acute reversal of warfarin treatment is required.

D 15 mg IV protamine sulphate
https://mypastest.pastest.com/Secure/TestMe/TimedBrowser/1098274#Top 1/3
8/25/22, 10:00 PM MyPastest

Protamine sulphate is a reversal agent for heparin. It is not of value in reversing anticoagulation associated
with warfarin therapy.

E 25 units/kg IV prothrombin complex concentrate (PCC)

Typically, PCC contains clotting factors VII, IX and X. It is used for acute reversal of warfarin therapy when
there is life-threatening bleeding or a need for acute surgical intervention.
73122

Rate this question:

Next Question

Previous Question Tag Question

Feedback End Review

Difficulty: Average

Peer Responses %

Q. Answered Flagged

Q1

Q2

Q3

Q4

Q5

Q6

https://mypastest.pastest.com/Secure/TestMe/TimedBrowser/1098274#Top 2/3
8/25/22, 10:01 PM MyPastest

0:00:00/03:00:00

A 38-year-old woman presents to the Emergency Department with sudden painless loss of vision over a few
minutes affecting her left eye. She takes the oral contraceptive pill and had a left below-knee deep vein
thrombosis (DVT) 20 years earlier.

A left retinal image is shown below:


What is the most likely diagnosis?

A Acute closed-angle glaucoma

B Acute optic neuritis

C Branch retinal artery occlusion

D Central retinal artery occlusion

E Central retinal vein occlusion

Explanation 

D Central retinal artery occlusion

https://mypastest.pastest.com/Secure/TestMe/TimedBrowser/1098274#Top 1/3
8/25/22, 10:01 PM MyPastest

Central retinal artery occlusion leads to sudden painless visual loss, a pale retina and a cherry red spot, as
seen here. The cherry red spot occurs because a small area of the retina is still perfused by the posterior
ciliary arteries. The patient may have an inherited coagulation disorder, given the history of deep vein
thrombosis (DVT) at a young age, and potentially increased risk of arterial occlusion because of oral
contraceptive pill use. Ocular massage and acute lowering of the intraocular pressure are initial steps in
therapy.

Image credit: By sidthedoc, CC BY-SA 4.0.

A Acute closed-angle glaucoma

Acute closed-angle glaucoma is associated with optic disc cupping and intense ocular pain. It does not fit
with the presentation shown by this patient.

B Acute optic neuritis

Acute optic neuritis is often associated with ocular pain and evidence of optic disc swelling, without the
cherry red spot seen in this patient.

C Branch retinal artery occlusion

Branch retinal artery occlusion leads to sectoral retinal pallor, not to the cherry red spot that this patient
presented with.

E Central retinal vein occlusion

Central retinal vein occlusion does lead to sudden painless visual loss. However, it causes multiple retinal
haemorrhages, rather than the pale retina and cherry red spot seen here.
73170

Rate this question:

Next Question

Previous Question Tag Question

Feedback End Review

Difficulty: Average

Peer Responses %

https://mypastest.pastest.com/Secure/TestMe/TimedBrowser/1098274#Top 2/3
8/25/22, 10:01 PM MyPastest

0:00:00/03:00:00

A 41-year-old man presents to the Endocrinology Clinic, complaining of frequent headaches and that the
shape of his face has changed over recent months with the increased prominence of his lower jaw. He also
has problems with sweating and hypertension, for which he has been seeing his General Practitioner. His
blood pressure is 148/90 mmHg, and his heart rate is 75 bpm and regular. His body mass index is 23 kg/m 2.
Investigations:

Investigation Result Normal value

Haemoglobin (Hb) 132 g/l 115–155 g/l

White cell count (WCC) 8.2 × 10 9/l 4–11 × 10 9/l

Platelets (PLT) 190 × 10 9/l 150–400 × 10 9/l

Sodium (Na +) 143 mmol/l 135–145 mmol/l

Potassium (K +) 4.9 mmol/l 3.5–5.0 mmol/l

Creatinine (Cr) 85 µmol/l 50–120 µmol/l

Glucose (fasting) 7.3 mmol/l < 7 mmol/l


What is the most useful next investigation?

A 24-hour urinary catecholamines

B 24-hour urinary-free cortisol

C 24-hour urinary 5-hydroxyindoleacetic acid

https://mypastest.pastest.com/Secure/TestMe/TimedBrowser/1098274#Top 1/4
8/25/22, 10:01 PM MyPastest

D Insulin-like growth factor 1 (IGF-1)

E Random serum cortisol

Explanation 

D Insulin-like growth factor 1 (IGF-1)

This patient’s facial features with the increased prominence of the lower jaw, coupled with the other
symptoms, including sweating, hypertension and elevated blood glucose, raise the possibility of acromegaly.
Insulin-like growth factor 1 (IGF-1) is produced by the liver and has a long half-life, and levels rise in
proportion with increasing levels of growth hormone. In the event that the IGF-1 level is elevated, an oral
glucose tolerance test may be performed with growth hormone measurement. In the event that growth
hormone levels fail to suppress, then acromegaly is confirmed. A pituitary magnetic resonance imaging
should also be performed.

Image credit: C036- Clinical Photography, Science Photo Library.

A 24-hour urinary catecholamines

Urinary catecholamines are used in the diagnosis of phaeochromocytoma, which is suggested by paroxysmal
episodes of raised blood pressure and does not fit with the change in facial appearance seen in this patient.

B 24-hour urinary-free cortisol

The 24-hour urinary-free cortisol is an investigation of choice for Cushing syndrome. Although it does lead to
impaired glucose tolerance, it also leads to rapid weight gain, which is not seen in this patient.

C 24-hour urinary 5-hydroxyindoleacetic acid

The 24-hour urinary 5-hydroxyindoleacetic acid is the investigation of choice for carcinoid syndrome, which is
characterised by flushing, diarrhoea and palpitations.

E Random serum cortisol

Cortisol has a short half-life and levels frequently fluctuate during the course of a day. As such, measuring
the cortisol level as a single value is not useful.
73142

Rate this question:

https://mypastest.pastest.com/Secure/TestMe/TimedBrowser/1098274#Top 2/4
8/25/22, 10:01 PM MyPastest

0:00:00/03:00:00

An 18-year-old man presents to his General Practitioner with complaints that he has not yet properly entered
puberty. He also has no sense of smell but wonders whether this is because he had a head injury as a baby.
Examination reveals a short penis, small testes and no significant pubic or axillary hair.

Investigations:

Investigation Result Normal value

Follicle-stimulating hormone (FSH) 1.6 IU/l 1.5–12.4 IU/l

Luteinising hormone (LH) 2.3 IU/l 1.8–8.6 IU/l

Testosterone 4.1 nmol/l > 6 nmol/l

What is the most likely diagnosis?

A Androgen insensitivity syndrome

B Kallmann syndrome

C Klinefelter syndrome

D Testicular atrophy

E Turner syndrome

Explanation 

B Kallmann syndrome

This patient has failed to enter puberty, with no sense of smell and low follicle-stimulating hormone (FSH),
luteinising hormone (LH) and testosterone levels. This can only fit with a diagnosis of Kallmann syndrome
where the olfactory bulbs fail to develop. The gonadotrophin-releasing hormone is consequently not
produced by the hypothalamus, which is the cause of low LH and FSH levels and failure to enter puberty.
Testosterone supplementation can drive the development of secondary sexual characteristics. If and when
fertility is required, then gonadotrophin supplementation can initiate sperm production.

A Androgen insensitivity syndrome

https://mypastest.pastest.com/Secure/TestMe/TimedBrowser/1098274#Top 1/3
8/25/22, 10:01 PM MyPastest

Androgen insensitivity syndrome is driven by testosterone resistance. It leads to the presence of female
external genitalia and elevated testosterone levels.

C Klinefelter syndrome

Klinefelter syndrome is associated with small tests as here. However, LH and FSH levels are moderately
elevated. Patients also have a normal sense of smell.

D Testicular atrophy

Testicular atrophy in young men may occur in relation to mumps infection. High FSH and LH levels would be
expected, however, compared to the low values, taking into account the low testosterone level seen here.

E Turner syndrome

Turner syndrome is associated with an XO karyotype, short stature and female external genitalia. Therefore, it
does not fit with the scenario seen in this patient.
73139

Rate this question:

Next Question

Previous Question Tag Question

Feedback End Review

Difficulty: Average

Peer Responses %

Q. Answered Flagged

https://mypastest.pastest.com/Secure/TestMe/TimedBrowser/1098274#Top 2/3
8/25/22, 10:01 PM MyPastest

0:00:00/03:00:00

A 34-year-old woman who is 18 weeks pregnant with her second child presents to the Emergency
Department with colicky right upper quadrant abdominal pain and vomiting. Her temperature is 38.2 °C. Her
heart rate is 92 bpm and regular, and her blood pressure is 110/82 mmHg. On examination, there is
significant right upper quadrant tenderness.

Investigation Result Normal value

Haemoglobin (Hb) 119 g/l 115–155 g/l

White cell count (WCC) 13.9 × 10 9/l 4–11 × 10 9/l

Platelets (PLT) 180 × 10 9/l 150–400 × 10 9/l

Sodium (Na +) 140 mmol/l 135–145 mmol/l

Potassium (K +) 3.5 mmol/l 3.5–5.0 mmol/l

Creatinine (Cr) 95 µmol/l 50–120 µmol/l

Alanine transaminase (ALT) 45U/l 1–40 U/l

Alkaline phosphatase 21 U/l 30–130 U/l

Bilirubin 181 µmol/l < 21 µmol/l

Thick-walled gall bladder


Abdominal ultrasound
containing two large gallstones

What is the best way to manage this patient?

A Co-amoxiclav for two weeks

B Endoscopic retrograde cholangiopancreatography

C Laparoscopic cholecystectomy

D Open cholecystectomy

E Ursodeoxycholic acid

Explanation 

https://mypastest.pastest.com/Secure/TestMe/TimedBrowser/1098274#Top 1/4
8/25/22, 10:01 PM MyPastest

C Laparoscopic cholecystectomy

This patient has active cholecystitis, which puts her at a significantly increased risk of miscarriage. Most of the
data on outcomes with laparoscopic cholecystectomy come from the first and second trimester and show that
it is associated with around 60% less chance of miscarriage vs the open operation. It is usually performed
head-up slightly and tilted to the left vs a laparoscopic cholecystectomy in a non-pregnant patient.

A Co-amoxiclav for two weeks

Given this patient has evidence of active gall bladder inflammation, antibiotics alone are likely to be
ineffective and increase this patient’s risk of miscarriage.

B Endoscopic retrograde cholangiopancreatography

Endoscopic retrograde cholangiopancreatography is of most value in treating obstructive jaundice when there
are stones within the common bile duct. In this case, laparoscopic cholecystectomy is preferred.

D Open cholecystectomy

Open cholecystectomy is not preferred because a meta-analysis suggests it has a poorer outcome vs a
laparoscopic intervention, particularly in the first and second trimesters.

E Ursodeoxycholic acid

Ursodeoxycholic acid takes some time to impact gallstone disease and the recurrence rate is high in patients
with gallstones treated with the drug. It is more useful in patients with primary biliary cirrhosis.
73149

Rate this question:

Next Question

Previous Question Tag Question

Feedback End Review

Difficulty: Average

Peer Responses %

https://mypastest.pastest.com/Secure/TestMe/TimedBrowser/1098274#Top 2/4
8/25/22, 10:01 PM MyPastest

0:00:00/03:00:00

A 29-year-old man is reviewed in the Cardiology Clinic following a diagnosis of hypertrophic obstructive
cardiomyopathy (HOCM). He has significant left ventricular outflow tract obstruction and is prescribed a beta-
blocker. He has suffered no episodes of collapse, although a 24-hour electrocardiogram (ECG) has shown
three periods of non-sustained ventricular tachycardia (VT). His father had HOCM and died following a
cardiac arrest.

What is the most appropriate next step?

A Implantable cardioverter–defibrillator

B Oral amiodarone

C Oral disopyramide

D Reassurance

E Septal myomectomy

Explanation 

A Implantable cardioverter–defibrillator

This patient has a family history of sudden death related to hypertrophic obstructive cardiomyopathy
(HOCM), as well as three episodes of ventricular tachycardia (VT) recorded on his 24-hour tape. Although
beta-blockers reduce symptoms of left ventricular outflow tract obstruction, they do not reduce the risk of VT
and this patient is at high risk. As such, implantation of a cardioverter–defibrillator is the most appropriate
next step.

B Oral amiodarone

Amiodarone reduces the risk of ventricular arrhythmias but carries significant treatment-related morbidity,
with an increased risk of liver and pulmonary fibrosis, thyroid dysfunction and photosensitivity. Therefore, an
implantable cardioverter–defibrillator is a more appropriate next step.

C Oral disopyramide

https://mypastest.pastest.com/Secure/TestMe/TimedBrowser/1098274#Top 1/3
8/25/22, 10:01 PM MyPastest

Oral disopyramide does not reduce the risk of ventricular arrhythmia. As such, it is not the most appropriate.

D Reassurance

Although beta-blockers reduce symptoms of left ventricular outflow tract obstruction, they do not impact on
the risk of ventricular arrhythmia. As such, reassurance is inappropriate here.

E Septal myomectomy

Septal myomectomy is only considered where patients are resistant to medical therapy for symptoms of
HOCM.
73110

Rate this question:

Next Question

Previous Question Tag Question

Feedback End Review

Difficulty: Average

Peer Responses %

Q. Answered Flagged

Q1

Q2

Q3

Q4

https://mypastest.pastest.com/Secure/TestMe/TimedBrowser/1098274#Top 2/3
8/25/22, 10:01 PM MyPastest

0:00:00/03:00:00

A 21-year-old woman presents to her General Practitioner with complaints of diarrhoea over the past six
months. She has lost six kg in weight. She is opening her bowels 3–4 times per day, and there is no blood or
mucus. Also of note is an intensely itchy vesicular rash over the buttocks and her upper posterior thighs.

Investigation Result Normal value

Haemoglobin (Hb) 98 g/l 115–155 g/l

White cell count (WCC) 7.1 × 10 9/l 4–11 × 10 9/l

Platelets (PLT) 193 × 10 9/l 150–400 × 10 9/l

Sodium (Na +) 142 mmol/l 135–145 mmol/l

Potassium (K +) 3.8 mmol/l 3.5–5.0 mmol/l

Creatinine (Cr) 85 µmol/l 50–120 µmol/l

Calcium 2.05 mmol/l 2.1–2.65 mmol/l

What is the most likely diagnosis?

A Chronic pancreatitis

B Coeliac disease

C Crohn’s disease

D Lactose intolerance

E Ulcerative colitis

Explanation 

B Coeliac disease

This patient has diarrhoea, anaemia and low calcium level, suggesting small bowel malabsorption. The rash
seen here is suggestive of dermatitis herpetiformis, which is associated with hypersensitivity to gluten and
resolves, as does diarrhoea when patients switch to a gluten-free diet. Anti-tissue transglutaminase (TTG)

https://mypastest.pastest.com/Secure/TestMe/TimedBrowser/1098274#Top 1/4
8/25/22, 10:01 PM MyPastest

antibodies are indicated next, to confirm the diagnosis, potentially followed by a small bowel biopsy.

A Chronic pancreatitis

Chronic pancreatitis leads to diarrhoea because of malabsorption related to pancreatic exocrine dysfunction. It
does not cause the vesicular rash seen in this patient.

C Crohn’s disease

Crohn’s disease is associated with anaemia and may lead to decreased calcium and vitamin D levels. It is not,
however, associated with dermatitis herpetiformis, the likely cause of the rash seen here.

D Lactose intolerance

Lactose intolerance may be associated with diarrhoea. It is not, however, associated with the rash seen here
that is highly suspicious of dermatitis herpetiformis.

E Ulcerative colitis

Ulcerative colitis is associated with symptoms of lower gastrointestinal inflammation, which may include
diarrhoea with blood and mucus and left-sided abdominal pain. However the anaemia and low calcium level
suggest small bowel malabsorption which is more associated with coeliac disease.
73146

Rate this question:

Next Question

Previous Question Tag Question

Feedback End Review

Difficulty: Average

Peer Responses %

https://mypastest.pastest.com/Secure/TestMe/TimedBrowser/1098274#Top 2/4
8/25/22, 10:01 PM MyPastest

0:00:00/03:00:00

A 72-year-old woman presents to the Emergency Department with palpitations and anxiety that have built
up over the past two weeks. Around three and a half weeks ago, she underwent a contrast computed
tomography of the abdomen for evaluation of diverticular disease. Palpation of the neck reveals a
multinodular goitre.

Investigation Result Normal value

Thyroid-stimulating hormone (TSH) < 0.05 0.2–4.2 U/ml

What is the most likely diagnosis?

A Anaplastic thyroid cancer

B Contrast-induced thyroiditis

C Graves’ disease

D Toxic adenoma

E Toxic multinodular goitre

Explanation 

B Contrast-induced thyroiditis

Contrast-induced thyroiditis is described 2–12 weeks after an initial investigation. It is more commonly seen in
the elderly and patients with incipient thyroid disease, including those with multinodular goitre. It is thought
that the iodine load may precipitate symptoms which are usually mild in nature. It is recommended that in the
elderly and those at risk of thyrotoxicosis, thyroid function should be monitored after a contrast investigation.

A Anaplastic thyroid cancer

Anaplastic thyroid cancer is rapidly growing and presents in elderly patients. It is not associated with
thyrotoxicosis.

C Graves’ disease

https://mypastest.pastest.com/Secure/TestMe/TimedBrowser/1098274#Top 1/3
8/25/22, 10:01 PM MyPastest

Graves’ disease is associated with a smooth goitre, rather than the multinodular goitre seen here. The anterior
neck is also usually painful to palpation.

D Toxic adenoma

A toxic adenoma is usually characterised by a single lesion, rather than by the multinodular goitre described
here. It is more likely thyroiditis has been precipitated by the iodine load provided by the contrast material.

E Toxic multinodular goitre

The proximity to the contrast computed tomography makes contrast-induced thyroiditis much more likely
than toxic multinodular goitre as the cause of this patient’s symptoms. The iodine load provided by the
contrast material may have precipitated her thyrotoxicosis.
73141

Rate this question:

Next Question

Previous Question Tag Question

Feedback End Review

Difficulty: Average

Peer Responses %

Q. Answered Flagged

Q1

Q2

Q3

https://mypastest.pastest.com/Secure/TestMe/TimedBrowser/1098274#Top 2/3
8/25/22, 10:02 PM MyPastest

0:00:00/03:00:00

A 71-year-old woman, who is usually well, presents to the Emergency Department with back pain of 48
hours duration which has not been controlled with regular ibuprofen and paracetamol. She smokes 20
cigarettes per day and underwent a total hysterectomy at the age of 40. Examination reveals pain over the
upper lumbar spine.
A plain X-ray is shown below:


What is the most likely cause of the X-ray changes?

A Bone metastases

B Myeloma

C Osteomalacia

D Osteoporosis

https://mypastest.pastest.com/Secure/TestMe/TimedBrowser/1098274#Top 1/4
8/25/22, 10:02 PM MyPastest

E Paget’s disease

Explanation 

D Osteoporosis

Osteoporosis leads to sudden onset of back pain and wedge fractures typical of that seen in this patient.
Bone densitometry would be an obvious next step and bone protection should be considered.
Bisphosphonates are the first-line intervention of choice according to the National Institute for Health and
Care Excellence guidelines. Smoking and early menopause are both risk factors for osteoporosis
development.

Image credit: By Dr Henry Knipe, CC BY NC SA 3.0

A Bone metastases

Bone metastases would seem unlikely in a patient who has no other symptoms of an underlying malignancy.
Isolated wedge fractures are also more suggestive of osteoporosis.

B Myeloma

Myeloma may be associated with a wedge fracture. However, the clinical picture of relatively good health
until this point does not support myeloma as the underlying diagnosis.

C Osteomalacia

Osteomalacia occurs due to vitamin D deficiency and is associated with long bone pain and proximal
myopathy. It is not associated with the isolated wedge fracture seen in this patient.

E Paget’s disease

Paget’s disease leads to bone resorption and disordered formation of new bone. This most commonly affects
the hip and would not be responsible for the features seen in this patient.
73155

Rate this question:

Next Question

Previous Question Tag Question

https://mypastest.pastest.com/Secure/TestMe/TimedBrowser/1098274#Top 2/4
8/25/22, 10:02 PM MyPastest

0:00:00/03:00:00

A 54-year-old woman who has type II diabetes is prescribed co-amoxiclav for a diabetic foot ulcer. She has
been taking the medication for 20 days when she presents to the Emergency Department with jaundice.
Other medications include dapagliflozin, liraglutide, amlodipine, ramipril and aspirin. Her blood pressure is
122/72 mmHg, and her heart rate is 72 bpm and regular. She has jaundiced sclerae. Her abdomen is soft,
with vague tenderness in the right-upper quadrant.

Investigation Result Normal value

Haemoglobin (Hb) 132 g/l 115–155 g/l

White cell count (WCC) 8.2 × 10 9/l 4–11 × 10 9/l

Platelets (PLT) 122 × 10 9/l 150–400 × 10 9/l

Sodium (Na +) 142 mmol/l 135–145 mmol/l

Potassium (K +) 4.1 mmol/l 3.5–5.0 mmol/l

Creatinine (Cr) 120 µmol/l 50–120 µmol/l

Alanine transaminase (ALT) 275 U/l 1–40 U/l

Alkaline phosphatase 493 U/l 30–130 U/l

Bilirubin 181 µmol/l < 21 µmol/l

What is the most likely cause of this patient’s liver dysfunction?

A Co-amoxiclav

B Dapagliflozin

C Hepatitis A

D Hepatitis B

E Ramipril

Explanation 

https://mypastest.pastest.com/Secure/TestMe/TimedBrowser/1098274#Top 1/3
8/25/22, 10:02 PM MyPastest

A Co-amoxiclav

Co-amoxiclav was the subject of an alert from the Medicines and Healthcare products Regulatory Agency
(MHRA) for cholestatic jaundice, which may occur during or shortly after therapy with the antibiotic. It is more
commonly seen in men and in patients over 65 years of age. Therapy for over 14 days is not recommended
because of the risk of jaundice occurring. Cholestasis associated with co-amoxiclav rarely has serious long-
term consequences and resolves after discontinuation of therapy.

B Dapagliflozin

Abnormal liver function is not reported as a consequence of dapagliflozin therapy. Indeed the drug may
actually reduce liver fat and lead to improvement in transaminase levels.

C Hepatitis A

Hepatitis A is associated with flu-like symptoms and a marked rise in transaminase levels, rather than the
cholestatic picture seen here.

D Hepatitis B

Hepatitis B is rarely associated with acute hepatitis, and a hepatitic, rather than cholestatic, picture on liver
function testing would be expected.

E Ramipril

Cholestatic jaundice is rarely seen in patients taking ramipril, and the proximity of symptoms to use of co-
amoxiclav makes this a much more likely cause of jaundice in this patient.
73118

Rate this question:

Next Question

Previous Question Tag Question

Feedback End Review

Difficulty: Average

Peer Responses %

https://mypastest.pastest.com/Secure/TestMe/TimedBrowser/1098274#Top 2/3
8/25/22, 10:02 PM MyPastest

0:00:00/03:00:00

A 62-year-old woman presents to the Dermatology Clinic with a rapidly growing skin lesion on her upper
chest. It has grown from a small pimple to the current lesion over the past 3–4 months.

Investigation:


What is the most likely diagnosis?

A Actinic keratosis

B Basal cell carcinoma

C Keratoacanthoma

D Papilloma

E Squamous cell carcinoma

Explanation 

C Keratoacanthoma

https://mypastest.pastest.com/Secure/TestMe/TimedBrowser/1098274#Top 1/3
8/25/22, 10:02 PM MyPastest

Keratoacanthomas have a similar appearance to squamous cell carcinomas but grow much more rapidly. They
also spontaneously resolve, which is not seen with squamous cell carcinomas. Histologically, they may well
be difficult to differentiate from squamous cell carcinoma. The aetiology is thought to be similar to that for
squamous cell cancers, with exposure to tar and pitch also thought to increase the risk of lesions occurring.
Excision is the intervention of choice, with medical interventions utilising retinoids or topical treatments, such
as 5-fluorouracil, only considered if surgery is not possible.

Image credit: By Jmarchn - CC BY-SA 3.0, https://commons.wikimedia.org/w/index.php?curid=15600515.

A Actinic keratosis

Actinic keratoses appear as rough, scaly skin patches over sun-exposed areas which do not fit with this
patient's presentations. Over time, they have an increased risk of developing into squamous cell carcinomas.

B Basal cell carcinoma

Early basal cell carcinomas are pearlescent, with overlying telangiectasia which does not fit with this patient's
presentations. They later develop into lesions with the classical rodent ulcer appearance.

D Papilloma

Papillomas are warty lesions. The skin lesion seen here with a necrotic keratinised core is much more
consistent with a keratoacanthoma.

E Squamous cell carcinoma

Squamous cell carcinomas are more slow-growing than keratoacanthomas but also present as raised lesions
with a necrotic keratinised core. The major difference is that keratoacanthomas both grow rapidly and resolve
over time.
73131

Rate this question:

Next Question

Previous Question Tag Question

Feedback End Review

Difficulty: Average

Peer Responses %

https://mypastest.pastest.com/Secure/TestMe/TimedBrowser/1098274#Top 2/3
8/25/22, 10:02 PM MyPastest

0:00:00/03:00:00

A 47-year-old man visiting from Brazil presents to the Infectious Diseases Clinic with a large ulcer on his left
calf associated with surrounding erythema. He says it began as an area of reddened skin and does not cause
him significant pain.

What is the most useful investigation?

A Bone marrow aspiration

B Dental scraping testing

C Ks30 dipstick testing

D Leishmaniasis direct agglutination test

E Punch biopsy and culture from the edge of the ulcer

Explanation 

E Punch biopsy and culture from the edge of the ulcer

This patient’s symptoms fit with cutaneous leishmaniasis, which often occurs as a single lesion in cases
associated with the New World such as this patient’s ulcer acquired in Brazil. In this case, without systemic
disease, serology testing is either negative or may represent past cleared infection. As such, a biopsy from the
edge of the lesion, coupled with Leishmania microscopy and culture, is a more useful investigation.
Miltefosine appears effective against both visceral and cutaneous leishmaniasis and is a potential intervention
here.

A Bone marrow aspiration

Bone marrow aspiration is useful in patients with generalised advanced visceral leishmaniasis where there is
usually some evidence of bone marrow infiltration in the form of anaemia/pancytopenia. It would not be
appropriate for this patient.

B Dental scraping testing

https://mypastest.pastest.com/Secure/TestMe/TimedBrowser/1098274#Top 1/3
8/25/22, 10:02 PM MyPastest

Dental scrapings to diagnose leishmaniasis are only useful in patients with generalised mucocutaneous
disease. It is not of value here.

C Ks30 dipstick testing

Similar to the direct agglutination test, Ks30 testing is not of value in diagnosing limited cutaneous
leishmaniasis.

D Leishmaniasis direct agglutination test

The leishmaniasis direct agglutination test remains positive for many years after the systemic infection has
cleared and is not useful in diagnosing localised skin disease. As such, it is not of value here.
73168

Rate this question:

Next Question

Previous Question Tag Question

Feedback End Review

Difficulty: Average

Peer Responses %

Q. Answered Flagged

Q1

Q2

Q3

Q4

https://mypastest.pastest.com/Secure/TestMe/TimedBrowser/1098274#Top 2/3
8/25/22, 10:02 PM MyPastest

0:00:00/03:00:00

A 25-year-old man presents to the Emergency Department with rectal pain associated with passage of blood
and mucus. He admits to unprotected sexual intercourse with three men over the past three weeks.
Proctoscopy reveals a rectal ulcer.

Investigation Result Normal value

Haemoglobin (Hb) 132 g/l 115–155 g/l

White cell count (WCC) 7.0 × 10 9/l 4–11 × 10 9/l

Platelets (PLT) 191 × 10 9/l 150–400 × 10 9/l

C-reactive protein (CRP) 32 mg/l < 10 mg/l

Potassium (K +) 3.0 mmol/l 3.5–5.0 mmol/l

Creatinine (Cr) 75 µmol/l 50–120 µmol/l

What is the most likely diagnosis?

A Chancroid

B Donovanosis

C Inflammatory bowel disease

D Lymphogranuloma venereum

E Mycobacterium chimaera infection

Explanation 

D Lymphogranuloma venereum

This patient has proctitis with ulceration following unprotected anal intercourse. Lymphogranuloma
venereum is caused by the L1, L2 or L3 serovars of Chlamydia trachomatis and is recognised as a cause of
proctitis. Unlike standard chlamydia infection, which affects squamocolumnar epithelial cells, these serovars

https://mypastest.pastest.com/Secure/TestMe/TimedBrowser/1098274#Top 1/3
8/25/22, 10:02 PM MyPastest

infect mononuclear phagocytes. Around 1% of rectal swabs of men who have sex with men and attend a
Sexual Health Clinic are positive for L1, L2 or L3 C. trachomatis. Doxycycline is the recommended
intervention.

A Chancroid

Chancroid is a rare cause of sexually transmitted infection in the United Kingdom and Europe and is caused by
Haemophilus ducreyi. Treatment options include single doses of azithromycin or ceftriaxone.

B Donovanosis

Donovanosis is a tropical sexually transmitted infection, also known as granuloma inguinale. It is recognised
as a cause of anal ulceration.

C Inflammatory bowel disease

Inflammatory bowel disease is unlikely here, given the proximity to unprotected sexual intercourse. In active
ulcerative colitis, a greater rise in CRP level might also be expected.

E Mycobacterium chimaera infection

Mycobacterium chimaera infection is a low-virulence Mycobacterium which primarily leads to infection after
cardiac valve surgery. The risk of being infected with this has been linked to a device used to heat and cool
the blood during some types of heart surgery, making it unlikely in this patient.
73163

Rate this question:

Next Question

Previous Question Tag Question

Feedback End Review

Difficulty: Average

Peer Responses %

https://mypastest.pastest.com/Secure/TestMe/TimedBrowser/1098274#Top 2/3
8/25/22, 10:02 PM MyPastest

0:00:00/03:00:00

A 73-year-old woman presents to her General Practitioner, complaining of an erythematous rash affecting
the area around the nose over the past few months. She said that it has slowly become more prominent.
Initially, the rash was made worse by facial flushing but has now become permanent. The skin is dry around
the nose, with prominent sebaceous glands.
Investigation:


What is the most appropriate initial intervention?

A Oral isotretinoin

B Oral ivermectin

C Oral minocycline

D Pulsed light therapy

E Topical metronidazole

Explanation 

E Topical metronidazole

https://mypastest.pastest.com/Secure/TestMe/TimedBrowser/1098274#Top 1/3
8/25/22, 10:02 PM MyPastest

Topical metronidazole is considered a first-line option for the treatment of rosacea, the diagnosis here, with
erythematous and dry skin, prominent papules and sebaceous glands around the nose. Azelaic acid gel is
potentially more effective than metronidazole, although skin hypersensitivity may limit its use in some
patients.
Image credit: By Michael Sand, Daniel Sand, Christina Thrandorf, Volker Paech, Peter Altmeyer, Falk G
Bechara, CC BY 2.5.

A Oral isotretinoin

Oral isotretinoin is a vitamin A analogue which is used for the treatment of rosacea that is refractory to other
interventions.

B Oral ivermectin

Topical, rather than oral, ivermectin is recommended for the treatment of rosacea. Given as a 1% cream, it is
actually superior to metronidazole with respect to outcomes.

C Oral minocycline

Although other tetracyclines, such as doxycycline and oxytetracycline, may be effective in managing rosacea,
minocycline has not been confirmed to be effective as part of the Cochrane review.

D Pulsed light therapy

Pulsed light therapy has been confirmed to be ineffective for the treatment of rosacea, according to a large
Cochrane review.
73129

Rate this question:

Next Question

Previous Question Tag Question

Feedback End Review

Difficulty: Average

Peer Responses %

https://mypastest.pastest.com/Secure/TestMe/TimedBrowser/1098274#Top 2/3
8/25/22, 10:02 PM MyPastest

0:00:00/03:00:00

A 44-year-old woman presents to the Hepatology Clinic with tiredness, lethargy, night sweats and anorexia.
On examination, there are a number of spider naevi and her abdomen is soft and non-tender. Her body mass
index is 22 kg/m 2.

Investigation Result Normal value

Haemoglobin (Hb) 101 g/l 115–155 g/l

White cell count (WCC) 8.1 × 10 9/l 4–11 × 10 9/l

Platelets (PLT) 75 × 10 9/l 150–400 × 10 9/l

Sodium (Na +) 142 mmol/l 135–145 mmol/l

Potassium (K +) 3.9 mmol/l 3.5–5.0 mmol/l

Creatinine (Cr) 85 µmol/l 50–120 µmol/l

Alanine transaminase (ALT) 1025 U/l 1–40 U/l

Alkaline phosphatase 205 U/l 30–130 U/l

Autoimmune profile Anti-smooth muscle antibody positive

What is the most likely underlying diagnosis?

A Autoimmune hepatitis

B Hepatitis C

C Primary biliary cholangitis

D Primary sclerosing cholangitis

E Sarcoidosis

Explanation 

A Autoimmune hepatitis

https://mypastest.pastest.com/Secure/TestMe/TimedBrowser/1098274#Top 1/4
8/25/22, 10:02 PM MyPastest

Autoimmune hepatitis fits with the hepatitic picture seen here on blood testing, and the anti-smooth muscle
antibodies also point to this as the underlying diagnosis. Other antibodies seen include anti-nuclear
antibodies and anti-liver–kidney microsomal antibodies (anti-LKM). Corticosteroids are the cornerstone of
therapy, with azathioprine added in for the long term as a steroid-sparing agent. Unfortunately, cirrhosis
develops in up to 50% of patients with autoimmune hepatitis.

B Hepatitis C

Hepatitis C is associated with positive autoantibodies. However, it is anti-nuclear antibodies that are the most
prevalent and there is no indication given of risk factors for hepatitis C.

C Primary biliary cholangitis

Primary biliary cholangitis is associated with an obstructive picture on liver function testing and with anti-
mitochondrial, not anti-smooth muscle, antibodies.

D Primary sclerosing cholangitis

Primary sclerosing cholangitis is associated with an obstructive liver function picture and the prevalence of
inflammatory bowel disease in patients with primary sclerosing cholangitis approaches 90%.

E Sarcoidosis

Anti-smooth muscle antibodies are seen in sarcoidosis. However, this is usually in patients with symptoms of
sarcoidosis who also have abnormal liver function tests. Here the primary problem is hepatic dysfunction.
73150

Rate this question:

End Session

Previous Question Tag Question

Feedback End Review

Difficulty: Average

Peer Responses %

https://mypastest.pastest.com/Secure/TestMe/TimedBrowser/1098274#Top 2/4

You might also like